Download as pdf or txt
Download as pdf or txt
You are on page 1of 176

Physics (042) / XI / TERM-2 /2021-22

KENDRIYA VIDYALAYA SANGATHAN


AHMEDABAD REGION

STUDY MATERIAL
TERM-2
CLASS XI
PHYSICS (042)

Session 2021-22
Physics (042) / XI / TERM-2 /2021-22

INSPIRATION
Shri(Dr) Jaydeep Das
Deputy Commissioner

KVS RO Ahmedabad

Smt Shruti Bhargava Smt Vinita Sharma


Assistant Commissioner Assistant Commissioner

KVS RO Ahmedabad KVS RO Ahmedabad

MENTOR
Shri Avijit Panda
Principal

Kendriya Vidyalaya,Sabarmati
Physics (042) / XI / TERM-2 /2021-22

CONTENT DEVELOPEMENT TEAM


S.NO. Name of Teacher Designation Name of KV
1 Mrs. Vandana Badola PGT Physics NO.1 SHAHIBAUG AHMEDABAD
2 Mr. R N Singh PGT Physics NO.1 SHAHIBAUG AHMEDABAD
3 Mr. Bhupesh Kumar Nagoda PGT Physics AHMEDABAD CANTT
4 Mr. R H Parmar PGT Physics SAC AHMEDABAD
5 Mr. M P Dabi PGT Physics CRPF GANDHINAGAR
6 Mr. T C Agrawal PGT Physics ONGC CHANDKHEDA
7 Mr. Gurmeet Singh PGT Physics HIMMATNAGAR
8 Mrs. Anju Kumari PGT Physics AFS WADSAR
9 Mr. Praveen Nogia PGT Physics BSF DANTIWADA
10 Mr. S K Devrani PGT Physics SEC 30 GANDHINAGAR
11 Mr. Sandeep Kumar Koli PGT Physics SEC 30 GANDHINAGAR
12 Mr. Bhoor Singh Meena PGT Physics CANTT GANDHINAGAR
13 Mr. Shabbir Hingorja PGT Physics NO 1 AFS BHUJ
14 Mrs. Anjana PGT Physics TRANSFERRED
15 Mr. Akhilesh Suryavanshi PGT Physics NO.3 AFS MAKARPURA
16 Mr. Deepak Kumar Singhal PGT Physics AFS NALIYA
17 Mr. Viswanath PGT Physics DHRANGADHRA
18 Mr. Yaksh Deep PGT Physics TRANSFERRED
19 Mr. Sahil Aneja PGT Physics NO.2 BHUJ CANTT
20 Mr. Ranjay Kumar Sharma PGT Physics RAJKOT
21 Mr. Om Prakash Yadav PGT Physics RAJKOT
22 Mr. Mukesh Kumar PGT Physics TRANSFERRED
23 Ms. Suryakant Vats PGT Physics TRANSFERRED
24 Mr. Ram PGT Physics INS VALSURA
25 Dr Neeraj Singh PGT Physics AFS II JAMNAGAR
26 Mr. Umesh Kumar PGT Physics TRANSFERRED
27 Mr. Pinkesh Rathod PGT Physics NO.3 AFS MAKARPURA
28 Mr. Naveen Kumar PGT Physics DIU
29 Mr. Rakesh Kumar PGT Physics NO 1 BARODA
30 Mr. S V Singh PGT Physics EME BARODA
31 Mr. Sudhir Bhootda PGT Physics EME BARODA
32 Mr. Sanjeev Chaddha PGT Physics NO 1 AFS JAMNAGAR
33 Mr. J K Jain PGT Physics RAILWAY GANDHIDHAM
34 Mrs. Sangeeta Arora PGT Physics ONGC BARODA
35 Mr. Chitresh Pandya PGT Physics ONGC CHANDKHEDA
36 Mr. C M Sharma PGT Physics ONGC ANKLESHWAR
37 Mr. V K Pathak PGT Physics NO 1 SURAT
38 Mr. Pawan Kumar PGT Physics KRIBHCO SURAT
39 Mr. R M Shukla PGT Physics ONGC SURAT
40 Mr. A P Singh PGT Physics SILVASSA
41 Mr. P K Sah PGT Physics VV NAGAR
42 Mrs. Anju PGT Physics TRANSFERRED
43 Mr. Kamlesh Kumar PGT Physics TRANSFERRED
44 Mr. Balwant Kumar PGT Physics AHMEDABAD CANTT
45 Mr. Vikas Sharma PGT Physics SABARMATI
Physics (042) / XI / TERM-2 /2021-22

PHYSICS
Class-XI
INDEX
S.No Particulars
1 SYLLABUS (TERM-2)

2 Question Paper Design (TERM-2)

3 Chapter-9 : Mechanical Properties of Solids

4 Chapter-10 : Mechanical Properties of Fluids

5 Chapter-11 : Thermal Properties of Matter

6 Chapter-12 : Thermodynamics
7 Chapter-13 : Kinetic Theory

8 Chapter-14 : Oscillations

9 Chapter-15 : Waves

10 Two Sample Papers (TERM-2) with Answer key

*Every Chapter Contains:


1. Gist of Lesson
2. 2 Marks Questions with Answer
3. 3 Marks Questions with Answer
4. Case Study Based Questions with Answer key
5. Extra Question: 1 Mark Questions with Answer
Physics (042) / XI / TERM-2 /2021-22

SYLLABUS
TERM-2
Session 2021-22
PHYSICS, CLASS-XI
UNIT Chapters Marks
Unit VII Properties of Bulk Matter
CHAPTER 9 - Mechanical Properties of Solids
CHAPTER 10 - Mechanical Properties of Fluids
CHAPTER 11 - Thermal Properties of Matter
Unit VIII Thermodynamics 23
CHAPTER 12 - Thermodynamics
Unit IX Behaviour of Perfect Gases & Kinetic Theory
of Gases
CHAPTER 13 - Kinetic Theory of Gases

Unit X Oscillations and Waves


CHAPTER 14 – Oscillations 12

CHAPTER 15 - Wave

Total 35
Physics (042) / XI / TERM-2 /2021-22

QUESTION PAPER DESIGN


TERM-2 EXAM
2021-22

Time Allowed : 2 Hours M.M. : 35


S.No. Type of Question No. of Marks Total
Question Marks

1. SECTION A 3 (Internal 2 each 6


Choice in One
Short Answer Type (SA 1)
Question)

2. SECTION B 8 (Internal 3 each 24


Choice in Two
Short Answer Type (SA 2)
Questions)

3. SECTION C 1 (5 MCQ) 5 5
Case Study Based (in MCQ
Form)

Total 12 …………………. 35
CHAPTER -9
Mechanical Properties of Solids
GIST OF CHAPTER
• Deforming force:- A force acting on a body which produces change in its shape of
body instead of its state of rest or uniform motion of the body.
• Elasticity:-The property of matter by virtue which it regains its original shape and
size, when the deforming forces are removed is called elasticity.
• Plasticity:- The inability of a body to return to its original shape and size, when the
deforming forces are removed is called plasticity.
• Hooke’s law:- when a wire is loaded within elastic limit, the extension produced in
wire is directly proportional to the load applied.
OR
Within elastic limit stress α strain
Stress = Constant
Strain
• Stress :- Restoring force set up per unit area when deforming force acts on the body
Stress = Restoring force
Area
S.I Unit of stress = N/m2 or Pascal (Pa)
Dimensional formula = M L-1T-2
Types of stress:- Normal stress and tangential stress
Normal stress
a. Tensile stress (When there is an increase in dimension of the body along the direction of
normal stress

b. Compression stress (when there is decrease in dimension) Stress


c. Hydraulic/ volumetric stress( when deforming force acts on the body normally from all
directions)
2.. Tangential stress (When deforming force acts tangential to the surface of body )

Strain:- The ratio of change in dimension to the original dimension is called strain

It has no unit
Longitudinal strain=Change in Length/ Original Length
Types of strain:-

Volumetric strain = Change in Volume/ Original Volume

Shearing Strain = φ =x/L , Where x = lateral displacement of the face


on which tangential force is applied and L is the height of the face

Hooke’s L aw:- Within elastic limit, stress α strain


Stress / Strain = Constant (Modulus of Elasticity)
Modulus of elasticity are of 3 types.
1. Young’s Modulus (Y) = Normal stress/ LongitudinalStrain = F /A . l/∆l
2. Bulk Modulus (K) = Normal stress /Volumetric Strain
3. Modulus of rigidity modulus (ƞ) =Tangentialstress /ShearingStrain
• Compressibility : the reciprocal of bulk modulus of a material is called its
compressibility
Compressibility = 1/K
Stress – Strain- diagram

• Proportionality limit(P) – The stress at the limit of proportionality point P is known


as proportionality limit
• Elastic limit - the maximum stress which can be applied to a wire so that on
unloading it return to its original length is called the elastic limit
• Yield point(Y)- The stress, beyond which the length of the wire increase virtually for
no increase in the stress
• Plastic region- the region of stress- strain graph between the elastic limit and the
breaking point is called the plastic region.
• Fracture point or Breaking point(B)- the value of stress corresponding to which the
wire breaks is called breaking point
• Work done in stretching a wire per unit volume/energy sored per unit volume of
specimen
= ½ x stress x strain
• Elastic after effect:- The delay in regaining the original state by a body after the
removal of the deforming force is called elastic after effect.
Elastic fatigue:- the loss in strength of a material caused due to repeated alternating strains
to which the material is subjected.
CASE STUDY BASED QUESTIONS:
CASE 1. TOPIC: If a body gets deformed under the action of the external force then at each
section an internal force of reaction is setup which tends to restore the body into its
original state. The internal restoring force setup per unit area of cross sectional.
Stress = Applied force / Area
The ratio of change in any dimension produced in the body to the original dimension is
called strain, with in the elastic limit the stress is directional proportional to strain. The
ratio of stress and strain is constant and called modulus of elasticity.
(i). A force which changes the shape and size of a body is called
(A) Restoring force
(B) deforming force
(C) stress
(D) Strain
(ii). The value of Young’s Modulus for a perfectly rigid body is
(A) Infinite
(B) Zero
(C) One
(D) None of the above
(iii). The internal restoring force set per unit area of a deformed body is called
(A) Strain
(B) Young’s modulus
(C) stress
(D) Bulk Modulus
(iv). The slope of stress-strain graph gives
(A) Modulus of elasticity
(B) Pressure
(C) Poisson’s ratio
(D) All the above
(v). Stress has same dimension as
(A) Strain
(B) Pressure
(C) Momentum
(D) Elasticity
CASE 2. TOPIC: Elastic limit is the upper limit of deforming force up to which, if deforming
force is removed, the body regains its original form completely and beyond which if
deforming force is increased, the body loses its property of elasticity and gets
permanently deformed. Elastic limit is the property of a body whereas elasticity is the
property of material of a body.
(i). Elasticity is shown by material because inter-atomic or inter-molecular forces
(A) increases when a body is deformed
(B) decreases when a body is deformed
(C) remains same when a body is deformed
(D) becomes non zero when a body is deformed
(ii). The maximum load a wire can withstand is reduced to half of its original, length will
(A) Be double
(B) Be half
(C) Be four times
(D) Remain same
(iii). Stress-strain curves for the materials A and B are shown below. Then,

(A) A is brittle material


(B) B is ductile material
(C) B is brittle material
(D) Both (a) and (b)
(iv). A and B are two wires. The radius of A is twice that of B. They are stretched by the
same load, then the stress on B is
(A) equal to that on A
(B) four times that on A
(C) two times that on A
(D) half that on A
(v).On suspending a weight Mg, the length l of elastic wire having area of cross-
sectional A, becomes double. The instantaneous stress acting on the wire is
(A) Mg/A
(B) Mg/2A
(C) 2 Mg/A
(D) 4 Mg/A
ANSWERS(CASE STUDY BASED QUESTIONS):
ANS (i) ANS (ii) ANS (iii) ANS (iv) ANS (v)
CASE 1 B A C A B
CASE 2 D D C B C
TWO MARK QUESTIONS:

1. QUESTION: The load versus elongation graph for four wires of same material is
shown. The line represents the thickest wire?

𝐹𝐿
ANSWER: Y= STRESS/STRAIN ⇒ Y=𝐴Δ𝐿

𝐹 𝐴
As, Y is constant for all wires so , 𝛼
Δ𝐿 𝐿

𝐹
for given same length 𝛼 A ( area of cross section of wire)
Δ𝐿

slope of curve 𝛼 thickness of wire so, line P shows thick wire.

2. QUESTION: The stress versus strain graphs for wires of two materials A and B are as
shown in the figure. If YA and YB are the Young ‘s modulii of the materials, then
what is the relation between their young modulus.

ANSWER: YA = tan 600 : YB = tan 300

YA / YB = tan 600 / tan 300 so, YA = 3YB

3. QUESTION: A uniform cube is subjected to volume compression. If each side is


decreased by 1%, then what will be the bulk strain?

Δ𝑉 Δ𝐿
ANSWER: V= L3 or, =3x = 3X1/100=0.03
𝑉 𝐿

Δ𝑉
= 0.03
𝑉

4. QUESTION: Steel and copper wires of same length and area of cross-section are
stretched by the same weight one after the other. Young’s modulus of steel and
copper are 2X1011 N/m2 and 1.2x1011 N/m . What is the ratio of increase in their
length?

1
ANSWER: Δ𝐿 𝛼 𝑌

so, ΔLS / ΔLC =1.2x1011/ 2X1011 = 3/5

5. QUESTION: When a pressure of 100 atmosphere is applied on a spherical ball, then


its volume reduces to 0.01%. What will be the bulk modulus of the material of the
rubber in dyne/cm2 ?

Δ𝑉 0.01
ANSWER: B= P/ = 100/ = 10 6 atm = 1011 N/m2 = 1012 dyne/cm2
𝑉 100

6. QUESTION: For the given graph, which region obeyed Hooke's law ?

ANSWER: Since OE is a straight line so, stress vs strain 𝛼 Hooke's law is obeyed in
the region OE of the graph.

7. QUESTION: The diagram below shows the change in the length X of a thin uniform
wire caused by the application of stress F at two different temperatures T1 and T2
Which is the higher temperature?

ANSWER: When same stress is applied at two different temperatures, the increase
in length is more at higher temperature. Thus T1 > T2

8. QUESTION: What per cent of length of wire increases by applying a stress

of 1 kg weight/mm2 on it? (Y = 1 × 1011 N/m2 and 1 kg weight = 9.8 newton)


ANSWER:

9. QUESTION: Two identical solid balls, one of ivory and the other of wet-clay, are

dropped from the same height on the floor. Which one will rise to a greater height
after striking the floor and why?

ANSWER: Ans. Since, ivory ball is more elastic than wet-clay ball, therefore, ivory
ball tries to regain its original shape quickly. Hence, more energy and momentum is
transferred to the ivory ball in comparison to the wet clay ball and therefore, ivory
ball will rise higher after striking the floor.

10. QUESTION: Water is more elastic than air. Why?

ANSWER: Since volume elasticity is the reciprocal of compressibility and since air is
more compressible than water hence water is more elastic than air.

THREE MARK QUESTIONS:

1. QUESTION: The approximate depth of an ocean is 2700 m. The compressibility of


water is 45.4 × 10–11 Pa–1 and density of water is 103 kg/m3.What fractional
compression of water will be obtained at the bottom of the ocean?

ANSWER: Compressibility of water,

K = 45.4 × 10–11 Pa–1

density of water P = 103 kg/m3

depth of ocean, h = 2700 m


Δ𝑉
We have to find =?
𝑉

As we know, compressibility,

Δ𝑉/𝑉
K = 1/B = 𝑃

SO, Δ𝑉/𝑉 = K h ρ g = 45.4 × 10–11 × 103 × 10 × 2700 = 1.2258 × 10–2

2. QUESTION: Define :- 1) Elastic Body 2) Plastic Body 3) Elasticity.

ANSWER: 1) Elastic Body → A body which completely regains its original


configuration immediately after the removal of deforming force on it is called elastic
body. eg. Quartz and phosphor Bronze.

2) Plastic Body → A body which does not regain its original configuration at all on

the removal of deforming force, howsoever the deforming force may be is called

plastic body eg:- Paraffin wax.

3) Elasticity → The property of the body to regain its original configuration, when

the deforming forces are removed is called plasticity.

3. QUESTION: In the following stress – strain curve, which has: -

1) Greater young’s Modulus 2) More Ductility 3) More Tensile strength.

ANSWER: 1) Since young’s Modulus is given by the slope of stress – strain graph,
since slope of A is more than that of B, hence it has greater young’s Modulus.

2) Ductility is the extent of plastic deformation, and it is greater for A.

3) Tensile strength is the direct measure of stress required, from by graph, it is

greater for A.
4. QUESTION: A wire stretches by a certain amount under a load. If the load and radius
both are increased to four times, find the stretch caused in wire.

ANSWER: Young's modulus Y= WL/A𝓵


Elongation 𝓵=WL/AY/
Elongation 𝓵 =WL/ πr2 Y ..................(1) (∵A=πr2)
When both load and radius are increased to four times, the elongation becomes,
𝓵′ = 4W L/ (4r)2 π Y
𝓵′ = W L / 4 π r2 Y
So, from (1)
𝓵′ = 𝓵 /4
5. QUESTION: Two wires are made of same material and have the same volume.
However, wire 1 has cross-sectional area A and wire 2 has cross-sectional area 3A. If
the length of wire 1 increases by ΔL on applying force F, how much force is needed
to stretch wire 2 by same amount?

𝐹𝐿 𝐹 𝐴𝐿 𝐹𝑉
ANSWER: ⇒ Y=𝐴Δ𝐿 = 𝐴2 Δ𝐿 = 𝐴2 Δ𝐿

So F 𝛼 A2

Thus F’ 𝛼 (3A)2

So F’ = 9F

6. QUESTION: A wire of diameter 1mm breaks under a tension of 1000 N. Another


wire, of same material as that of the first one, but of diameter 2 mm, how much
tension is required to break it?

ANSWER: Breaking tension 𝛼 r2


𝑇1 𝑟 1
So = (𝑟1 )2 (1|2) 2 = 4
𝑇2 2

Hence T2 = 4 T1 = 4x1000 = 4000N

7. QUESTION: The compressibility of water is 4x10-5 per unit atmospheric pressure.


What will be the decrease in volume of 100 cm3 of water under a pressure of 100
atmosphere?
𝛥𝑉
ANSWER: As Compressibility =
𝑝𝑉

𝛥𝑉
So, 4x10-5 = 100 𝑋 100 or ΔV = 0.4 cm3

8. QUESTION: (a) Write the unit of Bulk modulus. Is it a vector or a scaler quantity?

(b) A cube is subjected to a uniform volume compression. If the side of cube


decreases by 2%, what is the value of Bulk strain?

ANSWER: (a) The SI unit of Bulk modulus is Nm-2 or Pascal(Pa). It is a scaler quantity.
𝛥𝑉 𝛥𝑙 2
(b) As V = 𝑙 3 So =3 = 3 (100) = 0.06
𝑉 𝑙

9. QUESTION: Figure shows the strain-stress curve

for a given material. What are (a) Young’s modulus and

(b) approximate yield strength for these materials?

ANSWER:

10. QUESTION: Two wires of diameters 0.25 cm, one made of steel and the other made
of brass are loaded as shown in figure. The unloaded length of steel wire is 1.5 m
and that of brass Wire is 1.0 m. Compute the elongations of the steel and the
brass wires.
ANSWER:

11. QUESTION: Draw stress-strain curve for steel rod under load and explain each term.

ANSWER:

From the graph, we can see that in the region between

O to A, the curve is linear. In this region, Hooke’s law

is obeyed. The body regains its original dimensions

when the applied force is removed. In this region, the

solid behaves as an elastic body.

In the region from A to B, stress and strain are not proportional.

The point B in the curve is known as yield point (also known as elastic limit) and

the corresponding stress is known as yield strength (σy) of the material.


The portion of the curve between B and D shows this. When the load is removed,
say at some point C between B and D, the body does not regain its original
dimension. In this case, even when the stress is zero, the strain is not zero. The
material is said to have a permanent set. The deformation is said to be plastic

deformation. The point D on the graph is the ultimate tensile strength (σu) of the
material. Beyond this point, additional strain is produced even by a reduced applied
force and fracture occurs at point E. If the ultimate strength and fracture points D
and E are close, the material is said to be brittle. If they are far apart, the

material is said to be ductile.

13. QUESTION: State Hook’s Law of elasticity. What are different types modulus of
elasticity? Explain.

ANSWER: Hook’s law: For small deformations the stress and strain are proportional
to each other. This is known as Hooke’s law.

Thus, stress ∝ strain

stress = k × strain,

where k is the proportionality constant and is known as modulus of elasticity.

Modulus of elasticity: The proportional region within the elastic limit of the stress-
strain curve The ratio of stress and strain, is called modulus of elasticity.
𝑠𝑡𝑒𝑟𝑠𝑠
= modulus of elasticity.
𝑠𝑡𝑟𝑎𝑖𝑛

Young’s Modulus:

for a given material, the magnitude of the strain produced

is same whether the stress is tensile or compressive.

The ratio of tensile (or compressive) stress (σ) to

the longitudinal strain (ε) is defined as Young’s modulus and

is denoted by the symbol Y.

Y= σ/ε
Y = (F/A)/(∆L/L) = (F × L) /(A × ∆L)
Bulk modulus: Within elastic limit, the ratio of hydraulic

stress to the corresponding hydraulic strain is called

bulk modulus. It is denoted by symbol B.

B = – p/(∆V/V)

The negative sign indicates the fact that with an increase

in pressure, a decrease in volume occurs.

The reciprocal of the bulk modulus is called compressibility and is denoted by k. It is


defined as the fractional change in volume per unit increase in pressure.

k = (1/B) = – (1/∆p) × (∆V/V)

The bulk moduli for solids are much larger than for liquids, which are again much

larger than the bulk modulus for gases (air).

14. QUESTION: (1) Following are the graphs of elastic materials. Which one corresponds
to that of brittle material?

(2) Which graph corresponds to elastomer?

(3) Why the electric poles given hollow structure?

(4) The ratio stress/strain remains constant for small Deformation. What happens to
this ratio, if deformations are made very large.

ANSWER: (1) Curve (a)

(2) Curve (c)

(3) This is because the hollow shaft is stronger than a solid shaft made from the
same and equal amount of material.

(4) If the deforming force exceeds the elastic limit, the strain increases more

rapidly than stress. Hence the ratio stress/strain decreases.


ADDITIONAL QUESTIONS(ONE MARK QUESTIONS):

1. QUESTION: What is the value of Young’s modulus of a perfectly rigid body ?

ANSWER: For a perfectly rigid body strain produced is zero for the given force
applied, so Y = stress/strain = ∞

2. QUESTION: What is yield point on a stress- strain curve of steel wire?

ANSWER: Yield point is the point, beyond which the wire starts showing large
increase in strain even a small change in the stress.

3. QUESTION: For an equal stretching force F, how the young's modulus (Ys) for steel
and rubber (Yr) are related with each other?

ANSWER: Young's modulus of steel is highest so, Ys > Yr .

4. QUESTION: State Hook’s law of elasticity.

ANSWER: Within elastic limit, stress is directly proportional to strain.

5. QUESTION: Arrange the following in correct increasing order of coefficient of


elasticity: Copper, Steel, Glass and Rubber

ANSWER: Rubber < Glass < Copper < Steel

6. QUESTION: According to Hook’s law of elasticity, if stress is increased, then what


will be the ratio of stress to strain?

ANSWER: The ratio of stress to strain is always constant. If stress is increased, strain
will also increase so that their ratio remains constant.

7. QUESTION: What is the reciprocal of bulk modulus of elasticity is called?

ANSWER: Compressibility

8. QUESTION: If the density of the material increases, how the value of Young`s
modulus changes?

ANSWER: Increases.

9. QUESTION: In a wire of length L, the increase in its length is 𝑙. If the length is


reduced to half, what will be the increase in its length?
ANSWER: Increase in length will be half

10. QUESTION: What is the impact on Young Modulus of Elasticity with rise in
temperature?

ANSWER: Decreases

11. QUESTION: Which of the two forces – deforming or restoring is responsible for
elastic behavior of substance?

ANSWER: Restoring force.

12. QUESTION: What is the Bulk modulus for a perfect rigid body?

ANSWER: bulk modulus for a perfectly rigid body is infinity (∞).

13. QUESTION: What are the factors which affects the elasticity of a substance?

(any two)

ANSWER: (a) Hammering and annealing (b) Change in temperature

(c) Impurity in substance

14 QUESTION: The value of tan (90° – 𝜃) in the graph gives, which physical quantity?

ANSWER: tan (90° – 𝜃) = stress/strain, Young's modulus of elasticity

15 QUESTION: Define elastomers with example

ANSWER: The materials for which strain produced is much larger than the stress
applied, with in the limit of elasticity are called elastomers, e.g., rubber, the elastic
tissue of aorta.
CHAPTER - 10

MECHANICAL PROPERTIES OF FLUIDS

GIST OF LESSION:

➢ Thrust -The total force exerted by the liquid at rest, on any surface in contact
with it is called the thrust of the liquid on that surface .
➢ Pressure P = Thrust / Area = F / A . Its Sl unit is N/m² or Pascal (Pa),
Pressure exerted by a liquid column P = pgh
where, h-height of liquid column, g-acceleration due to gravity. p-density of
liquid.
➢ Pascal's law it states that if an external pressure is applied to an enclosed
liquid, it is transmitted undiminished equally to all other points of the liquid and
to the walls of container.
➢ Atmospheric pressure The pressure exerted by the atmosphere is called
atmospheric pressure.
At sea-level, we have atmospheric pressure = 1.013x105 Nm-2
➢ Absolute Pressure and gauge pressure The total or actual pressure p at a
point is called absolute pressure. Gauge pressure is the difference between
the actual pressure (absolute pressure) at a point and the atmospheric
pressure. Thus,
Pg = P - Pa or P = P a + Pg
Absolute pressure = Atmospheric pressure + Gauge pressure.
➢ Buoyancy It states that when a body is partially or fully dipped into a fluid, the
fluid exerts an upward force on the body called as buoyant force and this
phenomenon is called buoyancy.
➢ Archimedes principle It states that when a body is partially or fully dipped in
a fluid at rest, the fluid exerts an upward force of buoyancy equal to the weight
of the displaced fluid.
➢ Surface tension (S) Surface tension is the property of a liquid due to which
its free surface behaves like stretched elastic membrane and tends to have
least possible surface area.
SI unit of surface tension is Nm-1 or Jm-2 .
➢ Capillarity The phenomenon of rise or fall of a liquid in the capillary tube is
called capillarity.
𝟐𝑺 𝑪𝒐𝒔 𝜽
Height of rise of liquid in capillary is given by h = ,
𝝆𝑹𝒈

where, R=radius of capillary.


➢ Streamline and Laminar Flow Flow of a fluid is said to be streamlined if
each element of the fluid passing through a particular point travels along the
same path, with exactly the same velocity as that of the preceding element.
A special case of streamline flow is laminar flow, in which a fluid has a
steady flow in the form of parallel layer and these do not mix with one another.
➢ A turbulent flow is the one in which the motion of the fluid particles is
disordered or irregular. In such a flow, most of the energy used up in
maintaining the flow, is spent in causing eddies in the fluid and only a small
part of the energy is used for the actual forward flow.
➢ Equation of continuity It states that when an incompressible and non-
viscous fluid flows steadily through a tube of non-uniform cross-section, then
the product of area of cross-section and the velocity of flow is same at every
point in the tube.
a v = constant
➢ Bernoulli's theorem it states that the sum of pressure energy, kinetic energy
and potential energy per unit volume of an incompressible and non-viscous
fluid in a streamlined irrotational flow remains constant at every cross-
sectional throughout the liquid flow.
P + 1/2 𝝆v2 + pgh = constant
➢ Velocity of efflux (Torricelli's theorem) The speed of liquid coming out
through a hole at a depth (h) below the free surface is called velocity of efflux,
v = √2hg
➢ Viscosity It is the property of a fluid due to which an opposing force comes
into play whenever there is relative between its different layers.
➢ Newton's formula for viscous force The viscous drag between two parallel
𝑑𝑣
layers each of area A and having velocity gradient is given by
𝑑𝑥
𝑑𝑣
F=-𝜂A where, 𝜂 is the coefficient of viscosity of the liquid.
𝑑𝑥
➢ Stoke's law It states that the backward dragging force of viscosity acting on a
spherical body of radius r moving with velocity v through a fluid of viscosity 𝜂
is
F = 6 𝜋 𝜂 r v.
➢ Terminal velocity It is the maximum constant velocity attained by a spherical
body while falling through a viscous medium.
The terminal velocity of a spherical body of density p and radius r moving
through a fluid of density p' and viscosity 𝜂 is given by
2 𝑟2
v= (p - p') g
9 𝜂

➢ Reynold's number It is a dimensionless number which determines the nature


of the flow of the liquid. For a liquid of viscosity 𝜂 density p and flowing
through a pipe of diameter D, Reynold's number is given by
NR = pvD / 𝜂
If NR < 1000, the flow streamline or laminar.
If NR > 2000 , the flow is turbulent.
If 1000 < NR < 2000, the flow is unstable. It may change from laminar to
turbulent and vice-versa.

CASE STUDY BASED QUESTIONS:


CASE 1. HYDRAULIC LIFT

A hydraulic lift is a device for moving objects using force created by pressure on
a liquid inside a cylinder that moves a piston upward. Incompressible oil is
pumped into the cylinder, which forces the piston upward. When a valve opens
to release the oil, the piston lowers by gravitational force.
The principle for hydraulic lifts is based on Pascal‘s law for generating force or
motion, which states that pressure change on an incompressible liquid in a
confined space is passed equally throughout the liquid in all directions.
The concept of Pascal‘s law and its application to hydraulics can be seen in the
example below, where a small amount of force is applied to an incompressible
liquid on the left to create a large amount of force on the right.
(i). SI unit of pressure is

(A) Newton

(B) Pascal

(C) Newton /meter

(D) None of these

(ii). Pressure is a

(A) Scalar quantity

(B) Vector quantity

(C) Tensor quantity

(D) Scalar and Vector both

(iii). Principle of Hydraulic Lift is

(A)Pascal ‘s law

(B) Bernoulli principal

(C) Boyles law

(D) Charles law

(iv). Dimension of Pressure is

(A) [M1L0T-1]
(B) [ML1T-1]
(C) [M0L1T-2]
(D) [M1L-1T-2]
(v). Radius of one arm of hydraulic lift is four times of radius of another arm.
What force should be applied on narrow arm to lift 100 kg?

(A) 26.5 N

(B) 62.5 N

(C) 6.25 N

(D) 8.3 N
CASE 2. Ball moving with spin

A ball which is spinning drags air along with it. If the surface is rough more air
will be dragged. Fig shows the streamlines of air for a ball which is moving and
spinning at the same time. The ball is moving forward and relative to it the air is
moving backwards. Therefore, the velocity of air above the ball relative to it is
larger and below it is smaller. The stream lines thus get crowded above and
rarified below.

(i). Bernoulli equation is an example of conservation of


(A) Momentum

(B)Angular momentum

(C) Mass

(D) Energy

(ii). An aeroplane gets its upward lift due to a phenomenon described by the
(A) Archemedes’ principle

(B) Bernoulli’s principle

(C) Buoyancy principle

(D) Pascal law

(iii). Scent sprayer is based on


(A) Archemedes’ principle

(B) Bernoulli’s principle

(C) Buoyancy principle

(D) Pascal law


(iv). The reading of pressure meter attached with a closed pipe is 3.5 × 105 Nm-
2. On opening the value of the pipe, the reading of the pressure meter is
reduced to 3 × 105 Nm-2. Calculate the speed of the water flowing in the pipe.

(A) 10 m/s

(B) 5 m/s

(C) 20 m/s

(D) 15 m/s

(v). Which one is not correct for ideal fluid?

(A) Steady flow

(B) incompressible flow

(C) non-viscous flow

(D) rotational flow

CASE 3 Stokes’ law

When a body falls through a fluid it drags the layer of the fluid in contact with
it. A relative motion between the different layers of the fluid is set and as a
result the body experiences a retarding force. Falling of a raindrop and swinging
of a pendulum bob are some common examples of such motion. It is seen that
the viscous force is proportional to the velocity of the object and is opposite to
the direction of motion. The other quantities on which the force F depends are
viscosity η of the fluid and radius a of the sphere. Sir George G. Stokes (1819-
1903), an English scientist enunciated clearly the viscous drag force F as

F = π6ηav

This is known as Stokes’ law.

(i). For a ball falling in a liquid with constant velocity, ratio of resistance force
due to the liquid to that due to gravity is
(A) 1:2

2𝑎2 𝜌𝑔
(B) 9𝜂 2

2𝑎2 (𝜌−𝜎)𝑔
(C) 9𝜂

(D) none of these

(ii). A spherical of mass m and radius r is falling in the column of a viscous liquid.
Terminal velocity attained by falling object is proportional to
(A)r2

(B) r

(C) r-2

(D) r-1

(iii). Critical Velocity of the liquid


(A) decreases when radius decreases

(B) increases when radius increases

(C)decreases when density increases

(D)increases when density increases

(iv). A steel ball is dropped in oil, then


(A) The ball attains constant velocity after some time

(B) The ball stops

(C) The speed of ball will keep on increasing

(D) None of the above

(v). Motion of a liquid in a tube is best described by


(A) Bernoulli’s theorem

(B) Poiseuille’s equation

(C) Stokes’ Law

(D) Archemedes’ principle


CASE 4 Surface tension

Surface tension is a force per unit length (or surface energy per unit area) acting
in the plane of the interface between the plane of the liquid and any other
substance; it also is the extra energy that the molecules at the interface have as
compared to molecules in the interior. At any point on the interface besides the
boundary, we can draw a line and imagine equal and opposite surface tension
forces S per unit length of the line acting perpendicular to the line, in the plane
of the interface. The line is in equilibrium. To be more specific, imagine a line of
atoms or molecules at the surface. The atoms to the left pull the line towards
them; those to the right pull it towards them! This line of atoms is in
equilibrium under tension. If the line really marks the end of the interface, as in
fig there is only the force S per unit length acting inwards.

(i). What is the work done in blowing a soap bubble of radius r and surface
tension S?

(A) 8 π S r2

(B) 4 π S r2

(C)8 π S r

(D) 4 π S r

(ii). If a mercury drop is divided into 8 equal parts, its total energy

(A) remain same

(B) become twice

(C) become half

(D) become 4 times


(iii). The SI unit of surface tension is

(A)dyne/cm

(B) N/m2

(C) N/m

(D) Nm

(iv). The water droplets in free fall are spherical due to

(A) Gravity

(B) Viscosity

(C) Intermolecular attraction

(D) Surface tension

(v). The surface tension of liquid decreases with a rise in

(A) temperature of the liquid

(B) viscosity of the liquid

(C) diameter of container

(D) thickness of container

CASE 5 Viscosity

Viscosity is, essentially, fluid friction. Like friction between moving solids,
viscosity transforms kinetic energy of (macroscopic) motion into heat energy.
Heat is energy of random motion at the molecular level, so to have any
understanding of how this energy transfer takes place, it is essential to have
some picture, however crude, of solids and/or liquids sliding past each other as
seen on the molecular scale.

(i).The tangential force or viscous force on any layer of the liquid is directly
proportional to the velocity gradient. Then the direction of velocity gradient is

(A) parallel to the direction of the flow of the liquid

(B) opposite to the direction of the flow of the liquid


(C) independent to the direction of the flow of the liquid

(D) perpendicular to the direction of flow of the liquid

(ii). The SI unit of coefficient of viscosity is

(A) Nm/s

(B) Nm2/s

(C) N/m2s-1

(D) Nms2

(iii). Viscous force exerted by the liquid flowing between two plates in a
streamline flow depends upon

(A) Velocity gradient in the direction perpendicular in the plates

(B) Area of plates

(C) Coefficient viscosity of the liquid

(D) All of these

(iv). The coefficient of viscosity for hot air is

(A) greater than the coefficient of viscosity for cold air

(B) smaller than the coefficient of viscosity for cold air

(C) same as the coefficient of viscosity for cold air

(D) increase or decrease depending on the external pressure

(v). While studying about fluid mechanics, the equations and postulates stand
for the fluid which are

(A) Non-viscous only

(B) Incompressible only

(C)Non-viscous and Incompressible

(D) Liquid only


ANSWERS(CASE STUDY BASED QUESTIONS):

ANS (i) ANS (ii) ANS (iii) ANS (iv) ANS (v)

CASE 1 B A A D B

CASE 2 D B B A D

CASE 3 D A C A A

CASE 4 A B C D A

CASE 5 D C D A C
TWO MARK QUESTIONS:
1. QUESTION: State the angle of contact and on what values do the angle of contact
depends?
ANSWER: Angle of contact between a liquid and a solid is defined as the angle
enclosed between the tangents to the liquid surface and the solid surface inside the
liquid, both the tangents being drawn at the point of contact of liquid with the
solid. It depends upon:-
1) Upon nature of liquid and solid in contact
2) The Medium which exists above the free surface of liquid.

2. QUESTION: Find out the dimensions of co-efficient of viscosity?


ANSWER: Coefficient of viscosity (η)= Fdx/Adv where F = tangential force, dx = distance
between the layers , dv = velocity. By putting the dimensional formula of each we get :-
[η]= [M1L1T-2][M0L1T0] / [M0L2T0] [M0L1T-1] = [M1L-1T-1]
So Dimensional Formula of Coefficient of viscosity (η)=[M1L-1T-1]

3. QUESTION: What is stoke’s law and what are the factors on which viscous drag
depends?
ANSWER: The force acting between the liquid and falling body interface is
proportional to velocity and radius of the spherical object and viscosity of fluid.
Factors on which viscous drag depends are :-
F = 6πηrv
Where, F is the drag force or frictional force at the interface.
η is viscosity of a liquid.
r is radius of the spherical body.
V is velocity of object.

4. QUESTION: Two soap bubbles of different diameter are in contact with a certain
portion common to both the bubbles. What will be the shape of the common
boundary as seen from inside the smaller bubble? Support your answer with a neat
diagram and justify your answer.
ANSWER:

When seen from inside the smaller bubble the common surface will appear
concave as :- the pressure (excess) = 2T/R , Will be greater for concave surface & as
R is small for the smaller bubble, the pressure will be greater.

5. QUESTION: Write any two differences between streamline and turbulent flow.
ANSWER: (Instruction :- in exam you can write any two of the following)
Streamline flow :-
(i) The path of every particle is same
(ii) The velocity of particle is constant in magnitude and direction at a point.
(iii) The flow remains streamline below critical velocity.
(iv) No circular currents or eddies are developed
Turbulent flow :-
(i)The path of every particle is different
(ii) The velocity of particle at each point is not constant
(iii) The flow becomes turbulent after critical velocity.
(iv) Random circular currents called vertices are developed

6. QUESTION: Why are the droplets of mercury when brought in contact pulled
together to form a bigger drop? Also, a state with reasons whether the
temperature of this bigger drop will be the same or more or less than the
temperature of the smaller drop.
ANSWER: It is due to large cohesive force acting between the molecule as of
mercury that the droplets of mercury when brought in contact pulled together to
form a bigger drop in order to make potential energy minimum. The temperature
of this bigger drop increases since the total surface area decreases.

7. QUESTION: A drop of oil placed on the surface of water spreads out. But a drop of
water placed on oil contracts to a spherical shape. Explain both the phenomenon.
ANSWER: A drop of oil placed on the surface of water spreads because the force of
adhesion between water molecules and oil molecules dominates the cohesive
force-of oil molecules. So oil drop on water spreads.
On the other hand, the cohesive force of water molecules dominates the adhesive
force between water and oil molecules. So drop of water on oil contracts to a
spherical shape.

8. QUESTION: Show that liquid at rest exerts force perpendicular to the surface of the
container at every point.
ANSWER: Let a given quality of liquid exert force F in a direction θ with the
horizontal. Resolving, we get Fcos θ along horizontal surface of liquid and Fsin θ
along the vertical.

As there is no flow of liquid so Fcos θ should be zero. Since, F ≠ 0, cos θ = 0 Or


θ = 90° i.e., a liquid at rest exerts normal force to the walls of the container.

9. QUESTION: Explain the working of hydraulic lift.


ANSWER: Hydraulic lift works on Pascal’s law , if the pressure is applied to uniform
fluids that are confined, the fluids will then transmit the same pressure in all
directions at the same rate.

Consider the cross sectional area of the first piston is A1 and the cross sectional
area of the second piston is A2. The piston directly exerts a force F1 to the fluid. The
pressure P = F1/A1
Now P = F1/A1 is transferred throughout the fluid to the second piston of area A 2
and creates a upward force F2 ,
So, F2 = P×A2
F2 = F1(A2/A1) , by putting the value of P
Due to this F2 (large generated force by factor of A2/A1) , the piston is capable of
supporting a large weighted object.

10. QUESTION: Mention two similarities and two dissimilarities between frictional force
and viscous force.
ANSWER: Similarity:-
(i)Both came in to play whenever there is relative motion.
(ii)Both opposes relative motion.
Dissimilarity:-
(i)Viscous force depends on the velocity gradient and area of contact and frictional
force independent of area of contact and relative velocity.
(ii)Viscosity of liquid decrease with increase in temperature, Where as friction
independent of temperature.

THREE MARK QUESTIONS:


1. QUESTION: What is the effect of gravity on fluid pressure.
ANSWER:

Let P1 and P2 be liquid pressure at top and bottom of container respectively and
pressure is same at all points inside container else liquid will not be in equilibrium.

Force due to liquid pressure at top F1= P1A acting downwards

Force due to liquid pressure at the bottom F2=P2A acting upwards

Weight of liquid cylinder acting downwards W=Ahg

Where  is the density of liquid g is gravity


As liquid is in equilibrium so net downward force = net upward force

F1+W=F2 so F2-F1= W

P2A-P1A= Ahg

P2-P1=hg

P1 is atmospheric pressure at top of container is exposed to air so the pressure will


be exerted by the atmosphere on top surface of liquid and P 2 is pressure by liquid
above the bottom of container so P2= P and P1= Pa

So P-Pa = hg

P= Pa + hg

So the pressure at any point inside the fluid depends on the depth h.

Effect of gravity on Pascal's law:--

If we neglect the effect of gravity then :-

P2-P1= hg =0

So P2 = P1

That is pressure at all points inside the liquid is same in the absence of gravity. This
is Pascal's law and in presence of gravity Pascal's law is modified as P2-P1 = hdg

2. QUESTION: Show that the pressure exerted by a liquid column is proportional to its
height.
ANSWER:

Consider a vessel of height ‘h’ and cross-sectional area ‘A’ filled with a liquid of
density ‘’.

The weight of the liquid column exerts a downward thrust on the bottom of the
vessel and the liquid exerts pressure.

Weight of the liquid column,

W= mass of liquid x g

= volume x density x g

= Ah x  x g = Ahg

Pressure exerted by the liquid column on the bottom of the vessel is

P = Thrust/ area

P =W/A

P = Ahg / A

P = hg

P∝h where  and g are constant.

Hence proved.

3. QUESTION: Explain how Pascal demonstrated experimentally hydrostatic paradox.


ANSWER: In fluid dynamics, Hydrostatic Paradox speaks about the liquid pressure at
all the points at the same depth (horizontal level). It is defined as: “The pressure at a
certain horizontal level in the fluid is proportional to the vertical distance to the
surface of the fluid.
Hydrostatic paradox. Pascal demonstrated experimentally that the pressure exerted
by a liquid column depends only on the height of the liquid column and not on the
shape of the containing vessel. As shown in the following Fig. the experimental
arrangement consists of three glass vessels A, B and C of different shapes. The area
of the lower open end of all the vessels is same. The lower end of each vessel is
closed by supporting a disc against it. Each disc is connected to a pressure-meter.
When the three vessels are filled with the same liquid upto the same height, all the
three meters record the same pressure. This appears anomalous because the three
vessels have different shapes and contain different amounts of liquid. This
apparently unexpected result is known as hydrostatic paradox.

Explanation. The liquid presses normally on the surface of the container. In the case
of vessel (a), the reaction R is inclined in the downward direction as shown in Fig.
The vertical component V increases the downward thrust of the liquid. In the case
of vessel (b) the pressure is normal to the walls. The pressure acts horizontally on
the walls. The reaction R of the walls is also horizontal. In case of vessel (c), the
reaction R of the wall is inclined upwards. The vertical component V reduces the
downward thrust of liquid.

4. QUESTION: If a big drop of radius R is formed by 1000 small droplets of water, then
find the radius of small drop?
ANSWER: Let r = Radius of small drop ,R = Radius of Big drop
Now, Let P = Density of water
Mass of 1000 small droplets = 1000 × volume × Density
= 1000 x 4/3πr3 x P
Mass of big drop = 4/3πR3xP
Now Mass of 1000small drops = Mass of big drop
1000x4/3πr3xP = 4/3πR3xP
1000 r3 = R3
r = R/10
Hence the radius of small drop is (1/10) times the radius of big drop.
5. QUESTION: A U-tube contains water and methylated spirit separated by mercury.
The mercury columns in the two arms are in level with 10.0 cm of water in one arm
and 12.5 cm of spirit in the other. What is the specific gravity of spirit?
ANSWER: The given system of water , mercury and methylated spirit is shown as
follows :-

Height of the spirit column, h1 = 12.5 cm = 0.125 m


Height of the water column, h2 = 10 cm = 0.1 m
P0= Atmospheric pressure ; 1= Density of spirit ; 2= Density of water
Pressure at point B = P0 + h1p1g ; Pressure at point D = P0 + h2p2g
Pressure at points B and D is the same.
P0 + h1p1g = P0 + h2p2g
h1p1g = h2p2g
h1p1 = h2p2
p1/ p2 = h2 / h1
=10 /12.5 =0.8
Therefore, the specific gravity of spirit is 0.8.
6. QUESTION: In the following Figures (a) and (b) refer to the steady flow of a (non-
viscous) liquid. Which of the two figures is incorrect? Why?

ANSWER: Take the case given in figure (b).

Where,

= Area of pipe1; = Area of pipe 2;

= Speed of the fluid in pipe1; = Speed of the fluid in pipe 2

Now From the law of continuity, we have:

When the area of cross-section in the middle of the venturimeter is small, the speed
of the flow of liquid through this part is more. According to Bernoulli's principle, if
speed is more, then pressure is less.

Pressure is directly proportional to height. Hence, the level of water in pipe 2 is less.

Therefore, figure (a) is not possible.

7. QUESTION: (a) What is the largest average velocity of blood flow in an artery of
radius 2x10-3 m if the flow must remain laminar? (b) What is the corresponding flow
rate? (Take viscosity of blood to be 2.084x10-3Pa s). Reynolds' number for laminar
flow, NR = 2000
ANSWER: (a) Radius of the artery, r = 2x10-3 m

Diameter of the artery, d = 2x2x10-3 m

Viscosity of blood,

Density of blood,

Reynolds' number for laminar flow, NR = 2000

The largest average velocity of blood is given by the relation:

= 0.983 m/s

Therefore, the largest average velocity of blood is 0.983 m/s.

(b) Flow rate is given by the relation:

R = π r2 Vavg

Therefore, the corresponding flow rate is .


8. QUESTION: A liquid drop of diameter D breaks up into 27 tiny drops. Find the
resulting change in energy. Take surface tension of liquid as .
ANSWER: Radius of larger drop = D/2
Radius of each small drop = r

Initial surface area of large drop :-

Final surface area of 27 small drops:-

∴ Change in energy = increase in area x 

= 3πD2 - πD2

9. QUESTION: What is the energy stored in a soap babble of diameter 4 cm, given the
surface tension = 0.07 N/m?
ANSWER: Diameter of soap bubble = 4 cm = 4 × 10-2m

Radius of soap bubble = 2 × 10-2m

Increase in surface Area = 2 × 4 π R2

( 2, a bubble has 2 surfaces)

Increase in Surface Area = 2 × 4 π × (2 X 10-2)2

Now, energy stored = Surface Tension × Increase in Surface Area

Energy Stored :-

10. QUESTION: A liquid drop of diameter 4 mm breaks into 1000 droplets of equal size.
Calculate the resultant change in the surface energy. Surface tension of the liquid is
0.07 N/m?
ANSWER: Since the diameter of drop = 4mm

So Radius of drop R= 2mm = 2x10-3m

S = Surface tension = 0.07N/m

Let r be the radius of each of the small droplets volume of big drop = 1000 x volume
of the small droplets

4/3πR3 =1000 x 4/3πr3

or R = 10r

Original surface area of the big drop = 4πR2 ; surface area of 1000 small drops
=1000x4πr2

So increase in surface area = 4π [1000r2-R2]

Increase in surface = 4x (22/7) [ 1000(2x10-4)2 - (2x10-3)2 ]

= 4x (22/7) [ 4x10-5 – 0.4x10-5 ]

= 4x (22/7) [ 4 - 0.4]10-5

= 4x (22/7) x 3.6 x 10-5

Increase in surface energy = Surface tension x Increase in surface area

= 0.07x 4x (22/7) x 3.6 x 10-5

= 3168x10-8 J

= 3.168x10-5 J
11. Show that there is an excess of pressure on curved surface of liquid, which is more
on concave face that on convex face of curved surface of liquid.
ANSWER: If the liquid surface is curved, the molecule O on the liquid
surface is acted upon, bythe forces (S, S) due to surface tension along
the tangents to the surface, (fig) Resolving these forces into horizontal
and vertical components, we note that the horizontal components
cancel each other whereas vertical components add up.
Thus a resultant force would be acting on the curved surface of the
liquid which acts towards its centre of curvature. It means, the
resultant force is directed inwards incase of convex surface and is
directed outward in case of concave surface.
12. QUESTION: Explain why

(a) The angle of contact of mercury with glass is obtuse, while that of water with
glass is acute.

(b) Water on a clean glass surface tends to spread out while mercury on the same
surface tends to form drops. (Put differently, water wets glass while mercury does
not.)

(c) Surface tension of a liquid is independent of the area of the surface.


ANSWER:

(a) The angle between the tangent to the liquid surface at the point of contact and
the surface inside the liquid is called the angle of contact (Ɵ), as shown in the given
figure.
= (2s cos Ɵ / r1pg ) - (2s cos Ɵ / r2 pg )

Sla ,Ssa and Ssl, are the respective interfacial tensions between the liquid-air, solid air,
and solid-liquid interfaces. At the line of contact, the surface forces between the
three media must be in equilibrium, i.e.,

Cos Ɵ = (Ssa - Ssl ) / Sla

The angle of contact θ , is obtuse if Sla , Ssa<Sla (as in the case of mercury on glass).
This angle is acute if Ssl<Sla (as in the case of water on glass) .

(b)Mercury molecules (which make an obtuse angle with glass) have a strong force
of attraction between themselves and a weak force of attraction toward solids.
Hence, they tend to form drops.

On the other hand, water molecules make acute angles with glass. They have a
weak force of attraction between themselves and a strong force of attraction
toward solids. Hence, they tend to spread out.

(c) Surface tension is the force acting per unit length at the interface between the
plane of a liquid and any other surface. This force is independent of the area of the
liquid surface. Hence, surface tension is also independent of the area of the liquid
surface.
14. QUESTION: Explain why

(a) To keep a piece of paper horizontal, you should blow over, not under, it.

(b) When we try to close a water tap with our fingers, fast jets of water gush
through the openings between our fingers.

(c) The size of the needle of a syringe controls flow rate better than the thumb
pressure exerted by a doctor while administering an injection.

ANSWER: (a) When air is blown under a paper, the velocity of air is greater under
the paper than it is above it. As per Bernoulli's principle, atmospheric pressure
reduces under the paper. This makes the paper fall. To keep a piece of paper
horizontal, one should blow over it. This increases the velocity of air above the
paper. As per Bernoulli's principle, atmospheric pressure reduces above the paper
and the paper remains horizontal.

(b) According to the equation of continuity:

= Constant

For a smaller opening, the velocity of flow of a fluid is greater than it is when the
opening is bigger. When we try to close a tap of water with our fingers, fast jets of
water gush through the openings between our fingers. This is because very small
openings are left for the water to flow out of the pipe. Hence, area and velocity are
inversely proportional to each other.

(c) The small opening of a syringe needle controls the velocity of the blood flowing
out. This is because of the equation of continuity. At the constriction point of the
syringe system, the flow rate suddenly increases to a high value for a constant
thumb pressure applied.

15. QUESTION: State the principle on which Hydraulic brakes work and explain its
working with suitable diagram?
ANSWER: Hydraulic Brakes: - Hydraulic brakes work on the principle of Pascal’s
law.

According to this law whenever pressure is applied on a fluid it travels uniformly in

all the directions.

Therefore when we apply force on a small piston, pressure gets created which is

transmitted through the fluid to a larger piston. As a result of this larger force,

uniform braking is applied on all four wheels.

As braking force is generated due to hydraulic pressure, they are known as


hydraulic brakes.

Liquids are used instead of gas as liquids are incompressible.

Construction:-

The figure shows the schematic diagram of a hydraulic brake system. The brake
system has a main cylinder filled with brake oil. The main cylinder is provided with a
piston P which is connected to the brake pedal through a lever assembly. The fluid
in the hydraulic brake is known as brake fluid.

The wheel cylinders consist of two pistons P1 and P2 between which fluid is filled.

wheel brake consists of a cylinder brake drum. This drum is mounted on the inner

side of wheel. The drum revolves with the wheel.Two brake shoes S1 and S2 which

are mounted inside the drum remain stationary.


Working of Hydraulic brake:-

When the brake pedal is pressed, piston P is pushed due to the lever assembly
operation. The pressure in the main cylinder is transmitted to P1 and P2. The
pistons P1 and P2 push the brake shoes away, which in turn press against the inner
rim of the wheel. Thus the motion of the wheel is arrested. The area of the pistons
P1 and P2 are greater than that of P. Therefore according to Pascal’s law a small
force applied to the brake pedal produces a large thrust on the wheel rim. The main
cylinder is connected to all the wheels of the automobile through pipe line for
applying equal pressure to all the wheels. The frictional force between these force
components causes the vehicle to stop.

Mathematical explanation of working of hydraulic brakes:-

Let A1 and A2 are the areas of the pistons P and (P1 or P2) respectively, F is the
force applied on brake lever on piston P and W is the work done (braking force) by
pistons P1 and P2 on brake shoes S1 and S2, then

F/A1 = W/A2 or ; W = F (A2/A1) {by Pascal’s Law}

So Braking Force (by each piston P1 & P2) = F (A2/A1)

In the above equation (A2/A1) is called the mechanical advantage of the Hydraulic
Brakes.

So by applying small force F on brake lever we can stop the moving vehicle.
16. QUESTION: (a) State & prove Bernoulli’s theorem.

(b) A plane is in level flight at constant speed and each of its two wings has an area
of 25m2. If the speed of the air is 180 km/h over the lower wing and 234 km/h over
the upper wing surface, determine the plane's mass. (Take air density to be 1
kg /m2).
ANSWER: (a) According to Bernoulli’s theorem, the sum of the energies possessed
by a flowing ideal liquid at a point per unit volume is constant provided that the
liquid is incompressible and non-viscous and flows in streamline.

Pressure energy + Kinetic energy + Potential energy = Constant

P + ½ v2 + gh =constant

Proof:

p1 = Pressure applied on liquid at A

p2 = Pressure applied on liquid at B

a1, a2 = Area of cross – section at A & B

h1, h2 = height of section A and B from the ground.

v1, v2 = Normal velocity of liquid at A and B

s= Density of ideal liquid

Let P1 > P2

m = Mass of liquid crossing per second through any section of tube.

a1 v1 s = a2 v2 s = m

or a1 v1 = a2 v2 = m/s = v

As a1 > a2 ∴ v2 >v1

Force of on liquid at A = p1 a1

Force on liquid at B = p2 a2

Work done/second on liquid at A = p1 a1 x v1 = p1V

Work done/second on liquid at B = p2 V


Net work done | second by pressure energy in moving the liquid from A to B

= p1 v – p2 v →(1)

If ‘m’ mass of liquid flows in one second from A to B then Increases in potential
energy per second from A to B = mgh2 – mgh1 →(2)
1 1
Increase in kinetic energy/second of liquid from A to B = 2 m(v2)2 - 2 m(v1)2 → (3)

From, work energy principle:-

Work done by pressure energy = Increase in P. E. /sec + Increase in K. E/sec

From equation 1, 2, & 3


1 1
P1 v – p2 v = (mgh2 – mgh1) + m(v2)2 - 2 m(v1)2
2

1 1
P1 v + mgh1 + m(v1)2 = p2 v + m(v2)2 + mgh2
2 2

Dividing throughout by m →

……..→ (4)
𝑚
Now Density s= 𝑣

Hence, →4)
𝑝 1
+ gh + 2 v2 = Constant
𝑠

𝑝
= Pressure energy per unit mass
𝑠

gh = potential energy per unit mass


1 2
v= kinetic energy per unit mass
2

Hence from equation), Bernoulli’s theorem is proved.


(b) The area of the wings of the plane, A = 2 25 = 50

Speed of air over the lower wing, = 180 km/h = 50 m/s

Speed of air over the upper wing, = 234 km/h = 65 m/s

Density of air,

Pressure of air over the lower wing = ; Pressure of air over the upper wing=

The upward force on the plane can be obtained using Bernoulli's equation as:

………(i)

The upward force (F) on the plane can be calculated as:

Using equation (i)

= 43125N

Using Newton's force equation, we can obtain the mass (m) of the plane as:

F = mg

= 4400.51 kg 4400 kg ; Hence, the mass of the plane is about 4400 kg.
17. QUESTION: (a)what is the phenomenon of capillarity ? Derive an expression for the
rise of liquid in a capillary tube and show that the height of the liquid column
supported is inversely proportional to the radius the of the tube.

(b) What will happen if the length of a capillary tube is less than the height upto
which the liquid may rises in it? Explain briefly.
ANSWER: (a) A capillary tube is a tube with a very fine bore. The phenomenon of a
rise and fall of a liquid inside a capillary tube with respect to its surrounding (in
which liquid it is dipped) is called capillary action or capillarity.

Ascent formula. Consider a capillary tube of radius r dipped in a liquid of surface


tension and density p. Suppose the liquid wets the sides of the tube. Then its
meniscus will be concave. The shape of the meniscus of water will be nearly
spherical if the capillary tube is of sufficiently narrow bore.

As the pressure is greater on the concave side of a liquid surface , so excess of


pressure at a point A just is above the meniscus compared to point B just below the
meniscus is :

P = 2/R

where R= radius of curvature of the concave meniscus. If Ɵ is the angle of contact,


then from the right angled triangle shown in above Fig. We have:

r/R = cosƟ

or R = r/ cosƟ

∴ p = 2 cosƟ/r
Due to this excess pressure p, the liquid rises in the capillary tube to height h when
the hydrostatic pressure exerted by the liquid column becomes equal the excess
pressure p. Therefore, at equilibrium we have :

p=hpg

or 2 cosƟ/r =hpg

or h = 2 cosƟ/rpg

This is the ascent formula for the rise of liquid in capillary tube. If we take into
account the volume of quid contained in the meniscus, then the above formula gets
modified as :

h = (2 cosƟ/rpg ) – r/3

However, the factor r/3 can be neglected for a narrow tube.

The ascent formula shows that the height h to which a liquid rises in the capillary
tube is :

(i) Inversely proportional to the radius of the tube.

(ii) Inversely proportional to the density of the liquid.

(iii) directly proportional to the surface tension of the liquid.

Hence a liquid rises more in a narrower tube than in wider tube.


ANSWER: (b) Rise of liquid in a tube of insufficient height. The height to which a
liquid rises in a capillary tube is given by:

h = 2 cosƟ/rpg

The radius r of the capillary tube and radius of curvature R of the liquid meniscus
are related by

r = RcosƟ ……………..(1)

and we know, h = 2 cosƟ/rpg …………….(2)

from equation (1) & (2)

h = 2 cosƟ/ RcosƟ pg or h = 2 /Rpg

As , p and g are constants , so

hR = 2 /pg

or hR = a constant

so it implies hR =h’R’

where R' is the radius of curvature of the new meniscus at a height h'.

As h’< h ; so R’> R

Hence in a capillary tube of insufficient height, the liquid rises to the top and
spreads out to a new radius of curvature R' given by :

R' = hR/ h'

Hence the liquid will not overflow.


ADDITIONAL QUESTIONS (ONE MARK QUESTIONS):
1. QUESTION: The blood pressure of humans is greater at the feet than at the brain?
ANSWER: The height of the blood column in the human body is more at the feet
than at the brain as since pressure is directly dependent on height of the column, so
pressure is more at feet than at the brain.
2. QUESTION: Define surface tension?
ANSWER: It is measured as the force acting on a unit length of a line imagined to be
drawn tangentially anywhere on the free surface of the liquid at rest.
3. QUESTION: Oil is sprinkled on sea waves to calm them. Why?
ANSWER: When oil is sprinkled, the breeze spreads the oil on the sea-water in its
own direction. The surface tension of sea-water (without oil) is greater than oily
water. Hence the water without oil pulls the oily water against the direction of
breeze, and the sea waves become calm.
4. QUESTION: Water rises in a capillary tube but mercury falls in the same tube. Why?
ANSWER: This is due to the property of surface tension. Water makes an acute
angle of contact with glass, hence Cos θ is positive, so it rises while mercury makes
an obtuse angle of contact with glass, hence Cos θ is negative so it falls in a capillary
tube.
5. QUESTION: Two balls of same material are given. The diameter of ball A is half that
of ball B. What will be their ratio of their terminal velocities in water?
ANSWER: The terminal velocity is directly proportional to the square of radius of the
ball, therefore the ratio of terminal velocities will be 1:4.
6. QUESTION: Hydrostatic pressure is a scalar quantity even though pressure is force
divided by area, and force is a vector. Explain?
ANSWER: Since due to applied force on liquid, the pressure is transmitted equally in
all directions, inside the liquid. Since there is no fixed direction for the pressure due
to liquid. Hence it is a scalar quantity.
7. QUESTION: Define viscosity?
ANSWER: Viscosity is the property of a fluid by virtue of which an internal frictional
force comes into play when the fluid is in motion and opposes the relative motion of
its different layers.
8. QUESTION: Give two areas where Bernoulli’s theorem is applied?
ANSWER:Bernoulli’s theorem is applied in atomizer and in lift of an aero plane wing.
9. QUESTION: What is conserved in Bernoulli’s theorem?
ANSWER: According to Bernoulli’s theorem, for an incompressible non – Viscous
liquid (fluid) undergoing steady flow the total energy of liquid at all points is
constant.
10. QUESTION: Why are rain drops spherical?
ANSWER: Due to surface tension the drops try to occupy minimum surface area, and
for a given volume sphere has minimum surface area.
11. QUESTION: Define terminal velocity.
ANSWER: Terminal velocity is defined as the maximum velocity attained by a body
as it falls through a fluid. It occurs when the sum of the drag force and the
buoyancy, or upthrust, is equal to the downward gravitational force acting on the
body.
12. QUESTION: Why is it easier to skate on ice than on a smooth aluminum sheet?
ANSWER: The ice below the feet of the person smelts on account of increased
pressure. Tiny drops of water are formed. These behave like ‘rollers’ and thus it is
easier to skate on ice. No such thing is possible in the case of aluminum sheets.
13. QUESTION: What happens when a capillary tube of insufficient length is dipped in a
liquid?
ANSWER: When a capillary tube of insufficient length is dipped in a liquid, the radius
of curvature of the meniscus increase so that hr = constant. That is pressure on
Concave side becomes equal to pressure exerted by liquid Column so liquid does
not overflow.
14. QUESTION: By which phenomenon the water rises from roots to leaves of plants?
ANSWER: Plants use capillary action to bring water from the soil up through
capillaries (roots), small tubes in the plants, to the leaves of the plant.
15. QUESTION: Define force of cohesion.
ANSWER: The force of cohesion is defined as the force of attraction between
molecules of the same substance.
16. The accumulation of snow on an aero plane wing may reduce the lift. Explain?
ANSWER: Due to the accumulation of snow on the wings of the aero plane, the
structure of wings no larger remains as that of aerofoil. As a result, the net upward
force (i.e. lift) is decreased.
CHAPTER 11

THERMAL PROPERTIES OF MATTER

GIST OF LESSON:

1. Heat is the form of energy transferred between two (or more) systems or a system and its
surroundings by virtue of temperature difference. The SI unit of heat energy transferred is
expressed in joule (J).
In CGS system, unit of heat is calorie and kilocalorie (kcal).
1 cal = 4.186 J and 1 kcal = 1000 cal = 4186 J.

2. Temperature of a substance is a physical quantity which measures the degree of hotness or


coldness of the substance. The SI unit of temperature is kelvin (K) and °C is a commonly used unit
of temperature.

3. Thermal Expansion
The increase of size of a body due to the increase in the temperature is called thermal expansion.
Three types of expansions can take place in solids viz. linear, superficial and volume expansion.
(i) Linear Expansion: The increase in the length of a solid on heating is called linear expansion.
If the temperature of a rod of original length l is raised by a small amount Δt, its length increases
by Δl. Then the linear expansion is given by
Δl = l α Δt
where a is the coefficient of linear expansion of the given solid. The unit of α is per degree Celsius
(°C-1) in the CGS and per kelvin (K-1) in the SI system.
(ii) Superficial or Area Expansion: The increase in surface area of the solid on heating is called
superficial expansion.
If A0 is the area of a solid at 0°C and A( its area at t°C then

At = A0(l + βt)
where β is known as the coefficient of superficial expansion. Unit of β is °C-1 or K-1.
(iii) Volume Expansion: The increase in volume of the solid on heating is called volume expansion.
The relation among coefficients of linear expansion (α), superficial expansion (β) and volume
expansion (γ) is given as

• For a given solid, the three coefficients of expansion α , β, γ are not constant. Their values
depend on the temperature range.

Thermal Capacity
The thermal capacity of a body is the quantity of heat required to raise the temperature of the
whole of the body through a unit degree. It is measured in calorie per °C or joule per K.
If Q be the amount of heat needed to produce a change in temperature (Δt) of the substance,
then thermal capacity of the substance is given by
Dimensional formula of heat capacity is [ML2T -2K-1 ],

• Specific Heat Capacity


The specific heat capacity (also referred to as specific heat) of a substance is the amount of heat
required to raise the temperature of a unit mass of substance through 1 °C. It is measured in cal
g-1(°C)-1 or J kg-1 K-1.
The specific heat capacity of a substance is given by

where m is mass of substance and Q is the heat required to change its temperature Δt.
• Molar specific heat capacity of a substance is defined as the amount of heat required to raise
the temperature of 1 mole of the substance by 1°C.

• Latent Heat
Latent heat of a substance is the amount of heat energy required to change the state of unit mass
of the substance from solid to liquid or from liquid to gas/vapour without any change in
temperature.
• The latent heat of fusion (Lf) is the heat per unit mass required to change a substance from solid
into liquid at the same temperature and pressure. The latent heat of vaporisation (L v) is the heat
per unit mass required to change a substance from liquid to vapour state without change in
temperature and pressure.

• Heat Transfer
Heat can be transferred from one place to another by three different methods, namely,
conduction, convection and radiation. Conduction usually takes place in solids, convection in
liquids and gases, and no medium is required for radiation.
(i) Conduction: According to Maxwell, conduction is the flow of heat through an unequally heated
body from places of higher temperature to those of lower temperature. Rate of heat transfer is
given by

where K is called Thermal Conductivity and A is area of cross-section.

• Black Body Radiation


(i) Emissive Power: The amount of heat energy rediated per unit area of the surface of a body,
per unit time and per unit wavelength range is constant which is called as the ’emissive power’
(eλ) of the given surface, given temperature and wavelength. Its S.I. unit is Js-1 m-2 .
(ii) Absorptive Power : When any radiation is incident over a surface of a body, a part of it gets
reflected, a part of it gets refracted and the rest of it is absorbed by that surface. Therefore, the
‘absorptive power’ of a surface at a given temperature and for a given wavelength is the ratio of
the heat energy absorbed by a surface to the total energy incident on it at a certain time. It is
represented by (aλ). It has no unit as it is a ratio.
(iii) Perfect Black Body: A body is said to be a perfect black body is its absorptivity is 1. It neither
reflects nor transmits but absorbs all the thermal radiations incident on it irrespetive of their
wavelengths.

(iv) Wein’s Displacement Law : This law states that as the temperature of black body increases,
the maximum value of the radiant energy emitted by the black body, move towards shorter
wavelengths. Wein found that “The product of the wavelength ( λm) corresponding to maximum
intensity of radiation and the Kelvin temperature (T) of the black body should remain constant.”
λm x T= b
Where b is constant known as Wein’s constant. Its value is 2.898 x 10-3 mk.

(v) Stefan’s Law : This law states that the total energy radiated per unit surface area of a black
body in unit time is directly proportional to the fourth power of its absolute temperature T.
Emission coefficient or degree of blackness of a body is represented by a dimensionless quantity
ε, 0 < ε < 1. If ε = 1 then the body is perfectly black body. Hence
CASE STUDY BASED QUESTIONS:
CASE 1. TOPIC: THERMAL EXPENSION

You can often loosen a tight metal jar lid by holding it under a stream of hot water.
Both the metal of the lid and the glass of the jar expand as the hot water adds
energy to their atoms. (With the added energy, the atoms can move a bit farther
from one another than usual, against the spring-like inter-atomic forces that hold
every solid together.) However, because the atoms in the metal move farther apart
than those in the glass, the lid expands more than the jar and thus is loosened. Such
thermal expansion of materials with an increase in temperature must be
anticipated in many common situations. When a bridge is subject to large seasonal
changes in temperature, for example, sections of the bridge are separated by
expansion slots so that the sections have room to expand on hot days without the
bridge buckling. When a dental cavity is filled, the filling material must have the
same thermal expansion
properties as the surrounding
tooth; otherwise, consuming
cold ice cream and then hot
coffee would be very painful.
When the Concorde aircraft
(Fig.) was built, the design had
to allow for the thermal
expansion of the fuselage
during supersonic flight because of frictional heating by the passing air. When a
Concorde flew faster than the speed of sound, thermal expansion due to the
rubbing by passing air increased the aircraft’s length by about 12.5 cm. (The
temperature increased to about 128 C at the aircraft nose and about 90 C at the tail,
and cabin windows were noticeably warm to the touch.)

(i). A liquid with coefficient of cubical expansion γ is contained in a vessel having


coefficient of linear expansion γ/7. When heated, what will happen to the level of
the liquid in the vessel?
(A) It falls
(B) It rises
(C) Remains unchanged
(D) It may rise or fall depending upon the nature of the container

(ii). The temperature determines the direction of net change of:


(A) gross potential energy
(B) intermolecular potential energy
(C) gross kinetic energy
(D) intermolecular kinetic energy
(iii). Select True/False statements :
(A) Coefficients of expansion differ when lengths are measured in C.G.S. and S.I.
system.
(B) A substance may contract on heating
(C) A glass stopper is taken out easily from the bottle neck on heating the neck.
(D) None of above
(iv). Two bars of copper having same length but an equal diameter are heated to the
same temperature. The change in length will be:
(A) more in thicker bar
(B) more in thinner bar
(C) same for both
(D) determined by the ratio of length and diameter of the bars
(v). On which of the following property of the body, the coefficient of thermal
expansion depends?
(A) Shape
(B) Size
(C) Temperature
(D) P.E. curve
CASE 2. TOPIC: ABSORPTION OF HEAT BY SOLIDS AND LIQUIDS

In determining and then using the specific heat of any substance, we need to know
the conditions under which energy is transferred as heat. For solids and liquids, we
usually assume that the sample is under constant pressure (usually atmospheric)
during the transfer. It is also conceivable that the sample is held at constant volume
while the heat is absorbed. This means that thermal expansion of the sample is
prevented by applying external pressure. When energy is absorbed as heat by a
solid or liquid, the temperature of the sample does not necessarily rise. Instead, the
sample may change from one phase, or state, to another. Matter can exist in three
common states: In the solid state, the molecules of a sample are locked into a fairly
rigid structure by their mutual attraction. In the liquid state, the molecules have
more energy and move about more. They may form brief clusters, but the sample
does not have a rigid structure and can flow or settle into a container. In the gas, or
vapor, state, the molecules have even more energy, are free of one another, and
can fill up the full volume of a container.

(i). Thermal equilibrium implies equality of:


(A) energy
(B) internal energy
(C) K.E.
(D) temperature
(ii). Heat is
(A) energy in transit
(B) P.E. of molecules
(C) K.E. of molecules
(D) P.E. and K.E. of molecules
(iii). A beaker is completely filled with water at 4°C. It will overflow
(A) when heated but not when cooled
(B) when cooled but not when heated
(C) neither when heated nor when cooled
(D) both when cooled or heated
(iv). By increasing the temperature of a liquid, its
(A) volume and density increases
(B) volume and density decreases
(C) volume increases and density decreases
(D) volume decreases and density increases
(v). Heat is absorbed by a body but its temperature does not rise. Which of the
following statement is True/False about the phenomenon?
(A) Only P.E. of intermolecular force fields increases.
(B) Only K.E. of vibration increases.
(C) No increase in internal energy takes place.
(D) Increase in K.E. is balanced by decrease in P.E.
CASE 3. TOPIC: GREEN HOUSE EFFECT AND RADIATION

Earth's atmosphere is composed primarily of nitrogen and oxygen. These gases are
transparent to incoming solar radiation. They are also transparent to outgoing
infrared radiation, which means that they do not absorb or emit solar or infrared
radiation. However, there are other gases in Earth's atmosphere that do absorb
infrared radiation. These
gases are known as
greenhouse gases. Below
are the most important
greenhouse gases that
influence Earth's climate
system. When an object is
hot enough, you can see the
radiation it emits as visible
light. For example, when a
stovetop burner reaches
1,000 Kelvin (K) — 726°
Celsius (C) or 1,340° Fahrenheit (F) — it will glow red. All objects actually emit
radiation if their temperature is greater than absolute zero. Absolute zero is equal to
zero Kelvin, which is equal to -273°C or -460°F. Both the sun and Earth's surface
behave as blackbodies. An object that absorbs and emits all possible radiation at 100
percent efficiency is called a blackbody. For this reason, the following two laws
(Stefan-Boltzmann and Wien’s laws) can be used to explain the correlation between
temperature and radiation for the sun and Earth. The Stefan-Boltzmann law, a
fundamental law of physics, explains the relationship between an object's
temperature and the amount of radiation that it emits. This law (expressed
mathematically as E = σT4) states that all objects with temperatures above absolute
zero (0K or -273°C or -459°F) emit radiation at a rate proportional to the fourth
power of their absolute temperature.

i). The radiations emitted by hot bodies are called as


(A) X-rays
(B) Black-body radiation
(C) Gamma radiations
(D) Visible light
ii). Gas molecules absorbing thermal infrared radiation and present in large quantity
to change the climate system is known as
(A) ozone gases
(B) beta radiations
(C) alpha radiations
(D) greenhouse gases
iii). An iron rod is heated. The colours at different temperatures are noted. Which of
the following colours shows that the iron rod is at the lowest temperature?
(A) Red
(B) Orange
(C) White
(D) Blue
iv). Which statement is false about the greenhouse effect
(A) Life on the earth is possible due to the greenhouse effect.
(B) The greenhouse effect is a natural process that maintains the earth’s
temperature.
(C) Increased emission of greenhouse gases is a natural process.
(D) Increased emission of greenhouse gases in the atmosphere increases earth’s
temperature.
v). For an object other than a black body, it’s emissivity, e is
(A) 1
(B) 0 < e < 1
(C) e > 1
(D) e = 0

ANSWERS (CASE STUDY BASED QUESTIONS):


ANS (i) ANS (ii) ANS (iii) ANS (iv) ANS (v)
CASE 1 C D (a) False C D
(b) True
(c) True
CASE 2 D A D C (a) True
(b) False
(c) False
(d) False
CASE 3 B D A C B
TWO MARK QUESTIONS:
1. QUESTION: Why is mercury used in a thermometer?
ANSWER: The mercury is used in thermometer due to the following reasons-:
a) It has a uniform coefficient of expansion over a wide range of temperature
b) Mercury is opaque and bright so its level can be seen easily in a glass tube
c) It does not stick to the walls of the glass tube
d) It is nonvolatile.
2. QUESTION: What do you understand by anomalous expansion of water ?
ANSWER: Almost all liquids expand on being heated but water behaves in a peculiar
manner. When water at 00C is heated, its volume decreases and, therefore, its density
increases, until its temperature reaches 40C. Above 40C, the volume increases and
therefore the density decreases. Thus water at 40C has the maximum density.
3. QUESTION: Prove that the coefficient of cubical expansion of an ideal gas at constant
pressure is equal to the reciprocal of its absolute temperature.
ANSWER:

4. QUESTION: What do you mean by triple point of water? Why it is unique ?


ANSWER: It is the temperature at which the three phases of water: ice , liquid water &
water vapour are equally stable and exist simultaneously .It is unique because it occurs at
specific temperature ( 273.16K ) and a specific pressure of about 0.46cm of Hg column
5. QUESTION: The difference between lengths of a certain brass rod and that of a steel road is
claimed to be constant at all temperatures. Is this possible?
ANSWER: Yes. This is possible when the lengths of the rods are in inverse ratio of their
coefficient of linear expansion. For the difference in the lengths of the two rods to remain
same,
∆l1= ∆l2 or l1α1∆T= l2α2∆T or l1/ l2 = α2/ α1
6. QUESTION: Why is invar used in making a clock pendulum?
ANSWER: Invar, which is an alloy of nickel and steel, has extremely small temperature
coefficient of expansion and so the length of a pendulum made of invar does not change
appreciably during summer and winter seasons and hence clock gives almost correct time.
7. QUESTION: Why is water used as an effective coolant in radiator?
ANSWER: The specific heat of water is very high. When it runs over hot parts of an engine
or machinery, it absorbs a large amount of heat. This helps in maintaining the temperature
of the engine low.
8. QUESTION: Why do animals curl into a ball , when they feel very cold ?
ANSWER: The total energy radiated by a body depends on its surface area. Thus when the
animals feel very cold , they curl their bodies into a ball so as to decrease the surface area
of their bodies which in turn helps to reduce the amount of heat lost by them.
9. QUESTION: Why are clear nights colder than cloudy nights ?
ANSWER: Clouds are opaque to heat radiations. So on a cloudy night, radiations from the
earth‘s surface fail to escape. But on a clear night, the surface of the earth is cooled due to
excessive radiation. So a clear night is colder than a cloudy night.
10. QUESTION: Ice of 00C is converted into steam at 1000C. State the isothermal changes in the
process.
ANSWER: The isothermal changes are
a) Conversion of ice at 00C into water at 00C
b) Conversion of water at 1000C into steam at 1000C

THREE MARK QUESTIONS:


1. QUESTION: A steel tape 1m long is correctly calibrated for a temperature of 27.0 °C. The
length of a steel rod measured by this tape is found to be 63.0 cm on a hot day when the
temperature is 45.0 °C. What is the actual length of the steel rod on that day? What is the
length of the same steel rod on a day when the temperature is 27.0 °C? Coefficient
of linear expansion of steel = 1.20 × 10–5 K–1.
ANSWER: Length of the steel tape at temperature T = 27°C, l = 1 m = 100 cm.
At temperature T1 = 45°C, the length of the steel rod, l1 = 63 cm
Coefficient of linear expansion of steel, α = 1.20 × 10–5 K–1
Let l2 be the actual length of the steel rod and l' be the length of the steel tape at 45°C

Hence, the actual length of the steel rod measured by the steel tape at 45°C can be
calculated as:

Therefore, the actual length of the rod at 45.0°C is 63.0136 cm. Its length at 27.0°C is 63.0
cm.
2. QUESTION: A large steel wheel is to be fitted on to a shaft of the same material. At 27 °C,
the outer diameter of the shaft is 8.70 cm and the diameter of the central hole in the
wheel is 8.69 cm. The shaft is cooled using ‘dry ice’. At what temperature of the shaft does
the wheel slip on the shaft? Assume coefficient of linear expansion of the steel to be
constant over the required temperature range:
αsteel = 1.20 × 105 K–1
ANSWER: The given temperature, T = 27°C can be written in Kelvin as:27 +273 = 300 K
Outer diameter of the steel shaft at T, d1 = 8.70 cm
Diameter of the central hole in the wheel at T, d2 = 8.69 cm
Coefficient of linear expansion of steel, αsteel = 1.20 × 10–5 K–1
After the shaft is cooled using ‘dry ice’, its temperature becomes T1. The wheel will slip on
the shaft, if the change in diameter, ∆d = 8.69 –8.70 = –0.01 cm
Temperature T1, can be calculated from the relation:
∆d= d1asteel (T1 –T)
= 8.70 × 1.20 × 10–5 (T1 –300)
(T1 –300) = 95.78
T1= 204.21 K
= 204.21 –273.16= –68.95°C
Therefore, the wheel will slip on the shaft when the temperature of the shaft is –69°C.

3. QUESTION: A brass rod of length 50 cm and diameter 3.0 mm is joined to a steel rod of
the same length and diameter. What is the change in length of the combined rod at 250
°C, if the original lengths are at 40.0 °C? Is there a ‘thermal stress’ developed at the
junction? The ends of the rod are free to expand (Co-efficient of linear expansion of brass
= 2.0 × 10–5 K–1, steel = 1.2 × 10–5 K–1).
ANSWER: Initial temperature, T1 = 40°C ,Final temperature, T2 = 250°C
Change in temperature, ΔT = T2 – T1 = 210°C
Length of the brass rod at T1, l1 = 50 cm
Diameter of the brass rod at T1, d1 = 3.0 mm
Length of the steel rod at T1, l2 = 50 cm
Diameter of the steel rod at T2, d2 = 3.0 mm
Coefficient of linear expansion of brass, α1 = 2.0 × 10–5K–1
Coefficient of linear expansion of steel, α2 = 1.2 × 10–5K–1
For the expansion in the brass rod, we have:

For the expansion in the steel rod, we have:


Total change in the lengths of brass and steel,
∆l = ∆l1 + ∆l2
= 0.2205 + 0.126 = 0.346 cm
Total change in the length of the combined rod = 0.346 cm
Since the rod expands freely from both ends, no thermal stress is developed at the
junction

4. QUESTION: In an experiment on the specific heat of a metal, a 0.20 kg block of the metal at
150 °C is dropped in a copper calorimeter (of water equivalent 0.025 kg) containing 150
cm3 of water at 27 °C. The final temperature is 40 °C. Compute the specific heat of the
metal. If heat losses to the surroundings are not negligible, is your answer greater or
smaller than the actual value for specific heat of the metal
ANSWER: Mass of the metal, m = 0.20 kg = 200 g
Initial temperature of the metal, T1 = 150°C
Final temperature of the metal, T2 = 40°C
Calorimeter has water equivalent of mass, m’ = 0.025 kg = 25 g
Volume of water, IN = 150 cm3
Mass (M) of water at temperature T = 27°C:
150 × 1 = 150 g
Fall in the temperature of the metal:
∆T = T1 –T2 = 150 –40 = 110°C
Specific heat of water, Cw = 4.186 J/g/°K
Specific heat of the metal = C
Heat lost by the metal, ∆Q = mC∆T ... (i)
Rise in the temperature of the water and calorimeter system:
∆T′ = 40 –27 = 13°C
Heat gained by the water and calorimeter system:
∆Q′′ = m1 Cw∆T’
= (M + m′) Cw∆T’ ... (ii)
Heat lost by the metal = Heat gained by the water and colorimeter system
mC∆T = (M + m’) Cw∆T’
200 × C × 110 = (150 + 25) × 4.186 × 13
If some heat is lost to the surroundings, then the value of C will be smaller than the
actual value.
5. QUESTION: A thermacole icebox is a cheap and efficient method for storing small quantities
of cooked food in summer in particular. A cubical icebox of side 30 cm has a thickness of
5.0 cm. If 4.0 kg of ice is put in the box, estimate the amount of ice remaining after 6 h.
The outside temperature is 45 °C, and co-efficient of thermal conductivity of thermacole is
0.01 J s–1 m–1 K–1. [Heat of fusion of water =335 × 103 J kg–1]
ANSWER: Side of the given cubical ice box, s = 30 cm = 0.3 m
Thickness of the ice box, l = 5.0 cm = 0.05 m
Mass of ice kept in the ice box, m = 4 kg
Time gap, t = 6 h = 6 × 60 × 60 s
Outside temperature, T = 45°C
Coefficient of thermal conductivity of thermacole, K = 0.01 J s–1 m–1 K–1
Heat of fusion of water, L = 335 × 103 J kg–1
Let m’ be the total amount of ice that melts in 6 h.
The amount of heat lost by the food

Where, A = Surface area of the box = 6s2 = 6 × (0.3)2 = 0.54 m3

Mass of ice left = 4 –0.313 = 3.687 kg


Hence, the amount of ice remaining after 6 h is 3.687 kg.

6. QUESTION: A brass boiler has a base area of 0.15 m2 and thickness 1.0 cm. It boils water at
the rate of 6.0 kg/min when placed on a gas stove. Estimate the temperature of the
part of the flame in contact with the boiler. Thermal conductivity of brass= 109 J s –1 m–1 K–
1
; Heat of vaporisation of water = 2256 × 103 J kg–1.
ANSWER: Base area of the boiler, A = 0.15 m2
Thickness of the boiler, l = 1.0 cm = 0.01 m
Boiling rate of water, R = 6.0 kg/min
Mass, m = 6 kg
Time, t = 1 min = 60 s
Thermal conductivity of brass, K = 109 J s –1 m–1 K–1
Heat of vaporisation, L = 2256 × 103 J kg–1
The amount of heat flowing into water through the brass base of the boiler is given by

Where,
T1 = Temperature of the flame in contact with the boiler
T2 = Boiling point of water = 100°C
Heat required for boiling the water:
Q = mL ... (ii)
Equating equations (i) and (ii), we get

Therefore, the temperature of the part of the flame in contact with the boiler is 237.98°C

7. QUESTION: What is greenhouse effect for the atmosphere of the earth and what is its
importance?
ANSWER: Greenhouse effect is the phenomenon which keeps the earth’s surface warm at
night. The radiation from the sun heats up the earth. Due to its lower temperature, the
earth re-radiates it mostly in the infrared region. These infrared radiations cannot pass
through the lower atmosphere, they get reflected back by gas molecules. Low lying clouds
also reflect them back to earth. These radiations heat up the objects on the earth’s surface
and so keep the earth’s surface warm at night.
8. QUESTION: Define coefficient of thermal conductivity. Write its SI and CGS units.
ANSWER: It is defined as the quantity of heat energy that flows in unit time between the
opposite faces of a cube of unit side, the faces being kept at one degree difference of
temperature. Its CGS unit is cals-1cm-1 0C-1 and SI unit is Js-1m-1k-1 or Wm-1k-1.
9. QUESTION: Explain the three modes of transfer of heat from one object to other.
ANSWER: The three modes of transfer of heat are as follows-:
a) Conduction-: It is a process in which heat is transmitted from one part of a body to
another at a lower temperature through molecular collisions , without any actual flow of
matter. Solids are heated by the process of conduction
b) Convection-: It is the process by which heat flows from the region of higher
temperature by the actual movement of the material particles. Fluids (liquids and gases )
are heated by the process of convection in which buoyancy and gravity play an important
role.
c) Radiation-: It is the process by which heat is transmitted from one place to another
without heating the transmitting medium. The heat from the sun reaches the earth by the
process of radiation.
10. QUESTION: Define the term latent heat capacity and its various types .
ANSWER: Latent heat -: The amount of heat required to change the state of unit mass of a
substance at constant temperature and pressure. It is given as
Q=mL
Where m is the mass of the substance undergoes a change from one state to another and
L is the latent heat of the substance. Its SI unit is J/Kg
Latent heat is broadly defined in two different ways-:
a) Latent heat of fusion -: The amount of heat required to change the state of unit mass of
a substance from solid to liquid at its melting point . It is denoted as Lf
b) Latent heat of vaporization-: The amount of heat required to change the state of unit
mass of a substance from liquid to vapour at its boiling point. It is denoted by L v.

11. QUESTION: What is meant by coefficient of linear expansion , superficial expansion and
cubical expansion ? Derive the relation between them.
ANSWER: a) Coefficient of linear expansion(α) -: For solid rod ,it is defined as the increase
in length per unit original length per degree rise in its temperature. The unit of α is 0C-1 or K-
1

b) Coefficient of superficial expansion(β)-: For metal sheet , it is defined as


increase in its surface area per unit original surface area per degree rise in its temperature.
The unit of α is 0C-1 or K-1
c) Coefficient of cubical expansion(γ)-: It is defined as the increase in volume per
unit original volume per degree rise in temperature. The unit of γ is 0C-1 or K-1

We know that,
L =L0(1+αΔT) α= coefficient of linear expansion
A= A0(1+βΔT) β= coefficient of aerial expansion

V= V0(1+γΔT) γ= coefficient of cubical expansion.

So, now

V= V0 + γV0ΔT

V= V0 (1+γΔT)

L³= L0³ (1+αΔT)³

L³= L0³(1+3αΔT + 3α²ΔT² +α³ΔT³)

L³= L0³(1+3αΔT)

{Neglecting 3α²ΔT² and α³ΔT³ because they are very smaller than 1}

L³= L0³(1+3αΔT)

V= L0³(1+3αΔT)

V0 (1+γΔT) = V0(1+3αΔT)

1+γΔT = 1+3αΔT

γΔT = 3αΔT

γ=3α

And β=2α

A= A0 (1+βΔT)

L²= L0²(1+αΔT)²

A= L0²(1+2αΔT+α²ΔT²)

A= A0 (1+2αΔT)

A0(1+βΔT) = A0(1+2αΔT)

{α²ΔT² Neglecting them due to very smaller volume}

β=2α

α:β:γ=1:2:3
12. QUESTION: What are convection currents? What role do they play in relation to trade winds
and monsoons ?
ANSWER: Convection currents are those currents which are created when fluids are heated
from below. When heated the hot portion of fluid at bottom expands and become less
dense. Because of buoyancy, this lighter portion rises up. The denser colder fluid takes its
place by moving downwards which set up a current known as convection current in the
fluid.
Role of convection current
a) In the formation of trade winds.- Natural convection plays an important role in the
formation of trade winds. The surface of the earth and hence the air above it near the
equator gets strongly heated by the sun. The heated air expands and rises upwards. The
colder air from polar region rushes in towards the equator. This produces northward wind
in northern hemisphere and southward in southern hemisphere. Due to rotation of the
earth about its axis from west to east, the air close to the equator has an eastward speed of
1600kmh-1, while it is zero close to the poles. As a result, the actual direction of the wind in
the northern hemisphere is north east and in the southern hemisphere , south west. These
winds are called trade winds.
b) Monsoons- Water has much more specific heat than soil or rock. In summer, the land
mass of the Indian subcontinent gets much hotter than the Indian ocean. This sets up
convection current with hot air from land rising and moving towards the Indian ocean, while
the moisture laden air from the ocean moves towards the land. When obstructed by
mountains , the moist air rushes upwards to great height and gets cooled. The moisture
condenses and causes wide-spread rains in India. In winter, the landmass is cooler than the
ocean. Winds blow from the land to ocean. These winds take up moisture as they pass the
Bay of Bengal and cause rainfall in Tamilnadu and Srilanka.
13. QUESTION: State and explain
a) Stefan’s Boltzmann law
b) Wein’s displacement law
ANSWER: Stefan’s Boltzmann law-It states that the total heat energy emitted by a perfect
black body per second per unit area is directly proportional to the fourth power of the
absolute temperature of its surface. Thus
E α T4 or E=σT4
If H is the rate of radiant energy emitted by a black body of surface area A, then Stefan-
Boltzmann law takes the form
H=EA=σT4A
Here σ is a universal constant called Stefan-Boltzmann constant. The above relation is
known as Stefan-Boltzmann law
In SI units σ=5.67x10-8Js-1m-2k-4 or 5.67x10-8Wm-2k-4
In CGS unit σ=5.67x10-5ergs-1cm-2k-4
If a black body is in enclosure at temperature T0,then the rate at which the black body
absorbs radiation from the enclosure is σT04. Therefore the net loss of energy by the black
body per unit time per unit area is
E=σ(T4- T04)
If the body is not a perfect black body and has emissivity ϵ, then the above relation get
modified as follows:
E= ϵσT4
E= ϵσ(T4- T04)
Wein’s displacement law-It states that the wavelength (λm) corresponding to which the
energy emitted by a black body is maximum is inversely proportional to its absolute
temperature (T). Mathematically,
λm α 1/T or λmT=b
Where b is Wien’s constant . Its value is 2.9x10-3mK.
Wien’s law can be used to estimate the surface temperatures of the moon, sun and other
stars. Light from the moon shows a maximum intensity at λm=14μm.By applying Wien’s law ,
the temperature of the surface of the moon turns out to be 200K.Similarly solar radiation
shows a maximum at λm=4753A0.This corresponds to a surface temperature of 6060K.

ADDITIONAL QUESTIONS (ONE MARK QUESTIONS):


1. QUESTION: What is heat ?
ANSWER: Heat is a form of energy which produces in us the sensation of warmth .
2. QUESTION: What are the SI and CGS unit of heat? How are they related ?
ANSWER: SI unit of heat is joule and CGS unit of heat is calorie .1 calorie = 4.18 joule
3. QUESTION: What physical changes may be observed , if an object is heated ?
ANSWER: When an object is heated, the physical changes such as expansion, contraction,
change of states , change of electrical properties etc . are observed.
4. QUESTION: Define temperature
ANSWER: Temperature is the degree of hotness of a body. It is a condition which determines
the direction of flow of heat , when the two bodies are placed in contact with each other.
5. QUESTION: Name the various thermometric properties.
ANSWER: Some of the thermometric properties are length , volume, pressure , electrical
resistance , thermoelectric emf etc
6. QUESTION: What is the value of 00F on the kelvin scale ?
ANSWER: 00F = 255.23K
7. QUESTION: What is temperature at which Celsius and Fahrenheit scales give the same
reading ?
ANSWER: -400C = -400F

8. QUESTION: What is the minimum possible temperature ?


ANSWER: 0K
9. QUESTION: What are the units of α, β , γ ?
ANSWER: All have same units ,K-1 or 0C-1
10. QUESTION: Tea gets cooled , when sugar is added to it , Why ?
ANSWER: When sugar is added to tea, its heat gets shared by sugar . So the temperature of
the tea decreases.
11. QUESTION : Write the SI unit of specific heat.
ANSWER : JKg-1K-1
12 QUESTION : If the temperature of a black body is increased from 300K to 900K , by what
factor the rate of emission will increase ?
ANSWER : E α T4
13 QUESTION :What is the dimensional formula of the coefficient of thermal conductivity
ANSWER : [ MLT-3K-1 ]
14 QUESTION : What happens when CO2 at 4 atm pressure is cooled from room temperature at
constant pressure?
ANSWER : It condenses to solid directly without passing through the liquid phase.
15 QUESTION : Why does the temperature of every part of a metallic rod becomes constant in
steady state ?
ANSWER : In steady state , no part of the rod absorbs heat
CHAPTER 12
THERMODYNAMICS
GIST OF LESSON:
Thermal Equilibrium:- Two systems are said to be in thermal equilibrium with each other if they have
the same temperature

Thermo dynamical system:- An assembly of large numbers of particles having same temperature,
pressure etc is called thermo dynamical system.

Thermodynamic variables :- The variables which determine the thermodynamic behavior of a system
are called thermodynamic variables

Zeroth law of Thermodynamics :- IT states that if two system A and B are in thermal equilibrium with a
third system C , then the two system A and B are also in thermal equilibrium with each other.

Thermodynamic Process :- A thermodynamic process is said to be taking place , if the thermodynamic


variable of the system change with time.

Types of thermodynamic Process:-

Isothermal process – process taking place at constant temperature.

Adiabatic process – process where there is no exchange of heat.

Isochoric process – process taking place at constant volume

Isobaric process –Process taking place at constant Pressure.

Cyclic process:- Process where the system returns to its original state.

Equation of state : A relation between pressure, volume and temperature for a system is called its
equation of state .

Indicator diagram (P-V diagram) :- The graphical representation of the state of a system with the help
of two thermodynamical variables is called indicator diagram of the system.

Internal energy of a gas :- It is the sum of kinetic energy and the intermolecular potential energy of the
molecules of the gas. Internal energy is a function of temperature.

First law of Thermodynamics :- It states that if an amount of heat dQ I added to a system , a part of
heat is used in increasing its internal energy while the remaining part of heat may be used up as the
external work done dW by the system.

Mathematically dQ=dU+dW
dQ=dU+ PdV.
Work done during expansion / compression of gas:- When the volume of gas changes from V1 to V2 ,

the work done is given by W= = Area under the P -V diagram.


Thermodynamical operations are

(1) Isothermal process : A thermodynamic process that takes place at constant temperature is
called an isothermal process.

Equation of state for isothermal process : PV = constant.

Work done during an isothermal process

Wiso = RT loge = 2.303 RT loge

Adiabatic process : A thermodynamic process that takes place in such a manner that no heat enters or
leaves the system is called adiabatic process
Equation of state for adiabatic process

(i) PV γ = constant (ii) TV γ-1 =constant (iii)

=constant

Work done during adiabatic change

Wadia =

Reversible process :- It is a process in which the system can be retraced to its original state by
reversing the condiditions.

Irreversible process:- It is a process in which the system cannot be retraced to its original state by
reversing the conditions.

Second law of thermodynamics:


Kelvin’s statement of second law – It is impossible to derive a continuous supply of work by cooling a
body to a temperature lower than that of the coldest of its surrounding.

Clausius statement of second law – It is impossible for a self – acting machine unaided by any external
agency to transfer heat from a body to another body at higher temperature.

Heat Engine – a heat engine is a device for converting heat energy continuously into a mechanical
work.

Component of heat engine- (i) source of heat (ii) Sink (iii) Working substance

Efficiency of heat Engine :-It is defined as the ratio of the external work obtained to the amount of
heat energy absorbed from the heat source.

Mathematically

ȵ= = =1-
Carnot’s heat Engine :- it is an ideal heat Engine which is based on carnot’s reversible cycle.

Efficiency of carnot’s heat Engine

ȵ= 1- = 1-

Refrigerator or Heat pump:- it is heat engine working backward.

Co-efficient of performance : It is the ratio of heat absorbed from cold body to the work done by the
refrigerator.

Mathematically β= = =
CHAPTER 12
THERMODYNAMICS

CASE STUDY BASED QUESTIONS:


CASE 1. TOPIC: Thermodynamic process are characterized by changes in Pressure(P),
Volume(V), Temperature(T) and Internal energy (U). If Qis the heat absorbed by
the system and W is the work done by it, then ∆U = Uf - Ui = Q – W.
For an isothermal change, T = constant, ∆𝑇 = 0, ∆𝑈 = 0, Q = W.
For an adiabatic change, Q = 0, W = - ∆𝑈 = -nCv ∆𝑇,
The P – V diagram of 0.2 mole of diatomic ideal gas is shown in figure, where
the process BC is adiabatic. The value of 𝛾 for this gas is 1.4.

(i). The value of pressure at B is


(A) 1.5 atm
(B) 2 atm
(C) 3 atm
(D) 2.5 atm
(ii). The value of volume at C is
(A) 2 L
(B) 3 L
(C) 7.5 L
(D) 8 L
(iii). The value of ∆𝑄 for the process B → C is
(A) 0 J
(B) 100 J
(C) 200 J
(D) 1250 J
(iv). The value of ∆𝑈 for the process B→C is
(A) 0 J
(B) - 602 J
(C) 602 J
(D) 1200 J
(v). During adiabatic process, The pressure of gas is found to be proportional to
cube of its temperature. The ratio of Cp/CV for the gas is
(A) 4/3
(B) 2
(C) 5/3
(D) 3/2
CASE 2. TOPIC: First law of thermodynamics
As per first law of thermodynamics, The change in internal energy of a system is
equal to the heat absorbed by the system minus the work done by the system.
If the initial and final states of pressure, volume and temperature differ by finite
amount then, ∆U = ∆Q – ∆W.
By applying First law of thermodynamics one can derive equation of state for
Isothermal and adiabatic process as well as the work done during an isothermal
and Adiabatic Process.

(i). First law of thermodynamics is most general form of


(A) law of conservation of energy.
(B) law of conservation of charge.
(C) law of conservation of linear momentum.
(D) none of the above
(ii). Work done by a gas in non-cyclic process is given as the area enclosed by
the P → V curve and the
(A) Pressure axis
(B) Volume axis
(C) Temperature axis
(D) None of the above
(iii). A process in which no heat is transferred to or from the system though
other variables (P, V, T) may change, is called
(A) An isothermal process (B) An adiabatic process
(C) An isobaric process (D) An isochoric process
(iv). Which of the following statement is correct?
(A) In isothermal process, the total heat absorbed is equal to the work done
by the system.
(B) In adiabatic process, the total heat absorbed is equal to the work done
by the system.
(C) In isobaric process, the total heat absorbed is equal to the work done
by the system.
(D) In cyclic process, the total heat absorbed is equal to the work done
by the system.
(v). The slopes of the isothermal and adiabatic curves are related as
(A) isothermal curve slope = adiabatic curve slope
(B) isothermal curve slope = 𝛾 adiabatic curve slope
(C) adiabatic curve slope = 𝛾 isothermal curve slope
(D) adiabatic curve slope = ½ isothermal curve slope
CASE 3. TOPIC : Adiabatic process: A process in which no heat is transferred to or from
the system though other variables (P, V, T) may change, is called an adiabatic
process. In this process ∆𝑄 = 0.
For an adiabatic process ∶ 𝑃𝑉 𝛾 = constant
𝑅
Work done during adiabatic process : W = (𝛾−1)(T1 – T2)

Two moles of helium gas are taken over the cycle ABCDA as shown in the P → T
diagram as below.

(i). Assuming the gas to be ideal, the work done on the gas in taking from A to B
is
(A) 200 R
(B) 300 R
(C) 400 R
(D) 500 R
(ii). The work done on the gas in taking it from D to A is
(A) – 414 R (B) + 414 R
(C) – 690 R (D) + 690 R
(iii). The net work done on the gas in the cycle ABCDA is
(A) zero
(B) 276 R
(C) 1076 R
(D) 1904 R
(iv). During an adiabatic process, the pressure of gas is found to be proportional
to the cube of its temperature. The ratio of Cp/CV is
(A) 4/3
(B) 2
(C) 5/3
(D) 3/2
(v). A gas is suddenly compressed to ¼ of its original volume. What will be the
rise in temperature if the original temperature being 27˚C, 𝛾 = 1.5
(A) 100 K
(B) 200 K
(C) 300 K
(D) 400 K

ANSWERS(CASE STUDY BASED QUESTIONS):


ANS (i) ANS (ii) ANS (iii) ANS (iv) ANS (v)
CASE 1 B C A B D
CASE 2 A B B D C
CASE 3 C B B D C
TWO MARK QUESTIONS:
1. QUESTION: State the Second law of thermodynamics and write 2 applications of it?
ANSWER: According to second law of thermodynamics, when a cold body and a hot
body are brought into contact with each other, heat always from hot Body to the
cold body. Also, that no heat engine that works in cycle completely converts heat
into work.
Second law of thermodynamics is used in working of heat engine and of
refrigerator.
2. QUESTION: Why is conversion of heat into work not possible without a sink at lower
temperature?
ANSWER: For converting heat energy in to work continuously a part of heat energy
absorbed from the source has to be rejected. The heat energy can be rejected only
to a body at lower temperature which is sink, so we require a sink to convert heat
into work.
3. QUESTION: Write the sign conventions for the heat and work done during a
thermodynamic process?
ANSWER: 1) When heat is supplied to a system d Q is taken positive but when heat
is supplied by a system, d Q is taken negative.

2) When a gas expands, d w is taken as positive but when a gas compresses, work
done is taken as negative.

4. QUESTION: How does second law of thermodynamics explain expansion of gas?


ANSWER: Since from second law.

d S ≥ O d S = change in entropy

During the expansion of gas, the thermodynamic probability of gas is larger and
hence its entropy is also very large. Since form second law, entropy cannot decrease
∴ following the second law, gas molecules move from one partition to another.

5. QUESTION: Kelvin and Clausius’s statements of the Second law of thermodynamics


are equivalent. Explain?
ANSWER: Suppose we have an engine that gives a continuous supply of work when
it is cooled below the temperature of its surroundings.
This is a violation of Kelvin’s statement. Now if the work done by the engine is used
to drive a dynamo which produces current and this current produces heat in a coil
immersed in hot water, then we have produced a machine which causes the flow of
heat from a cold body to the hot body without the help of an external agent. This is
a violation of Clausius’s statement. Hence both statements are equivalent.
6. QUESTION: What is the ratio of slopes of P-V graphs of adiabatic and isothermal
process?
ANSWER: The slope of P-V graph is

For an isothermal process, (PV = constant)

So,

For an adiabatic process ( PVY = constant)

Divide 2) by 1)

So, the ratio of adiabatic slope to isothermal slope is Y.

7. QUESTION: Differentiate between isothermal and adiabatic process?


ANSWER:
Isothermal process Adiabatic process
1. In this, temperature remains 1. In this, no heat is added or removed.
constant
2. It occurs slowly 2. It occurs suddenly.
3. Here, system is thermally conducting 3. Here, system is thermally insulated
to surroundings from surroundings.
4. State equation :→ PV = constant 4. State equation → PVY = constant.
8. QUESTION: Two identical samples of gas are expanded so that the volume is
increased to twice the initial volume. However, sample number 1 is expanded
isothermally while sample number 2 is expanded adiabatically. In which sample is
the pressure greater? Why?
ANSWER: Pressure is greater in sample number 1 as can be explained: For
isothermal expansion.
P1V1 = P2V2 for no. 1 sample
Now V2 = 2V1
∴ P1V1 = P22V1
or
P2 = P1 /2 …(i)
Now for adiabatic expansion (for sample 2)
P 1V 1γ = P 2V 2γ
or
𝑉1 𝑉1
P2 = P1 ( 𝑉2 ) γ = P1 ( 2𝑉1 ) γ
= 𝑃1 …….(ii)

∴ From (i) and (ii) we find that pressure is greater in sample 1 as γ > 1.

9. QUESTION: State and explain the zeroth law of thermodynamics?


ANSWER: It states that if two systems A and B are in thermal equilibrium with a
third system C, then A and B must.be in thermal equilibrium with each other.

Explanation: The three systems are shown in the figure. Let T1, T2, T3 be the
temperatures of A, B, and C respectively.

Systems A and C, B and C will exchange heat and after a certain time, they will attain
thermal equilibrium separately.
i.e. T1 = T3 ….(1)
and T2 = T3 …. (2)

Thus from (1) and (2),


T1 = T2
i.e. A and B are now in thermal equilibrium with each other.

10. QUESTION: A cylinder with a movable piston contains 3 moles of hydrogen at


standard temperature and pressure. The walls of the cylinder are made of a heat
insulator, and the piston is insulated by having a pile of sand on it. By what factor
does the pressure of the gas increase if the gas is compressed to half its original
volume?
ANSWER: As the gas is completely insulated, the process is adiabatic.
THREE MARK QUESTIONS:
1. QUESTION: A motor car tyre has a Pressure of 2 atmospheres. At a room
temperature of 270C. The temperature at which the tyre suddenly bursts will be:

2. QUESTION: Two different adiabatic paths for the same gas intersect two isothermals

at T1 and T2 as shown in P-V diagram. How does Compare with ?

ANSWER: Now, A B and C D are isothermals at temperature T1 and T2 respectively


and BC and AD are adiabatic.

Since points A and D lie on the same adiabatic.

T1 VA Y-1 = T2 VDY-1
Also, points B and C lie on the same adiabatic,

or T1VB Y-1 = T2VCY-1

From equation 1) & 2)

3. QUESTION: A certain gas at atmospheric pressure is compressed adiabatically so


that its volume becomes half of its original volume. Calculate the resulting pressure?
ANSWER: Let the original volume, V1=V

∴ final volume V2 = (∴volume become half)

Initial pressure P1 = o.76m of Hg column

Final pressure P2 after compression =?

As the change is adiabatic, so

Y= = 1.4 for air

P2 = P1

= 0.76 ×
P2 = 0.76 × (2)1.4

P2 = 2m of Hg column

P2 = hsg

P2 = 2.672 x 105 N|m2

P2 = 2× (13.6x103)× 9.8

h = height of Hg column

s = Density of air

g = Acceleration due to gravity

4. QUESTION: Calculate the work done by the gas in going from the P-V graph of the
thermodynamic behavior of a gas from point A to point B to point C?

ANSWER: Work done by the gas in the process A → B is

W1 = - (area under curve A B)

= -

=-

P A B = 500 Pa

= 5×105 N|m2

5. QUESTION: An electric heater supplies heat to a system at a rate of 100W. If system


performs work at a rate of 75 Joules per second. At what rate is the internal energy
increasing?
ANSWER: Heat is supplied to the system at a rate of 100 W.

∴Heat supplied, Q = 100 J/s

The system performs at a rate of 75 J/s.

∴Work done, W = 75 J/s

From the first law of thermodynamics, we have:

Q=U+W

Where,

U = Internal energy

∴U = Q–W

= 100 –75
= 25 J/s
= 25 W
Therefore, the internal energy of the given electric heater increases at a rate of 25
W.
6. QUESTION: Temperature of a system containing 1 mole ideal gas increases by on
amount ΔT = 30 K as a result of heating at constant pressure. If gas obtains an
amount of heat Q= 3.2 KJ. then calculate
(a) work done by the gas
(b) change in its internal energy
(c) value of γ= Cp / Cv
ANSWER: (a) Work done when system undergoes isobaric process is
W = PdV
from ideal gas equation
PV = μRT
here μ = 1
=> PdV = RμT
or W = RΔT
, = 8.314 x 80
= .665 KJ
(b) from first law of thermodynamics
ΔU = Q- W = Q- RΔT
given that Q= 3.2 KJ
ΔU = 3.2 KJ - .665 KJ
= 2.536 KJ
(C) Again the know that
ΔU = μCvΔT
= RΔT / γ-1
γ-1 = RΔT / ΔU
or γ= (RΔT / ΔU) +1
= (.665 KJ x 8.314/ 2.535KJ) +1
= 1.26
7. QUESTION: At 00C and normal atmospheric pressure, the volume of 1g of water
increases from 1cm3 to 1.091 cm3 on free zing. What will be the change in its
internal energy? Normal atmospheric pressure is 1.013x105 N|m2 and the latent
heat of melting of ice is 80 cal/g?
ANSWER: Since, heat is given out by 1 g of water in freezing is

m = Mass of water = 1 g

Q = - (mLf) Lf = Latent heat of melting of ice = 80 cal|g

During freezing, the water expands against atmospheric pressure. Hence, external
work done (W) by water is :- W = P × ∆ V

P = 1.013×105 N|m2; ∆ V = 1.091 – 1 = 0.091 cm3 = 0.091 × 10-6 m3

∆ V = V2 – V1; V2 = final volume = 1.091 cm3

V1 = Initial volume = 1 cm3

So, W =

W = 0.0092 J

Since, 1 cal = 4.2J so,

W=

Since the work has been done by ice, it will be taken positive.

Acc. to first law of thermodynamics,

Q = ∆∪ + W ∆∪ = change in internal energy

So, ∆∪ = Q – W

= -80 – 0.0022 (using 1 and 2)

= -80.0022 cal
8. QUESTION: Consider the cyclic process A B C A on a sample 2 mol of an ideal gas as
shown. The temperature of the gas at A and B are300 K and 500K respected. Total
of 1200 J of heat is withdrawn from the sample. Find the work done by the gas in
part BC?

ANSWER: The change in internal energy during the cyclic process is zero. Therefore,
heat supplied to the gas is equal to work done by it,

∴ WAB + WBC + WCA = - 1200J →(1)

(- ve because the cyclic process is traced anticlockwise the net work done by the
system is negative)

The work done during the process AB is

WAB = PA (VB-VA) = nR(TB-TA) (PV=nRT)

WAB = 2×8.3(500-300) = 3320J →2)

R = Universal gas constant

N = No. of volume

Since in this process, the volume increases, the work done by the gas is positive.

Now, WCA = O ( volume of gas remains constant)

∴ 3320 + WBC + O = - 1200 (Using equation 1) & 2)

WBC = - 1200 – 3320

WBC = - 4520 J

9. QUESTION: The following figure shows a process A B C A per formed on an ideal gas,
find the net heat given to the system during the process?
ANSWER: Since the process is cyclic, the change in internal energy is zero.
Therefore, the heat given to the system is equal to work done by it. The net work
done by the gas in the process ABCA is:-

W = WAB + WBC + WCA


Now WAB = O

During the path BC, temperature remains constant. So it is an isothermal process.

So, WBC = nRT2 Loge

During the CA, Vα T so that is constant.

∴ Work done by the gas during the part CA is :-

WCA = P (V1 – V2)

= nR (T1 – T2)

= - nR (T2 – T1) → Using equation 1)

W= O + nR T2 Loge - nR (T2 – T1)

10. QUESTION: The specific heat capacity of water is 4200 J/kg K. What is the
change In the internal energy of 5 kg of water when it is heated from 20°G
to 80°C?
ANSWER: Here, C = 4200 J/kg K
m = 5 kg
T1 = 20°C + 273 = 293 K
T2 = 80 + 273 = 353 K
ΔT = T2 – T1
= 353 – 293 = 60 K
ΔU = ?

If ΔQ be the heat supplied to water for its heating, then using


ΔQ = mCΔT, we get
ΔQ = 5 × 4200 × 60
= 1260 KJ
Let us ignore the slight expansion in the volume of water,
∴ ΔW = PdV = 0
∴ ΔQ = ΔU + ΔW, gives
ΔU = ΔQ = 1260 KJ.
11. QUESTION: Derive an expression for the work done during isothermal expansion?

ANSWER: Consider one gram mole of ideal gas initially with pressure, volume and
temperature as P, V, T, Let the gas expand to a volume V2, when pressure reduces to
P2 and at the same temperature T

If A = Area of cross – section of piston

Force = Pressure × Area

F=PxA

If we assume that piston moves a displacement d x,

the work done : → d w = F d x

dw=P×A×dx

dw=P×dv

Total work done in increasing the volume from V1 to V2

W=

Since, PV = RT (from ideal gas equation)

P=

W=

W = RT

W = R T Loge

W=RT
W = R T Loge

W = 2.3026 R T Log 10

As P1 V1 = P2 V2

So W = 2.3026 R T Log 10

12. QUESTION: Calculate the work done during the adiabatic process. What happens to
the temperature during the process when a) W > 0 b) W < 0 ?

ANSWER: It implies how much work is done during adiabatic change of an ideal gas.
Initially ideal gas is at Pressure P1, Volume V1 and Temperature T1 (P1,V1,T1)
Final state of an ideal gas Pressure P2,Volume V2 and Temperature T2 (P2,V2,T2)
P V γ = const
γ =Cp/Cv
If an ideal gas undergoes a change in its state adiabatically from (P1, V1) to (P2, V2)
P1V1 γ = P2V2 γ
The work done in an adiabatic change of an ideal gas from the
state (P1, V1, T1) to the state (P2, V2, T2).
W =∫ P V dV = P ∫V dV (Integrating between the limits V2 and V1)
For Adiabatic Process

P V γ = constant This implies P= constant / V γ


W = constant ∫dV/ V γ
constant [V γ-1/- γ+1]
constant/1- γ [V21- γ – V1 1-γ]
= constant/1- γ[1/ V21- γ- 1/ V1 1-γ]
By solving Work done W= R/ (γ-1)(T2-T1), where
T2= final Temperature
T1=initial temperature
R=Universal gas constant
γ = Specific heat ratio
This is the work done during adiabatic change.
Consider W= R/ (γ-1)(T2-T1)
Case 1: W>0 (when T1>T2)

Temperature of the gas decreases.

Case 2:- W< 0 (T1<T2)

Temperature of the gas increases.

ADDITIONAL QUESTIONS (ONE MARK QUESTIONS):

1. QUESTION: What is the specific heat of a gas in an isothermal process and in an


adiabatic process? Why?
ANSWER: It ¡s infinite in isothermal process because ΔT = 0 (C = ΔQ/mΔT) and zero
in an adiabatic process as ΔQ = 0.
2. QUESTION: What is the nature of the P – V diagram for isobaric and isochoric
processes?
ANSWER: The P – V diagram for an isobaric process is a straight line parallel to the
volume axis while for an isochoric process; it is a straight line parallel to the
pressure axis.
3. QUESTION: When the air of the atmosphere rises up, it cools. Why?
ANSWER: When air rises up, it expands due to a decrease in the atmospheric
pressure and thus temperature falls. So it cools.
4. QUESTION: Which Thermodynamic variable is defined by the first law of
thermodynamics?
ANSWER: Internal energy
5. QUESTION: Why does the gas get heated on compression?
ANSWER: Because the work done in compressing the gas increases the internal
energy of the gas.
6. QUESTION: Which thermodynamic variable is defined by Zeroth law of
thermodynamics?
ANSWER: Temperature
7. QUESTION: What is the foundation of Thermodynamics?
ANSWER: The foundation of thermodynamics is the law of conservation of energy
and the fact the heat flows from a hot body to a cold body.
8. QUESTION: Draw a p – v diagram for isothermal and adiabatic expansion.
ANSWER:
9. QUESTION: Can a gas be liquefied at any temperature by increase of pressure
alone?
ANSWER: No, a gas can be liquefied by pressure alone, only when temperature of
gas is below its critical temperature.
10. QUESTION: Explain why two isothermal curves cannot intersect each other?
ANSWER: If they intersect, then at the point of intersection, the volume and
pressure of the gas will be the same at two different temperatures which is not
possible.
11. QUESTION: What is the source of energy when gas does work when expands
adiabatically?
ANSWER: During adiabatic expansion, the temperature and hence the internal
energy of the gas decreases. Thus work is done by the gas at the cost of its internal
energy.
12. QUESTION: A certain amount of work is done by the system in a process in which no
heat is transferred to or from the system. What happens to the internal energy and
the temperature of the system?
ANSWER: The temperature of the system decreases as the system is doing work and
no heat transfer is allowed to or from the system. As the temperature of the system
decreases, the internal energy of the system also decreases.
13. QUESTION: What factors determine the internal energy of a real gas?
ANSWER: Volume and temperature determine the internal energy of a real
gas.
14. QUESTION: Define an adiabatic process.
ANSWER: It is defined as the process in which no heat enters or leaves the
system.
15. QUESTION: Why is latent heat of vaporization of a material greater than that of
latent heat of fusion?
ANSWER: When a liquid change into a gas, there is large increase in the volume and
a large amount of work has to be done against the surrounding atmosphere and
heat associated with change from solid to gas is latent heat of vaporization and
hence the answer.
CHAPTER 13
KINETIC THEORY OF GASES

GIST OF LESSON:
Assumptions of Kinetic Theory of Gases
1. Every gas consists of extremely small particles known as molecules. The molecules of a given
gas are all identical but are different from those of another gas.
2. The molecules of a gas are identical spherical, rigid and perfectly elastic point masses.
3. Their molecular size is negligible in comparison to intermolecular distance (10 -9 m).
4. The speed of gas molecules lies between zero and infinity (very high speed).
5. The distance covered by the molecules between two successive collisions is known as free path
and mean of all free path is known as mean free path.
6. The number of collisions per unit volume in a gas remains constant.
7. No attractive or repulsive force acts between gas molecules.
8. Gravitational to extremely attraction among the molecules is ineffective due small masses and
very high speed of molecules.
Gas laws:
Assuming permanent gases to be ideal, through experiments, it was established that gases
irrespective of their nature obey the following laws.
1. Boyle’s Law
At constant temperature the volume (V) of given mass of a gas is inversely proportional to its
pressure (p), i.e.,
V ∝ 1/P ⇒ PV = constant

For a given gas, p1V1 = p2V2


2. Charles’ Law
At constant pressure the volume (V) of a given mass of gas is directly proportional to its
absolute temperature (T), i.e.,
V ∝ T ⇒ V / T = constant
For a given gas, V1/T1 = V2/T2
At constant pressure the volume (V) of a given mass of a gas increases or decreases by 1/273.15
of its volume at 0°C for each 1°C rise or fall in temperature.

Volume of the gas at t°C

Vt = V0 (1 + t/273.15)
where V0 is the volume of gas at 0°C.
3. Gay Lussac’s or Regnault’s Law
At constant volume the pressure p of a given mass of gas is directly proportional to its absolute
temperature T, i.e. ,

p ∝ T ⇒ V/T = constant

For a given gas,


p1/T1 = p2/T2
At constant volume (V) the pressure p of a given mass of a gas increases or decreases by
1/273.15 of its pressure at 0°C for each l°C rise or fall in temperature.

Volume of the gas at t°C, pt = p0 (1 + t/273.15)


where P0 is the pressure of gas at 0°C.
4. Avogadro’s Law
Avogadro stated that equal volume of all the gases under similar conditions of temperature and
pressure contain equal number molecules. This statement is called Avogadro’s hypothesis.
According Avogadro’s law

(i) Avogadro’s number The number of molecules present in 1g mole of a gas is defined as
Avogadro’s number.

NA = 6.023 X 1023 per gram mole


(ii) At STP or NTP (T = 273 K and p = 1 atm 22.4 L of each gas has 6.023 x 1023 molecules.
(iii) One mole of any gas at STP occupies 22.4 L of volume.

Standard or Perfect Gas Equation


Gases which obey all gas laws in all conditions of pressure and temperature are called perfect
gases.

Equation of perfect gas pV=nRT

where p = pressure, V = volume, T = absolute temperature, R = universal gas constant and n =


number of moles of a gas.

Universal gas constant R = 8.31 J mol-1K-1.


Real Gases
Real gases deviate slightly from ideal gas laws because

▪ Real gas molecules attract one another.


▪ Real gas molecules occupy a finite volume.
Real or Van der Waal’s Gas Equation
(p + a/V2) (V – b) = RT
where a and b are called van der Waals’ constants.

• The ideal gas equation is given by

𝑃𝑉 = 𝜇𝑅𝑇 = 𝑘𝐵 𝑁T

where is the number of moles.

N is the number of molecules

R and k B are universal gas constants and Boltzmann constant .

• The pressure of an ideal gas

P= (1/3). (mn/V). c2 = 1/3 ρ c2


For one mole of an ideal gas
P = (1/3). (M/V).c2
where n is the number density, m is the mass of the molecule and is the mean of the squared
speed.

• The RMS velocity of the gas molecule

Where T is the temperature.

• The translational kinetic energy is

• The mean free path

Pressure due to an ideal gas is given by


p = (1/3).(mn/V). c2 = 1/3 ρ c2
For one mole of an ideal gas
P = (1/3).(M/V).c2
of the gas. If P is the pressure of the gas and E is the kinetic energy per unit volume is E, then
P= (2/3).E

Degree of Freedom
The degree of freedom for a dynamic system is the number of directions in which it can move
freely or the number of coordinates required to describe completely the position and
configuration of the system.

It is denoted by for N.

Degree of freedom of a system is given by


f or N = 3A – R

where A = number of particles in the system and R = number of independent relations.


Degree of Freedom
1. For monoatomic gas = 3
2. For diatomic gas = 5
3. For non-linear triatomic gas = 6
4. For linear triatomic gas = 7
Specific heat of a gas
(a) At constant volume, CV = f/2 R
(b) At constant pressure, cp = (f/2 + 1) R
(c) Ratio of specific heats of a gas at constant pressure and at constant volume is given by
γ = 1 + 2/f

Law of Equipartition
The law of equipartition states that when an atom is kept under constant thermal condition, the
sum total of the molecule gets splitted up all throughout consistently where the amount of
degree of freedom is free from opposition.
For a gas under thermal equilibrium condition at a temperature T, the average Energy is given
by:
Mean Free Path
The average distance travelled by a molecule between two successive collisions is called mean
free path (γ).

Mean free path is given by

γ = kT / √2 π σ2p
where σ = diameter of the molecule, p = pressure of the gas,
T = temperature and k = Boltzmann’s constant.

Mean free path

λ ∝ T and λ ∝ 1/p
CASE STUDY BASED QUESTIONS:
CASE 1. TOPIC: KINETIC THEORY OF GASES
The molecules of a gas move in all directions with various speeds. The speeds of the
molecules of a gas increase with rise in temperature. During its random motion a
fast molecule often strikes against the walls of the container of the gas. The collisions
are assumed to be perfectly elastic i.e. the molecule bounces back with the same
speed with which it strikes the wall. Since the number of molecules is very large,
billions of molecules strike against the walls of the container every second. These
molecules exert a sizeable force on the wall. The force exerted per unit area is the
pressure exerted by the gas on the walls.
(i) According to kinetic theory of gases what is dimensional formula for pressure
(A) ML—1T-1
(B) ML-1T-2
(C) MLT2
(D) ML2T2
(ii) vrms=√3KT/m , it follows that the constant k should be expressed in units of
(A) newton per metre per Kelvin
(B) newton per Kelvin
(C) joule per Kelvin
(D) joule per kilogram per kelvin
(iii) Choose the only correct statement from the following
(A) The pressure of a gas is equal to the total kinetic energy of its molecules per
unit volume of the gas.
(B) The product of pressure and volume of a gas is always constant.
(C) the average kinetic energy of the molecules of a gas is proportional to its
absolute temperature.
(D) the root mean square speed of a molecule is proportional to the absolute
temperature of the gas.
(iv)If the temperature of a gas is increased from 270C to 9270C,the root mean
square speed of its molecules
(A) becomes half
(B) is doubled
(C) becomes four times
(D)remains unchanged
(v) The root mean square speed of oxygen gas molecule at T=320K is very nearly
equal to
(A) 300ms-1
(B) 500ms-1
(C)700ms-1
(D)900ms-1
CASE 2. TOPIC: MEAN FREE PATH
We know that molecules made up of one or more atoms constitute matter. In solids
which are tightly packed, molecules are spaced about a few A0 apart. In liquids the
separation between molecules is also about the same but the molecules are not
rigidly fixed and can move around. This enables a liquid to flow. In gases the
intermolecular distances are I tens of A0 and so molecules can travel without
colliding is called its mean free path. Detailed analysis shows that mean free path of
a gas is given by 1/√2 nπd2 where n –molecular number density and d-diameter of
gas molecule. Average time between two collisions of a molecule is called its
relaxation time
(i) Why does a liquid can flow
(A) intermolecular separation is small
(B) molecules are not rigidly fixed
(C) both A and B
(D) none
(ii) 1 A0 = -----m
(A) 10-8
(B) 10-9
(C) 10-10
(D) 10-11
(iii) Order of magnitude of mean free path of a gas = ---times of intermolecular
separation
(A) 10
(B) 100
(C) 1000
(D) 10000
(iv) Mean free path of a gas is ------------ to absolute temperature
(A) directly proportional
(B) inversely proportional
(C) proportional to square root
(D) none
(v) Mean free path of a gas molecule is ---------- pressure exerted by gas.
(A) directly proportional
(B) inversely proportional
(C) proportional to square root
(D) none
CASE 3. TOPIC :Properties of gases are easier to understand than that of solids and liquids.
This is mainly because in a gas, molecules are far from each other and their mutual
interaction are negligible except when two molecules are collide. Gas molecules
enclosed within a container are in a state of continuous random motion colliding
against each other and with the walls of the container. In an ideal gas each
molecule of a gas behaves as an independent particle and possesses momentum as
well as kinetic energy. Although molecular speeds in a gas may have all possible
values ranging from a small to a very high value but for the purpose of finding the
pressure exerted by a gas or average energy of a gas molecule we may presume
that all molecules are moving with a constant rms speed. The rms speed of the
molecules of a gas is defined as the square root of the mean of the squares of the
speeds of all the molecules in a gas sample. Mathematically
Vrms = √(v12 +v22+-------+vn2)/n
(i) Under what condition a real gas may be considered to behave as an ideal gas
(A) low pressure
(B) high temperature
(C) both
(D) none
(ii) For a given pressure and temperature rms speed of which gas is maximum
(A) hydrogen
(B) helium
(C) both
(D) none
(iii) rms speed of air under STP conditions
(A) 283m/s
(B) 383 m/s
(C) 483 m/s
(D) 583 m/s
(iv) rms speed of air will be more on
(A) dry day
(B) wet day
(C) same
(D) cannot say
(v) relation between Boltzmann constant(K) and gas constant(R)
(A) R = NK
(B) N = KR
(C) K =NR
(D) none

ANSWERS (CASE STUDY BASED QUESTIONS):

(i) (ii) (iii) (iv) (v)


CASE 1 B C C B B
CASE 2 C C B A B
CASE 3 C A C B A
TWO MARK QUESTIONS:
1 The earth without its atmosphere would be inhospitably cold. Explain Why?
Ans. The lower layers of earth’s atmosphere reflect infrared radiations from earth back to the
surface of earth. Thus, the heat radiations received by the earth from the sun during the day are
kept trapped by the atmosphere. If atmosphere of earth were not there, its surface would
become too cold to live.
2 At very low pressure and high temperature, the real gas behaves like ideal gas. Why?
Ans. An ideal gas is one which has Zero volume of molecule and no intermolecular forces. Now:
1) At very low pressure, the volume of gas is large so that the volume of molecule is negligible
compared to volume of gas.
2) At very high temperature, the kinetic energy of molecules is very large and effect of
intermolecular forces can be neglected.
Hence real gases behave as an ideal gas at low pressure and high temperature.
3 Calculate the degree of freedom for monatomic, diatomic and triatomic gas?
Ans. The degrees of freedom of the system is given by: - f = 3 N – K
Where, f = degrees of freedom
N = Number of Particles in the system.
K = Independent relation among the particles.
1) For a monatomic gas; N = 1 and K = 0
f=3X1–0=3
2) For a diatomic gas; N = 2 and K = 1
f=3X2–1=5
3) For a triatomic gas; N = 3 and K = 3
f=3X3–3
f=6
4 Air pressure in a car tyres increases during driving? Why?
Ans. During driving, the temperature of air inside the tyres increases due to motion. Acc. to
Charles’s law, pressure α Temperature, ∴ As temperature increases, Pressure inside the tyres
also increases.
5 What is an ideal perfect gas?
Ans. A gas which obeys the following laws or characteristics is called as ideal gas.
1) The size of the molecule of gas is zero
2) There is no force of attraction or repulsion amongst the molecules of gas.
6 Why the land has a higher temperature than the ocean during the day but a lower temperature
at night.
Ans. Specific Heat of water is more than land (earth). Therefore, for given heat change in temp.
of land is more than Ocean (water).
7 Although the Velocity of air molecules is nearly 0.5 km/s yet the smell of scent spreads at a much
slower rate why.
Ans. The air molecules travel along a zigzag path due to frequent collision as a result their
displacement per unit time is very small.
8 Find out the ratio between most probable velocity, average velocity and root Mean square
Velocity of gas molecules?
Ans. Since,
Most Probable velocity,

Average velocity,

Root Mean Square velocity: Vr.m.s.

So,

9 Why it is not possible to increase the temperature of a gas while keeping its Volume and
pressure constant.

Ans.
P and V are constant then T is also constant.
10 What is an average velocity of the molecules of an ideal gas?
Ans. Average velocity of the molecules of an ideal gas is zero, because molecules possess all sorts
of velocities in all possible directions, whose vector sum would be zero.

THREE MARK QUESTIONS:


1 What are the assumptions of kinetic theory of gas?
Ans. The assumptions of kinetic theory of gases are:-
1) A gas consists of a very large number of molecules which should be elastic spheres and
identical for a given gas.
2) The molecules of a gas are in a state of continuous rapid and random motion.
3) The size of gas molecules is very small as compared to the distance between them.
4) The molecules do not exert any force of attraction or repulsion on each other.
5) The collisions of molecules with one another and with walls of the vessel are perfectly elastic.
2 Equal masses of Oxygen and helium gases are supplied equal amount of heat. Which gas will
undergo a greater temperature rise and why?
1
Ans. (i) 𝜆 ∝ 𝑇 (ii) 𝜆 𝛼 𝜌 (iii) λ α m
3 Why evaporation causes cooling?
Ans. Hint- During evaporation fast moving molecules escape a liquid surface so the average
kinetic energy of the molecules left behind is decreased thus the temperature of the liquid is
lowered.
4 A tank of volume 0.3m3 contains 2 moles of Helium gas at 200C. Assuming the helium behave as
an ideal gas;
1) Find the total internal energy of the system.
2) Determine the r. m. s. Speed of the atoms.
Ans .1) n = No. of moles = 2
T = Temperature = 273+20 = 293K
R = Universal Gas constant = 8.31 J/mole.

Total energy of the system = E =

2) Molecular Mass of helium = 4 g | mol

Root Mean speed = Vr.m.s


Vr.m.s. = 1.35X103 m|s
5 Isothermal Curves for a given mass of gas are shown at two different temperatures T1 and
T2 state Whether T1 > T2 or T2 > T1, justify your answer.

𝑃𝑉
Ans. 𝑇 = 𝜇𝑅 as μ and R are constant so T = P V
since PV is greater for the curve at T2 than for the curve T1 therefore T2 > T1
Three Vessels at the same pressure and temperature have same volume and contain equal
number of molecules
6 If Nine particles have speeds of 5, 8, 12, 12, 12, 14, 14, 17 and 20 m/s. find :
1) the average speed
2) the Most Probable speed
3) the r.m.s. Speed of the particles?
Ans. 1) The average speed is the sum of speeds divided by the total number of particles.

Hence, Average speed,


2) The average value of the square of speeds is given by:-

3) Three of particles have a speed of 12m|s; two have a speed of 14m|s and the remaining have
different speeds. Therefore, the most probable speed,
VmP = 12 m /s.
7 At what temperature the rms speed of oxygen atom equal to r. m.s. speed of helium gas atom at
-10°C
Atomic mass of helium = 4
Atomic mass of oxygen = 32

Ans.
Let rims speed of oxygen is (Vrms)1 and of helium is (Vrms)2 is equal at temperature T1 and
T2 respectively.

8 Three vessels of equal capacity have gases at the same temperature and pressure. The first
vessel contains neon (monatomic), the second contains chlorine (diatomic), and the third
contains uranium hexafluoride (polyatomic). Do the vessels contain equal number of respective
molecules? Is the root mean square speed of molecules the same in the three cases? If not, in
which case is the largest?
Ans. Yes. All contain the same number of the respective molecules.
No. The root mean square speed of neon is the largest.
Since the three vessels have the same capacity, they have the same volume.
Hence, each gas has the same pressure, volume, and temperature.
According to Avogadro's law, the three vessels will contain an equal number of the respective
molecules. This number is equal to Avogadro's number, N= .
The root mean square speed ( ) of a gas of mass m, and temperature T, is given by the
relation:

Where, k is Boltzmann constant


For the given gases, k and T are constants.
Hence depends only on the mass of the atoms, i.e.,

Therefore, the root mean square speed of the molecules in the three cases is not the same.
Among neon, chlorine, and uranium hexafluoride, the mass of neon is the smallest. Hence, neon
has the largest root mean square speed among the given gases.
9 The density of Carbon dioxide gas at 0°C and at a pressure of 1.0 × 106 newton/m2 is 1.98 kg/m3.
Find the root mean square velocity of its molecules at 0°C and 30°C. Pressure is kept constant.
Ans.

(Vrms)30 = 389 × 1.053 = 410 m/s


10 Show that average kinetic energy of translation per molecule of gas is directly proportional to
the absolute temperature of gas?
Ans. Acc. to kinetic theory of gases, the pressure p exerted by one mole of an ideal gas is

P= M = Mass of gas

or PV = V = Volume of gas
Since PV = RT (for 1mole of gas)

or

So, C

Also,
Dividing by number of molecules of gas = N

or

Since,

So,

as constant
11 Establish the relation between 𝛾 = 𝑐𝑝 and degrees of freedom (n)?
𝑐 𝑣
𝑐𝑝
Ans. Now 𝛾 = 𝑐𝑣
Where CP = specific heat at constant pressure
CV = Specific heat at constant volume.
and n = Degrees of freedom → is the total number of co-ordinates or independent quantities
required to describe completely the position and configuration of the system.
Suppose, a polyatomic gas molecule has ‘n’ degrees of freedom.
∴ Total energy associated with a gram molecule of the gas i. e.
N = Total number of molecules
R = Universal Gas Constant
R = NK
1 𝑛
K = Boltzmann Constant 𝐸 = 𝑛 × 2 𝐾𝑇 = 2 𝑅𝑇
As,
Specific heat at constant volume,

Now Specific heat at constant Pressure, CP = CV + R

𝑐𝑝
As, 𝛾 =
𝑐𝑣

12 You are given the following group of particles, ni represents the number of molecules with speed
vi

ni 2 4 8 6 3
Vi(ms-1) 1 2 3 4 5
Compute (i) the average speed (ii) root mean square speed (iii) the most probable speed
𝑛 𝑣 +𝑛 𝑣 +𝑛 𝑣3 +𝑛4 𝑣4 +𝑛5 𝑣5
Ans. (i) the average speed is 𝜗𝐺𝑣 = 1 1 𝑛 2+𝑛2 +𝑛3 +𝑛 +𝑛
1 2 3 4 5

2×1+4×2+8×3+6×4+3×5 73
𝑣𝑎𝑣 = =
2+4+8+6+3 23

𝑣𝑎𝑣 = 3.17𝑚𝑠 −1

(ii) The root mean square speed is,


1
𝑛1 𝑣12 + 𝑛2 𝑣22 + 𝑛3 𝑣32 + 𝑛4 𝑣42 + 𝑛5 𝑣52 2
𝑣𝛾𝑚5 = ( )
𝑛1 + 𝑅𝑒 𝑛𝐽 + 𝑛𝑦 + 𝑛5
= 3.36 ms-1

(iii) the most probable speed is that speed which is possessed by maximum number of particles;
so it is 3.0 m s-1

ADDITIONAL QUESTIONS (ONE MARK QUESTIONS):

1 What is Mean free path?


Ans. Mean free path is defined as the average distance a molecule travels between collisions. It is
represented by λ. Units are meters (m).
2 What happens when an electric fan is switched on in a closed room?
Ans. When electric fan is switched on, first electrical energy is converted into mechanical energy
and then mechanical energy is converted into heat. The heat energy will increase the Kinetic
energy of air molecules; hence temperature of room will increase.

3 State the law of equipartition of energy?


Ans. According to law of equipartition of energy, the average kinetic energy of a molecule in each
1
degree of freedom is same and is equal to 𝑘𝑇.
2

4 Why the temperature less than absolute zero is not possible?


Ans. Since mean square velocity is directly proportional to temperature. If temperature is zero
then mean square velocity is zero and since K. E. of molecules cannot be negative and hence
temperature less than absolute zone is not possible.
5 Write two condition when real gases obey the ideal gas equation (PV = nRT).
N → number of mole.
Ans. (i) Low pressure (ii) High temperature
6 Draw the graph between P and 1/W (reciprocal of volume) for a prefect gas at constant
temperature.

Ans.
7 Name the factors on which the degree of freedom of gas depends.
Ans. Atomicity and temperature

8 A container has equal number of molecules of hydrogen and carbon dioxide, If a fine hole is
made in the container, then which of the two gases shall leak out rapidly?
Ans. Hydrogen
9 On what factors, does the average kinetic energy of gas molecules depend?
Ans. Average kinetic energy depends only upon the absolute temperature and is directly
proportional to it.
10 In the number of molecules in a container is doubled. What will be the effect on the rms speed
of the molecules?
Ans. No effect
11 What is the average translational K.E. of an ideal gas molecule at a temperature T?
Ans. 32 kT, where k is Boltzmann Constant.
12 Two different gases have the same temperature. Can we conclude that the r.m.s? velocities of
the gas molecules are also the same? Why?
Ans. No. If temperature is same, then 32 kT is same. Also 12 mC2 is same. But m is different for
different gases. C will be different.
13 Comment on the use of water as a coolant.
Ans. Since water has a high value of specific heat so it can be used as a coolant.
14 Why temperature less than absolute zero is not possible?
Ans. According to the kinetic interpretation of temperature, absolute temperature means the
kinetic energy of molecules.
As heat is taken out, the temperature falls and hence velocity decreases. At absolute zero, the
velocity of the molecules becomes zero i.e. kinetic energy becomes zero. So no more decrease in
K.E. is possible, hence temperature cannot fall further.
15 From kinetic theory of gases, show that Moon cannot have an atmosphere (Assume k = 1.38 ×
10-23 J K-1 Temperature T=0°C=273K).
Ans: Vescape = Vrms= 1.86 km s-1
CHAPTER 14
OSCILLATIONS
GIST OF LESSON:
• Periodic Motion
Motions, processes or phenomena, which repeat themselves at regular intervals, are called
periodic.
• Oscillatory Motion
The motion of a body is said to be oscillatory motion if it moves to and fro about a fixed point
after regular intervals of time. The fixed point about which the body oscillates is called mean
position or equilibrium position.
• Simple Harmonic Motion
Simple harmonic motion is a special type of periodic oscillatory motion in which
(i) The particle oscillates on a straight line
(ii) The acceleration of the particle is always directed towards a fixed point on the line.
(iii) The magnitude of acceleration is proportional to the displacement of the particle from the

• Characteristics of SHM
The displacement x in SHM at time t is given by
x = A sin (ωt+ Ф )
where the three constants A, ω and Ф characterize the SHM, i.e., they distinguish one SHM
from another. A SHM can also be described by a cosine function as follows:
x = A cos (ωt + δ)
• The displacement of an oscillating particle at any instant is equal to the change in its position
vector during that time. The maximum value of displacement in an oscillatory motion on either
side of its mean position is called “displacement amplitude” or “simple amplitude”.
Thus, amplitude A = x max.
• The time taken by an oscillating particle to complete one full oscillation to and fro about its
mean (equilibrium) position is called the “time period” of SHM. It is given by

• Frequency
The number of oscillations in one second is called frequency. It is expressed in sec-1 or Hertz.
Frequency and time period are independent of amplitude.

• Phase
The quantity (ωt+ Ф) is called the phase of SHM at time t; it describes the state of motion at that
instant. The quantity Ф is the phase at time f = 0 and is called the phase constant or initial phase
or epoch of the SHM. The phase constant is the time-independent term in the cosine or sine
function.
• The force responsible for maintaining the S.H.M. is called restoring force.
If the displacement (x) from the equilibrium position is small, the restoring force (F) acting on the
body is given by
F = -kx
where k is a force constant.
• Energy in S.H.M.
When a body executes SHM, its energy changes between kinetic and potential, but the total
energy is always constant. At any displacement x from the equilibrium position:
• The force responsible for maintaining the S.H.M. is called restoring force.
If the displacement (x) from the equilibrium position is small, the restoring force (F) acting on
the body is given by
F = -kx
where k is a force constant.
• Energy in S.H.M.
When a body executes SHM, its energy changes between kinetic and potential, but the total
energy is always constant. At any displacement x from the equilibrium position:

• Springs in Series
If two springs, having spring constant k1 and k2, are joined in series, the spring constant of the
combination is given by

• Springs in Parallel
If two springs, having spring constants k1 and k2, are joined in parallel, the spring constant of the
combination is given by
k = k 1 + k2

• Simple Pendulum
A simple pendulum is the most common example of bodies executing S.H.M. An ideal simple
pendulum consists of a heavy point mass body suspended by a weightless in extensible and
perfectly flexible string from a rigid support about which it is free to oscillate.
• The time period of simple pendulum of length ‘l’ is given by

The time period of a simple pendulum depends on


(i) length of the pendulum and
(ii) the acceleration due to gravity (g).
• A second’s pendulum is a pendulum whose time period is. 2s. At a place where g = 9.8 ms -2, the
length of a second’s pendulum is found to be 99.3 cm (= 1 m).

• Undamped and Damped Simple Harmonic Oscillations


Undamped Simple Harmonic oscillations: When a simple harmonic system oscillates with a
constant amplitude which does not change with time, its oscillations are called undamped simple
harmonic oscillations.
Damped Simple Harmonic oscillations: When a simple harmonic system oscillates with a
decreasing amplitude with time, its oscillations are called damped simple harmonic oscillations.

•Free and Forced Oscillations: A system is said to execute free oscillations, if on being
displaced or disturbed from its position of equilibrium, it oscillates itself without outside
interference.
When a system is compelled to oscillate with a frequency other than its natural frequency, it is
said to execute forced oscillations.
The external force which causes forced oscillation, is of sinusoidal nature.

•RESONANT OSCILLATIONS: Resonance is the phenomenon of setting a body into oscillations with large
amplitude under the influence of some external periodic force whose frequency is NEARLY equal to the
natural frequency of the given body. Such oscillations are called the “resonant oscillations”.
• The two or more oscillations linked together in such a way that the exchange of energy takes place
between them are called coupled oscillators. The oscillations produced by coupled oscillators are known
as coupled oscillations.
CASE STUDY BASED QUESTIONS:
CASE 1. Motion of a Swing
You have noticed many objects have a tendency to return to their original location after
they have been moved slightly. You have probably also noticed most objects not only
return to their original position but continue too far, so that they may swing back and
forth several times before they come to rest. A playground swing is a convenient example
for a discussion of this kind of motion, because we have all ridden one, and more
importantly, pushed one.

This whole operation is periodic: each complete swing takes the same amount of time,
regardless of how far the swing moves. The amount of time it takes the swing to
complete its cycle depends on the length of the rope and nothing else. If you make a
graph of the distance the swing travels crossways with time, the result is a sine wave.

(i).Sana is sitting on the swing and then she stands on the moving swing, time taken by
the swing to complete one oscillation will
(A) increase
(B)decrease
(C)first increase then decrease
(D) remains same
(ii).The amplitude of swing executing SHM is 5cm. its acceleration at a distance 1cm
from the mean position is 4 cm/s2. The maximum speed of the particle in cm/s is:
(A) 5
(B) 10
(C) 15
(D) 20
(iii). Time period of pendulum having infinite length is
(A)infinte
(B) zero
(C) 84.6 min
(D) 24 hours
(iv). A girl is sitting on the swing whose time period of oscillation is t1 and then a child
comes and sit on her lap and now time period of oscillation is t 2. Which of the following
statement is correct?
(A) t1 =t2
(B)t1>t2
(C)t1<t2
(D)t1 =t2=0
(v). For SHM which of the following statement is incorrect?
(A) Restoring force is maximum at the extreme positions.
(B) Total energy of the particle always remains constant.
(C) Acceleration of the particle is maximum at the equilibrium position.
(D) restoring force is always directed towards a fixed point
CASE 2. Simple Harmonic oscillation
A particle is moving to and from about meant equilibrium position due to restoring
force f where f is given by
𝑓 = −𝑘𝑦, k is spring constant and y is displacement
y= A sinωt, velocity v= Aωcosωt, acceleration a= -ω2y = - ω2A sinωt
K.E.= ½ mv2 and P.E. = ½ ky2

(i). the shortest distance travelled by a particle executing SHM from mean position in 2
seconds is equal to ½ times its amplitude. Determine its time period.
(A) 24sec (B) 12sec (C) 1.2sec (D) 2.4sec
(ii). In a SHM, when displacement is half of the amplitude, what is the ratio of the
kinetic energy to the potential energy
(A) 1/3 (B) 2 (C) 1/2 (D) 3
(iii). If a spring of mass 30 kg has spring constant 15 N/m, then its time period, is
(A) 2π s
(B)√2 π s
(C)2√2 π s
(D)2√2 s
(iv). Phase difference between instantaneous velocity and acceleration of a particle
executing SHM is
(A) π/2
(B)π
(C)π/3
(D)3π/2
(v). A body is executing SHM. The potential energy, kinetic energy and total energy are
measured as a function of displacement y. Which of the following statement is true?
(A) K.E. is maximum when y=0
(B) T.E. =0, when y=0
(C) K.E. is maximum when y=maximum
(D) P.E. is maximum when y=0
CASE 3. Simple Pendulum:
Simple pendulum moves to and from about its mean position due to restoring torque
which is always directed towards the mean position and proportional to angle
subtended by the pendulum with the vertical
τ α - kθ and thus executing simple harmonic motion

Time period depends on length of pendulum and gravity not on the mass of pendulum
(i). the length of a simple pendulum is increased by 44%. What is the percentage change
in the time period of the pendulum?
(A) 10%
(B) 20%
(C) 30%
(D) 24%
(ii). Pendulum clock placed at equator as compared to pendulum clock placed at poles
will be
(A) slower
(B) faster
(C) reads same time
(D) not work at equator
(iii).For a simple pendulum the time period of one oscillation is given by
𝑙
(A)T=2𝜋√
𝑔
𝑙
(B)T=2𝜋√
2𝑔
2𝑙
(C)T=2𝜋√
𝑔
𝑔
(D)T=2𝜋√
𝑙
(iv).A second’s pendulum executes
(A) 1.5 oscillation per second
(B) 1.0 oscillation per second
(C) 2.0 oscillations per second
(D) 0.5 oscillations per second
(v). Two simple pendulums of time period 2.0 s and 2.1 s are made to vibrate
simultaneously. They are in phase initially. After how many vibrations they are in same
phase
(A) 30
(B) 25
(C) 21
(D) 35
CASE 4. Periodic and Oscillatory motion
A motion which repeats itself over and over again after a regular interval of time is
called periodic motion
A motion which repeats itself over and over again after a regular interval of time about
its mean position, such that it remains confined within two well defined limits (called
extreme positions) on the either side of the mean position is called oscillatory or
vibrational motion
Periodic functions Any function that repeats its value at regular intervals of arguments
and those which can be represented by sine or cosine curve are called harmonic
functions and those which cannot be represented by single sine or cosine functions are
called non harmonic functions
Simple Harmonic motion are represented by harmonic functions
(i). displacement of the mass attached to spring from is mean position (in meter) is
given by:
Y= 0.2sin (20πt + 2.5π) cos (20πt + 2.5π)
The motion of the loaded spring is
(A) periodic but not SHM
(B) non periodic
(C) simple harmonic motion with period 0.05 s
(D) simple harmonic motion with period 0.1 s
(ii). Which of the following functions represent periodic motion?
(A) e-ωt (B) sinωt+ cosωt (C) logωt (D)1/logωt
(iii). The function sin2ωt represents
(A) a simple harmonic motion with a time period 2π/ω
(B) a simple harmonic motion with a time period π/ω
(C) a periodic, but not simple harmonic motion with a time period 2π/ω
(D) a periodic, but not a simple harmonic motion with a time period 2π/ω
(iv). Which of the following is Simple Harmonic motion?
(A) Particle moving in a circle with uniform speed
(B) Wave moving through a string fixed at both ends
(C) Earth spinning about its axis
(D) ball bouncing between two rigid vertical walls
(v). Distance and displacement moved by particle in SHM in one time period is
(A) r, 0
(B) 4r, 2r
(C) 4r, 0
(D) 2r,2r
CASE 5. Free, forced, resonant, damped oscillations:
Free Oscillation Forced Oscillation Damped Resonant
Oscillation Oscillation
The free oscillation When a body oscillates
possesses constant by being influenced by The oscillation
Oscillation of
amplitude and an external periodic that fades with
object with
period without any force, it is called forced time is called
maximum
external force to set oscillation. Here, the damped
amplitude,
the oscillation. amplitude of oscillation, oscillation.
when the
experiences damping but
In such a system, the amplitude frequency of
remains constant due to
the amplitude, of oscillation the applied
the external energy
frequency and reduces with force equal to
supplied to the system.
energy all remain time. the natural
constant. For example, when you frequency of
Energy loss
push someone on a the object is
from the
swing, you have to keep called
system in
periodically pushing resonance.
overcoming
them so that the swing
external forces
doesn’t reduce.
like friction or
air resistance
and other
resistive
forces.
damping

(i). During the phenomenon of resonance


(A) the frequency of oscillation becomes large
(B) the time period of oscillation becomes large
(C) the amplitude of oscillation becomes large
(D) all of the above
(ii). When an oscillator completes 200 oscillations, its amplitude reduces to 1/5 of its
initial value. What will be its amplitude, when it completes next 200 oscillations?
(A) 1/20 (B) 1/50
(C) 2/25 (D) 1/25
(iii). When an external force is applied on an oscillating particle and reduces the
amplitude of the oscillating particle then this is called as
(A) Simple Harmonic Motion
(B)Forced Oscillation
(C) Standing Wave
(D) Circular Motion
(iv).In case of forced vibrations, the resonance becomes very sharp, when the
(A) applied periodic force is small
(B) damping force is small
(C) restoring periodic force is small
(D) none of above
(v). In the forced oscillation of a particle, the amplitude is maximum for a frequency ω 1
of the force, while the energy is maximum for a frequency ω2 of the force. Then,
(A) ω1= ω2
(B) ω1 > ω2
(C) ω1< ω1
(D) ω1< ω1, when damping is small and ω1< ω1, when damping is large

ANSWERS(CASE STUDY BASED QUESTIONS):


ANS (i) ANS (ii) ANS (iii) ANS (iv) ANS (v)
CASE 1 B A C A C
CASE 2 A D C A A
CASE 3 B A A D C
CASE 4 C B D B C
CASE 5 C D B B A
TWO MARK QUESTIONS:
1. Fig. depicts four x-t plots for linear motion of a particle. Which of Like plots
represent periodic motion? What is the period of motion (in case of periodic
motion)?

ANSWER: Figure (b) and (d) represent periodic motions and the time period of each
of these is 2 seconds, (a) and (c) are non-periodic motions.
2. The piston in the cylinder head of a locomotive has a stroke (twice the amplitude)
of 1.0 m. If the piston moves with simple harmonic motion with an angular
frequency of 200 rev/min, what its maximum speed?
ANSWER:

3. Which of the following examples represent periodic motion?


(a) A swimmer completing one (return) trip from one bank of a river to the other
and back.
(b) A freely suspended bar magnet displaced from its N-S direction and released.
(c) A hydrogen molecule rotating about its centre of mass.
(d) An arrow released from a bow.
ANSWER: (a) It is not a periodic motion. Though the motion of a swimmer is to and
fro but will not have a definite period.
(b) Since a freely suspended magnet if once displaced from N-S direction and
released, it oscillates about this position, it is a periodic motion.
(c) The rotating motion of a hydrogen molecule about its centre of mass is periodic.
(d) Motion of an arrow released from a bow is non-periodic.
4. The acceleration due to gravity on the surface of moon is 1.7 ms-2. What is the
time period of a simple pendulum on the surface of moon if its time-period on the
surface of Earth is 3.5 s? (g on the surface of Earth is 9.8 ms-2 .)
ANSWER:

5. A simple pendulum of length l and having a bob of mass M is suspended in a car.


The car is moving on a circular track of radius R with a uniform speed v. If the
pendulum makes small oscillations in a radial direction about its equilibrium
position, what will be its time period?
ANSWER:
In this case, the bob of the pendulum is under the action of two accelerations.

6. A simple pendulum consisting of an inextensible length ‘l’ and mass ‘m’ is oscillating
in a stationary lift. The lift then accelerates upwards with a constant acceleration of
4.5 m/s2. Write expression for the time period of simple pendulum in two cases. Does
the time period increase, decrease or remain the same, when lift is accelerated
upwards?
𝑙
ANSWER: When the lift is stationary, T = 2π√𝑔
When the lift accelerates upwards with an acceleration of 4.5 m/s2
𝑙
T ́ = 2𝜋√𝑔+45
Therefore, the time period decreases when the lift accelerates upwards.
7. All trigonometric functions are periodic, but only sine or cosine functions are used
to define SHM. Why?
ANSWER: All trigonometric functions are periodic. The sine and cosine functions can
take value between -1 to +1 only. So they can be used to represent a bounded motion
like SHM. But the functions such as tangent, cotangent, secant and cosecant can take
value between 0 and ∞ (both negative and positive). So these functions cannot be
used to represent bounded motion like SHM.
8. QUESTION:
Which of the following examples represent (nearly) simple harmonic motion and
which represent
periodic but not simple harmonic motion?
(a) the rotations of earth about its axis.
(b) motion of an oscillating mercury column in a U-tube.
(c) motion of a ball bearing inside a smooth curved bowl, when released from a
point slightly above the lowermost point.
(d) general vibrations of a polyatomic molecule about its equilibrium position.
ANSWER: (a) Since the rotation of earth is not to and fro motion about a fixed point,
thus it is periodic but not S.H.M.
(b) It is S.H.M.
(c) It is S.H.M.
(d) General vibrations of a polyatomic molecule about its equilibrium position is
periodic but non SHM. In fact, it is a result of superposition of SHMs executed by
individual vibrations of atoms of the molecule.
9. QUESTION: The Length of a simple pendulum executing SHM is increased by 2.1%.
What is the percentage increase in the time period of the pendulum of increased
length?
𝑙
ANSWER: Time Period, T = 2𝜋√𝑔 i.e. T ∝√𝑙. The percentage increase in time period
is given by
∆𝑇 1 ∆𝑙
× 100 = × 100 (for small variation)
𝑇 2 𝑙
1
= × 2.1%= 1.05%
2
THREE MARK QUESTIONS:
1. A spring balance has a scale that reads from 0 to 50 kg. The length of the scale is
20 cm. A body suspended from this balance, when displaced and released,
oscillates with a period of 0.6 s. What is the weight of the body?
ANSWER:

2. A spring having with a spring constant 1200 Nm-1 is mounted on a horizontal table
as shown in Fig. A mass of 3 kg is attached to the free end of the spring. The mass
is then pulled sideways to a distance of 2.0 cm and released. Determine (i) the
frequency of oscillations, (ii) maximum acceleration of the mass, and (iii) the

maximum speed of the mass.


ANSWER:
3 QUESTION: The following figures correspond to two circular motions. The radius of
the circle, the period of revolution, the initial position, and the sense of revolution
(i.e., clockwise or anticlockwise) are indicated on each figure.

Obtain the corresponding simple harmonic motions of the x-projection of the


radius vector of the revolving particle P in each case.
ANSWER:
4. QUESTION: Figure (a) shows a spring of force constant k clamped rigidly at one end
and a mass m attached to its free end. A force F applied at the free end stretches
the spring. Figure (b) shows the same spring with both ends free and attached to a
mass m at either end. Each end of the spring in Figure – (b) is stretched by the
same force F.

a) What is the maximum extension of the spring in the two cases?


(b) If the mass in Fig. (a) and the two masses in Fig. (b) are released free, what is
the period of oscillation in each case?
ANSWER:

5. Time period of a particle in SHM depends on the force constant k and mass m of the
particle:T=2𝜋√𝑚/𝑘.
A simple pendulum executes SHM approximately. Why then is the time period of a
pendulum independent of the mass of the pendulum?
ANSWER: We know that:
The time period of the simple pendulum is T=2𝜋√𝑚/𝑘.
For a simple pendulum, k is expressed in terms of mass m, as: k∝m
𝑚
Where, is a constant.
𝑘
Thus, the time period T, of a simple pendulum is independent of the mass of the bob.

6. (i)The maximum velocity of a particle, executing S.H.M with amplitude of 7mm is 4.4
m/s. What is the period of oscillation?
(ii)Which of the following relationships between the acceleration ‘a’ and
the displacement ‘x’ of a particle involve simple harmonic motion?
a) a=0.7x (b) a=-200x2 (c) a = -10x (d) a=100x3
2 2A 2  22  .007
ANSWER: (i)Vmax= A  A , T   0.01s
T Vmax 7  4.4
(ii) (c) represent SHM.
7. QUESTION: (i)Write displacement equation respecting the following condition
𝜋
obtained in SHM-:Amplitude = 0.01m,Frequency = 600Hz, Initial phase = 6

(ii)The amplitude of oscillations of two similar pendulums similar in all respect are
2cm & 5cm respectively. Find the ratio of their energies of oscillations.
ANSWER: (i)Y = A Sin (2π𝜈𝑡 + 𝜙𝑜 )
𝜋
= 0.01 Sin (1200𝜋𝑡 + 6 )

𝐸 𝐴 2 2
(ii) 𝐸1 = (𝐴1 )2 = (3) = 4: 25
2 2

8. The kinetic energy of a particle executing S.H.M. is 16J when it is in its mean
position. If the amplitude of oscillations is 25cm and the mass of the particle is
5.12kg. Calculate the time period of oscillations?
ANSWER: K. E. = Kinetic energy = 16J

Now, m = Mass = 5.12kg,W = Angular frequency,a = amplitude = 25cm or 0.25m

The Maximum value of K. E. is at mean position which is =

So,

or or

Or or or, 10 -4+6 = w2

100 = w2 therefore w = 10 rad/s.


2𝜋 2𝜋 𝜋
Now, T = Time Period = = = sec.
𝜔 10 5
9. Derive the expressions for the kinetic and potential energies of a harmonic oscillator. Hence show
that total energy is conserved in SHM. Draw graphs for (a) energy vs. time and (b) energy vs.
displacement.

10. Show that for small oscillations the motion of a simple pendulum is simple harmonic. Derive an
expression for its time period. Does it depends on the mass of the bob ?
Consider simple pendulum — a small bob of mass m tied to an inextensible massless string
of length L. The other end of the string is fixed to a rigid support. The bob oscillates in a
plane about the vertical line through the support. Fig. (a) shows this system. Fig. (b) is a kind
of ‘free-body’ diagram of the simple pendulum showing the forces acting on the bob.

Let θ be the angle made by the string with the vertical. When the bob is at the mean position,
θ=0
There are only two forces acting on the bob; the tension T along the string and the vertical
force due to gravity (=mg). The force mg can be resolved into the component mg cosθ along
the string and mg sinθ perpendicular to it. Since the motion of the bob is along a circle of
length L and centre at the support point, the bob has a radial acceleration (ω2L) and also a
tangential acceleration; the latter arises since motion along the arc of the circle is not
uniform. The radial acceleration is provided by the net radial force T –mg cosθ, while the
tangential acceleration is provided by mg sinθ. It is more convenient to work with torque
about the support since the radial force gives zero torque. Torque τ about the support is
entirely provided by the tangential component of force
τ = –L (mg sinθ )
This is the restoring torque that tends to reduce angular displacement — hence the negative
sign. By Newton’s law of rotational motion, τ = I α
where I is the moment of inertia of the system about the support and α is the angular
acceleration. Thus, I α = –m g sin θ L
Or, α = − m g L sin θ/ I
Now if θ is small, sin θ can be approximated by θ and Eq. can then be written as,
α = − (m g L/I) θ i.e. angular acceleration α angular displacement (condition of SHM)
with α = − ω2 θ
𝑚𝑔𝐿 𝑚𝑔𝐿 2𝜋 𝐼 𝑀𝐿2
where 𝜔2 = or 𝜔 = √ time period T = = 2𝜋√𝑚𝑔𝐿 = 2𝜋√𝑚𝑔𝐿
𝐼 𝐼 𝜔

𝐿
TIME PERIOD T = 2𝜋√𝑔
11. QUESTION
(i) How does the time period of a simple pendulum changes if its length is increased?
(ii) What happens to the period if the mass is doubled?
(iii) What happens to the time period of a simple pendulum if the amplitude is doubled?
(iv) What happens to energy when amplitude is doubled?

ANSWER(i) The longer the length of string, the farther the pendulum falls; and therefore, the longer the
period, or back and forth swing of the pendulum. The greater the amplitude, or angle, the farther the
pendulum falls; and therefore, the longer the period.)
(ii) if the mass is doubled, the period increases by a factor of √2. A mass hanging from a spring is pulled
down from its equilibrium position through a distance A and then released at t = 0.
(iii) Time period is independent of the amplitude of vibration. Therefore, time period remains unchanged
on doubling the amplitude of oscillation. ... When amplitude is doubled, the maximum velocity becomes
twice. (iii) Acceleration at mean position is zero and is independent of the amplitude of vibration.
(iv) The energy transported by a wave is directly proportional to the square of the amplitude. So whatever
change occurs in the amplitude, the square of that effect impacts the energy. This means that a doubling
of the amplitude results in a quadrupling of the energy.
ADDITIONAL QUESTIONS (ONE MARK QUESTIONS:
1. Which of the following relationships between the acceleration a and the
displacement x of a particle involve simple harmonic motion?
(a) a = 0.7x (b) a = – 200 x2
(c) a = – 10x (d) a = 100 x3
ANSWER:
Only (c) i.e., a = – 10x represents SHM. This is because acceleration is proportional
and opposite to displacement (x).
2. The girl sitting on a swing stands up. What will be the effect on periodic time of
swing?
ANSWER:
The periodic time T is directly proportional to the square root of effective length of
pendulum (l). When the girl starts up, the effective length of pendulum (i.e. Swing)
decreases, so the time period T also decreases.

3. The soldiers marching on a suspended bridge are advised to go out of steps. Why?
ANSWER:The soldiers marching on a suspended bridge are advised to go out of
steps because in such a case the frequency of marching steps matches with natural
frequency of the suspended bridge and hence resonance takes place, as a result
amplitude of oscillation increases enormously which may lead to the collapsing of
bridge.
4. When a particle performing uniform circular motion of radius 10 cm undergoes the
SHM, what will be its amplitude?
ANSWER: 10 cm
5. Give some practical examples of S. H. M?
ANSWER: Some practical examples of S. H. M. are :-1) Motion of piston in a gas – filled
cylinder.2) Atoms vibrating in a crystal lattice.3) Motion of helical spring.
6. Define force constant of a spring.
ANSWER:The spring constant of a spring is the change in the force it exerts, divided
by the change in deflection of the spring.
7. Is the motion of a simple pendulum strictly simple harmonic?
ANSWER:It is not strictly simple harmonic because we make the assumption that
Sinθ =θ, which is nearly valid only if θ is very small.
8. At what distance from the mean position, is the kinetic energy in simple harmonic
oscillator equal to potential energy?

ANSWER: At
9. How is the time period of the pendulum effected when pendulum is taken to hills
Or in mines?
ANSWER: As T will increase. (since ‘g’ decreases)
10. Define angular frequency. Give its S.I. unit.
ANSWER:It is the angle covered per unit time or it is the quantity obtained by
multiplying frequency by a factor of 2 .
= 2π v, S.I. unit is rads s-1
11 How Will the time period of a simple pendulum change when its length is
doubled?

ANSWER:
12 In a forced oscillation of a particle, the amplitude is maximum for a frequency (ω1)
of the force, while the energy is maximum for a frequency (ω2) of the force. What
is the relation between and ?
Ans: Both amplitude and energy of the particle can be maximum only in the case of
resonance, for resonance to occur .
13 QUESTION:
A simple harmonic motion of acceleration 'a' and displacement 'x' is represented
by . What is the time period of S.H.M?

ANSWER: ,
14 What is the condition to be satisfied by a mathematical relation between time and
displacement to describe a periodic motion?
ANSWER: A periodic motion repeats after a definite time interval T. So, y(t) = y(t + T)
= y(t + 2T) etc.
CHAPTER 15

WAVES

GIST OF LESSON :
Motion of wave
It is the kind of disturbance which travel through a medium due to repeated motion of
particles of the medium , the disturbance being handed over from one particle to
particle. Wave transport energy and information
Longitudinal wave of motion.
In these types of wave , particles of the medium vibrate to and fro about their mean
position along the direction of propagation of wave .These are also called pressure wave.
Transverse wave motion
In this case particles of the medium vibrate up and down perpendicular to the direction of
propagation of wave.

Wavelength (λ) for longitudinal wave:- It is the distance between two successive
compressions and rarefactions.
Wavelength (λ) for transverse wave:- It is the distance between two successive crests
and troughs.
Relation between wave velocity v, wavelength λ and frequency ν.
V= λ X ν
Velocities of different waves:-
In general velocity of an elastic wave is given by v=√((Elasticity of
medium)/(Density of medium)) =√(E/ρ)

*For transverse waves in :-


In solids v=√(η/ρ) (η is modulus of rigidity)

In a stretched string v=√(T/m) (T is tension in the string and m is mass per unit
length of string)
*For longitudinal wave:-
In solids v=√((K+4η/3)/ρ) (K is bulk modulus of elasticity ,η is modulus of rigidity)
Solids in form of long rods v=√(Y/ρ) (Y is young’s modulus)
In liquid or gases v=√(K/ρ) (K is bulk modulus)
Newton’s formula:-According to Newton’s formula the velocity of sound is given by
v=√(P/ρ)
Laplace’s formula:- According to Laplace’s formula the velocity of sound is given by
v=√(ΥP/ρ) ,where γ is the ratio of two specific heat of gases.
Factor’s effecting speed of sound in gases:-
Effect of pressure:- Pressure has no effect on the speed of sound in gases.
Effect of density:- v=1/√ρ or v1/v2=√(ρ2/ρ1)
Effect of temperature:- v α√(T ) or v1/v2=√(T1/T2)
Temperature coefficient of velocity of sound is given by :-α=vt-v0/t
For air α=0.61ms-1c-1
Wave equation:- A plane progressive harmonic wave travelling along positive X- direction
may be represented as
Y= A sin (ωt - kx), where k=(2∏)/λ
𝑡 𝑥
Y=A sin 2∏(𝑇 - 𝜆 )
2∏
Y=A sin (𝑣𝑡 − 𝑥)
𝑇

Here v is the wave velocity, λ is wavelength, A is the amplitude of the oscillating particles
of the medium and y is the displacement of the particle at position x at any instant t.
Phase of waves:- Phase is the argument of sine or cosine function representing the wave.
Thus phase is given by
Φ=(ωt - kx) or 2∏(t/T-x/λ)
Principle of superposition of waves:- When a number of waves travelling through a
medium simultaneously ,the resultant displacement at any point of the medium is equal
to the vector sum of the displacements of the individual waves. If y ⃗1,(y2) ⃗…….,(yn) ⃗ are
the displacements of n waves superimposing each other at a point ,then the resultant
displacement at that point will be
y ⃗=y ⃗1+y2 ⃗ +⋯……Yn ⃗
Reflection of wave:- When a wave is reflected from a rigid boundary or a closed end ,it is
reflected back with a phase difference of 1800 but reflection at an open boundary takes
place without any phase change.
Stationary waves:- When two identical waves of the same frequency moving in the
opposite direction superpose ,they give rise to stationary waves .In this wave form does
not move.
Consider a plane progressive harmonic wave travelling along positive X direction.
Incident wave y1= A sin (ωt - kx) ,if the waves reflected from free end ,then the
Reflected wave will be y2= A sin (ωt + kx)
The stationary waves formed by the superposition of incident and reflected waves will be
Y= y1+ y2=2Acoskxsinωt. In this case points x=0,λ/2,3λ/2….. are antinodes.(where
amplitude of oscillation is maximum) and the points x=λ/4,3λ/4,5λ/4…… will be
nodes.(where particles of medium always remain at rest)
Similarly , When incident wave reflects from rigid boundary then the reflected wave will
be
Y2= -A sin (ωt + kx) and again they will superpose with incident wave to form stationary
wave.
Modes of vibration in a string:- On a string ,transverse stationary waves are formed due
to the superposition of direct and the reflected transverse waves.
For fundamental mode:- λ1= 2L, ν1=V/λ1 =1/2L √(T/m)=ν (say)
For second mode:- λ2=L, ν2=2ν ( Second harmonic or first overtone)
For third mode:- λ3=2L/3,ν3=3ν( Third harmonic or second overtone) and so on .
Organ pipe:- It is the simplest musical instrument in which sound is produced by setting
an air column into vibrations. Longitudinal stationary waves are formed on account of
superposition of incident and reflected longitudinal waves.
Modes of vibrations of closed organ pipes:- Longitudinal stationary waves are formed in
an organ pipe closed at one end.
𝑣 1 𝛾𝑃
Fundamental mode of vibration:- λ1=4L, ν1= λ1= 4𝐿 √ 𝜌 = ν (say)

For second mode:- λ2=4L/3 , ν2=3ν (Third harmonic or first overtone)


For third mode:- λ3=4L/5 , ν3=5ν (fifth harmonic or second overtone)
For pth mode:- λp=4L/(2p-1) , νp=(2p-1) ν (2p-1 harmonic or (p-1) overtone
Here ν1: ν2: ν3………=1:3:5:7 (only odd harmonics)
Modes of vibrations of closed organ pipes:- Antinodes are formed at both ends
,separated by one node in the fundamental mode.
Fundamental mode of vibration:- λ1,=2L, ν1’=V/( 〖λ1〗^,)=1/2L √(γP/ρ)=ν (say)
For second mode:- λ2’=L , ν2’=2ν (Third harmonic or first overtone)
For third mode:- λ3=2L/3 , ν3’=3ν (fifth harmonic or second overtone)
For pth mode:- λp,=2L/P , νp,=pν (pth harmonic or (p-1) overtone
Here ν1: ν2: ν3………=1:2:3:4…… (Both odd and even harmonics)
Beats:- The periodic variations in the intensity of sound due to superposition of two
sound waves of slightly different frequencies are called beats. One rise and one fall of
intensity constitute one beat. The number of beat per second is called beat frequency.
Beat Frequency:- ν beat=ν1-ν2

CASE STUDY BASED QUESTIONS:

CASE 1. TOPIC: Resonance Column Method

Figure shows schematically the diagram of a simple apparatus used in


laboratories to measure the speed of sound in air. A long cylindrical glass tube
(say about 1 m) is fixed on a vertical wooden frame. It is also called a resonance
tube. A rubber tube connects the lower end of this glass tube to a vessel which
can slide vertically on the same wooden frame. A meter scale is fitted parallel to
and close to the glass tube. The vessel contains water which also goes in the
resonance tube through the rubber tube. The level of water in the resonance
tube is same as that in the vessel. Thus, by sliding the vessel up and down, one
can change the water level in the resonance tube.
(i). In the state of first resonance, a pressure antinode is formed at the …………..
and a pressure node is formed near the …………

(A) water surface, open end.

(B) open end, water surface.

(C) open end, mid point of air column.

(D) water surface, mid point of air column.

(ii). The length of the air column, in first resonance, is

(A) equal to λ

(B) slightly greater than λ/4

(C) equal to λ/4

(D) slightly lesser than λ/4

(iii). If l1 and l2 are the first and second resonance lengths and ν is the frequency
of the tuning fork then velocity is sound waves through air column is

(A)ν(l1 – l2)

(B) 2ν(l1 – l2)

(C) 4ν(l1 – l2)

(D) ν(l1 – l2)/2

(iv). In a resonance column experiment, a tuning fork of frequency 400 Hz is


used. The first resonance is observed when the air column has a length of 20.0
cm and the second resonance is observed when the air column has a length of
62.0 cm. Find the speed of sound in air.

(A) 332 m/s

(B) 336 m/s


(C) 340 m/s

(D) 350 m/s

(v).In the above question, how much distance above the open end does the
pressure node form ?

(A) 1 cm

(B) 2 cm

(C) 3 cm

(D) 4 cm

CASE 2. TOPIC: Transverse and longitudinal waves are easily represented by sine waves.
Positions of maximum displacement from equilibrium are called antinodes A
and positions of zero displacement are called nodes N.

For a complete cycle of a wave of wavelength λ, the distance between any tow
consecutive nodes or antinodes is λ/2. Longitudinal stationary waves can be
produces in air columns in closed or open pipes. The closed end of such a pipe
is always a displacement node. The open end of the pipe is always a
displacement antinode.

(i). The auditory canal of the outer ear is essentially a 3.0 cm pipe closed at one
end by the ear drum. What is the fundamental frequency in air associated with
the air ?

(A) 1.1 x 102mHz

(B) 2.8 x 103 mHz

(C) 5.6 x 103mHz

(D) 1.7 x 105 mHz

(ii). When the pipe is open at both the ends, it resonates with a fundamental
frequency of 240 Hz. What is the fundamental frequency of the same pipe if it is
closed at one end ?

(A) 60 Hz (B) 120 Hz (C) 360 Hz (D) 180 Hz

(iii). A pipe resonates at 60 Hz, 100 Hz & 140 Hz. How long is the pipe ?

(A) 1.4 m
(B) 2.8 m

(C) 4.3 m

(D) 8.5 m

(iv). A flutist plays a note with a fundamental wavelength of 170 cm. How many
compression regions reach a nearby listener per second ?

(A) 170

(B) 200

(C)500

(D) Cannot be determined without knowing the distance between flutist and
listener.

(v) A pipe of length L is closed at both end. What is its fundamental


wavelength?

(A) L/2

(B) L

(C) 2L

(D) 4 L

CASE 3. TOPIC: A long string having a cross-sectional area 0.80 mm2 and density 12.5
g/cm3, is subjected to a tension of 64 N along the x-axis. One end of this string is
attached to a vibrator moving in transverse direction at a frequency of 20 Hz. At
t = 0, the source is at maximum displacement y = 1.0 cm.

(i) The speed of the wave travelling on the string is :

(a) 60 m/s (b) 80 m/s (c) 100 m/s (d) 120 m/s

(ii) For what tension is the string will the velocity of wave in the string be
doubled ?

(a) 32 N (b) 128 N (c) 256 N (d) 512 N

(iii) The equation for the wave travelling along the string is

(a) y = 1.0 sin [40πt – πx/2] cm

(b) y = 1.0 cos [ 40πt – πx/2] cm


(c) y = 1.0 sin [ 20πt – πx] cm

(d) y = 1.0 cos [ 20πt – πx/2] cm

(iv) The displacement of the particle of the string at x = 50 cm at time t = 0.05


sec is :

(a) 0.71 m (b) 0.071 m (c) 0.71 cm (d) 7.1 cm

(v) The velocity of the above mentioned particle at the same instant t = 0.05 sec
is :

(a) 89 cm/s (b) 0.89 cm/s (c) 8.9 cm/s (d) 0.089 cm/s

CASE 4. TOPIC : The displacement of the medium in a sound wave is given by the
equation

y1 = A cos (ax + bt) where A, a & b are positive constants. The wave is reflected
by an obstacle situated at x = 0. The intensity of the reflected wave is 0.64 times
that of the incident wave.

(i) The wavelength and frequency of the incident wave are :

(a) λ= π/a and n = b/2π

(b) λ= 2π/a and n = b/2π

(c) λ= 2π/a and n = b/π

(d) λ= π/a and n = b/π

(ii) The equation of the reflected wave is :

(a) y1 = 0.8 A sin (bt – ax)

(b) y1 = 0.8 A cos (bt – ax)

(c) y1 = - 0.8 A cos (bt – ax)

(d) y1 = - 0.8 A sin (bt – ax)

(iii) In the resultant wave formed after reflection, the maximum and minimum
values of the particle speed in the medium are :

(a) vmax = 1.8 Ab and vmin = 0.2 Ab

(b) vmax = 1.6 Ab and vmin = 0.1 Ab

(c) vmax = 1.4 Ab and vmin = 0.1 Ab


(d) vmax = 1.2 Ab and vmin = 0.2 Ab

(iv) The resultant wave as a superposition of a standing wave and a travelling


wave is expressed as :

(a) y = 2A sin bt sin ax + 0.2 A cos (bt – ax) (b) y = 2A cosbtcos ax + 0.2 A cos (bt
– ax)

(c) y = 2A cosbtcos ax + 0.2 A sin (bt – ax) (d) y = 2A sin bt sin ax + 0.2 A sin (bt –
ax)

(v) The positions of the antinodes of the standing waves are (r =0,1,2,3….) :

(a) (2r + 1) π/a (b) (r + 1) π/a

(c) (2r + 1)π/2a(d) (2r - 1)π/a

CASE 5. TOPIC : When two sound waves of slightly different frequencies travelling along
the same path in the same direction superpose each other, the intensity of
resultant sound at any point in the medium rises and falls alternately with time.
These periodic variations in the intensity of sound are beats. One rise and one
fall of intensity constitute one beat. The number of beats produced per second
is called beat frequency.

Beat frequency, νbeat= ν1 – ν2

For beats to be audible, the difference in the frequency of the two sound waves
should not be exceed 10. Due to persistence of hearing, our ears are not able to
distinguish between two sounds separate if the time interval between them is
less than (1/10)th of a second. Beats can be used to determine the frequency of
a tuning fork.

(i) For beats to be audible

(a) Frequency of sources should be different and amplitude should be same.

(b) Frequency of sources should be same and amplitude should be different.

(c) Frequency of sources should be different and amplitude should be different.

(d) Frequency of sources should be same and amplitude should be same.

(ii) A tuning fork A produces 4 beats/s with another tuning fork of frequency
320 Hz. On filing one of the prongs of A, 4 beats/s are again produced when
sounded with the same fork B. Then, the frequency of the fork A before filing is

(a) 328 Hz (b) 316 Hz (c) 324 Hz (d) 320 Hz

(iii) Following two wave trains are approaching each other : y1 = a sin 200 πt, y2
= a sin 208πt. The number of beats heard per second is

(a) 8 (b) 4 (c) 1 (d) 0

(iv) Two waves of wavelength 99 cm and 100 cm both traveling with velocity
396 m/s are made to interfere. The number of beats produced by them per
second are

(a) 1 (b) 2 (c) 4 (d) 8

(v) When the wavelength of sound changes from 1 m to 1.01 m, the number of
beats heard per second are 4. The velocity of sound is :

(a) 404 m/s (b) 4.04 m/s (c) 414 m/s (d) 400 m/s

ANSWERS(CASE STUDY BASED QUESTIONS):

ANS (i) ANS (ii) ANS (iii) ANS (iv) ANS (v)

CASE 1 A D B B A

CASE 2 B B C B C

CASE 3 B C B C A

CASE 4 B C A A C

CASE 5 A B B C A
TWO MARK QUESTIONS:

1. QUESTION: When the wire of a sonometer is plucked what is the nature of wave
in (i)the string (ii)in air?

ANSWER: The wave produced in the string are transverse and in air are longitudinal.

2. QUESTION: What is the angle between particle velocity and wave velocity in
(I)transverse wave (II)longitudinal wave ?

ANSWER: In transverse wave is π/2 in case of longitudinal wave is zero or π.

3. QUESTION: What are the essential points of difference between sound and light
waves?

ANSWER: (i) Sound waves are mechanical waves which are longitudinal in nature.
Light waves are electromagnetic waves which are transverse in character.

(ii) Sound waves need a material medium for propagation, whereas light waves do
not need any medium.

4. QUESTION: What should be minimum length of an open organ pipe for producing a
note of 110 Hz? The speed of sound is 33 m/s?

ANSWER: Frequency , v= 110 Hz

Speed of sound, ѵ = 330 m/s

V= ѵ/2L

L= ѵ/2v = 330/ 2X110 = 1.5 m

5. QUESTION: In an open organ pipe third harmonic is 450 Hz. What is frequency of
fifth harmonic?

ANSWER: Third harmonic = 3V= 450 Hz

: Fundamental frequency, V = 150 Hz

Fifth harmonic = 5v = 750 Hz


6. QUESTION: An incident wave is represented by Y(x, t)=20sin(2x-4t).Write the
expression for reflected wave

(i) From a rigid boundary

(ii) From an open boundary.

ANSWER: .(i) The wave reflected from a rigid boundary is

Y (x, t) = -20sin (2x+4t)

(i)The wave reflected from an open boundary is

Y (x, t) = 20sin (2x+4t)

7. QUESTION: Where to pluck and where to touch a stretched string to excite it to its
first overtone/ second harmonic?

ANSWER: The wire should be plucked at is one-fourth length and touch its center.

8. QUESTION: Name two instruments based on superposition of waves?

ANSWER: 1.Sonometer

2. Organ pipe.

9. QUESTION: Two sound sources produce 12 beats in 4 second. By how much do their
frequencies differ?

ANSWER: Number of beats produce per second,

B=12/4=3 beats per second so,

V1-V2=3Hz

10. QUESTION: State the factor on which the speed of a wave travelling along a
stretched ideal string depends.

ANSWER: The speed of the wave travelling on a string depends

1.the tension (T) in string and

2. Its mass per unit length (m).


THREE MARK QUESTIONS:

1. QUESTION: A hospital uses an ultrasonic scanner to locate tumors in a tissue. What is the
wavelength of sound in a tissue in which the speed sound is 1.7 km/s. the operating frequency
of the scanner is 4.2 MHz?

ANSWER: Here v = 1.7 km/s = 1.7 x 103 m/s

Ѵ = 4.2 1 MHz = 4.2 x 106 Hz

ƛ = Ѵ / v =1.7 x 103 / 4.2 x 106 = 4.047 X 10-4 m.

2. QUESTION: A bat emits ultrasonic sound of frequency of 100 kHz in air. If this sound meets a
water surface, what is the wavelength of (i) reflected sound , (ii) the transmitted sound?
Speed of sound in air = 340 m/s and is water = 1486 m/s?

ANSWER: Here v = 100 kHz = 105 Hz , va= 340 m/s , vw = 1486 m/s

Frequency of both the reflected and transmitted sound remains unchanged

Wavelength of reflected sound,

ƛ = va/ v = 340/ 105 = 3.4 X 10-3 m.

Wavelength of transmitted sound,

ƛ = vw/ v = 340/ 105 = 1.49 X 10-2 m.

3. QUESTION: A string of mass 2.50 kg is under a tension of 200 N .The length of the stretched
string is 20.0 m. If a transverse jerk is struck at one end of the string. How long does the
disturbance take to reach the other end?

ANSWER: Given m = 2.50/20.0 T= 200N

Speed of the transverse jerk is

Ѵ = √(T/m) = 40 m/s

Time taken by the jerk to reach the other end

distance /speed = 20/40 = 0.5 s

4. QUESTION: Write any three characteristics of stationary waves?


ANSWER: (i) in stationary waves, the disturbance does not advance forward. The conditions of
crest and trough merely appear and disappear in fixed position to be followed by opposite
condition after every half time period. (ii) The distance between two successive nodes or
antinodes is equal to half the wavelength. (iii) The amplitude varies gradually from zero at the
nodes to the maximum at the antinodes.

1
5. QUESTION: Find the temperature at which the velocity of sound in air will be 1 2 times the

velocity at 110 C?

ANSWER: Given , V1 = 3/2 V11

Or v0 √ (273 + t) / 273 = 3/2 v0 √ (273 + 11)

On squaring , (273 + t) / 273 = 9/4 * 284/273

On solving T= 336 c.

6. QUESTION: The speed of longitudinal wave `V` in a given medium of density ρ is given by the

formula, use this formula 𝑽 = √Ὺ𝑷⁄𝝆 to explain why the speed of sound in air.

(a) is independent at pressure


(b) increases with temperature and
(c) increases with humidity

ANSWER: (I) When Pressure change density also change in the same ratio

Speed of sound in a gas is directly proportional to the square root of its absolute temperature.

Speed of sound in is inversely proportional to the square root of its density, so sound travel
faster in moist air than dry air.

7. QUESTION: Prove analytically that in the case of open pipe of length L ,the frequencies of
vibrating air column are given by v= n (ѵ/ 2L) , where n is an integer?
ANSWER: Consider a cylinder pipe of length L lying along the x axis , with its open side at x=0
and x= L. The sound wave travelling along the pipe may be represented as

Y1= a sin (ωt – kx)

The wave reflected from the right open end may be represented as

Y2= a sin (ωt+kx)

There is no phase reversal on reflection from the open end because it is a free or loose
boundary. So the sign of A in the reflected wave is same as that in the incident waves.

By the principal os superposition, the resultant stationary wave is given by

Y = Y1 +Y2= a sin (ωt +kx) +a sin (ωt – kx) = 2AcosKxsinωt

For all values of t, the resultant displacement is maximum or antinodes are formed at the
open ends i.e at x=0 and x= L. This condition is satisfied if
+
Cos KL= 1 or KL = nΠ where n = 1,2,3……

Or 2 ΠL/ ƛ = nΠ or ƛn = 2L/n

The frequency of vibrations is given by

Vn = ѵ/ ƛn= nv/2L = n/2L* √(y P)/ ρ

For n = 1 , v1 = 1/2L * √(y P)/ ρ = v (say)

This is the smallest frequency of the stationary waves produced in the open pipe. It is called
fundamental frequency of first harmonic.

For n=2 , v2 = 2v/ 2L = 2v ( first overtone or second harmonic)

For n=3 , v3 = 3v/ 2L = 3v ( second overtone or third harmonic) and so on. The various modes
of vibrations of open pipes.

8. QUESTION: A string vibrates with a frequency of 200 Hz. Its length is doubled and its tension is
altered until it begins to vibrate with a frequency of 300 Hz. What is the ratio of new tension
to the original tension?
ANSWER: In first case 200 = 1/2L √ (T1/M) …………………1

In second case, 300 = 1/2* 2L √ (T2/M) ……………..2

Dividing equation 2 /1

300/ 200 = √ (T2/ T1) or

(T2/ T1) = 9:1

9. QUESTION: State the laws of vibrations of stretched strings?

ANSWER: Laws of transverse vibrations of a string : The fundamental frequency produced in a


stretched string of length L under tension T and having mass per unit length m us given by

V = 1/2L √ (T/M)

(i)Law of length :- The fundamental frequency of a vibration string is inversely proportional to


its length provided its tension and mass per unit length remain the same

V α 1/L

(ii) Law of Tension : :- The fundamental frequency of a vibration string is proportional to the
square root of its tension provided its length and the mass per unit length.

V α√T

(iii) Law of Mass : The fundamental frequency of a vibration string is proportional to the
square root of its mass per unit length provided that the length and tension remains same.

V α 1/√ M

10. QUESTION: The equation of a plane progressive wave is,

𝑦 = 10𝑆𝑖𝑛2𝜋(𝑡 − 0.005𝑥)Where y & x are in cm & t in second. Calculate the amplitude,


frequency, wavelength& velocity of the wave?
ANSWER: Given, y = 10𝑆𝑖𝑛2𝜋(𝑡 − 0.005𝑥)………….. (1)
𝑡 𝑥
Standard equation for harmonic wave is, 𝑦 = 𝐴 𝑆𝑖𝑛2𝜋(𝑇 − 𝜆)…………… (2)

1 1
Comparing equation (1) & (2), 𝐴 = 10, = 1, = 0.005
𝑡 𝜆

Amplitude A = 10cm
1
Frequency 𝜈 = 𝑇 = 1𝐻𝑧

1
Wavelength 𝜆 = 0.005 = 200𝑐𝑚

Velocity v = 𝜈 𝜆 = 1 x 200 = 200cm/s

11. QUESTION: (i) A steel rod 100 cm long is clamped at its middle. The fundamental frequency of
longitudinal vibrations of the rod is given to be 2.53 kHz. What is the speed of sound in steel?
(ii) A pipe 20 cm long is closed at one end. Which harmonic mode of the pipe is resonantly
exited by a 430 Hz source? Will this same source be in resonance with the pipe if both ends
are open? (Speed of sound = 340 m/s).

ANSWER: (i) For the fundamental mode,

λ = 2 L = 2 x 100 = 200 cm = 2m.

Frequency ν = 2.53 kHz = 2530 Hz

Speed of sound, v = νλ = 2530 x 2 = 5060 m/s

=5.06 km/s

(ii) Length of pipe L= 20 cm = 0.2 m

Speed of sound v= 340 m/s

Fundamental frequency of closed organ pipe


340
ν = v/4L = 4×0.2= 425 Hz sw can be excited

Fundamental frequency of open organ pipe


v 340
ν’ = 2𝐿 = 2×0.2 = 850 Hz

Hence source of frequency 430 Hz will not be in resonance with open organ pipe.
12. QUESTION: For a travelling harmonic wave, 𝑦 = 2𝐶𝑜𝑠(10𝑡 − 0.008𝑥 + 0.35) where x & y are
in cm and t in second. What is the phase difference between oscillatory motions at two points
𝜆 3𝜆
separated by a distance of (i) 4cm (ii) 0.5m (iii) 2 (𝑖𝑣) ?
4

ANSWER: 𝑦 = 2𝐶𝑜𝑠(10𝑡 − 0.008𝑥 + 0.35) … … … . (𝑖)

2𝜋𝑡 2𝜋𝑥
We know, 𝑦 = 𝐴𝐶𝑜𝑠 ( − + 𝜙) … … … . (𝑖𝑖)
𝑇 𝜆

2𝜋 2𝜋 2𝜋
From (i) & (ii), = 0.008, 𝜆 = 0,008 𝑐𝑚 = 0.80 𝑚.
𝜆

2𝜋 2𝜋
Phase difference, Δ𝜙 = ∗ 𝑝𝑎𝑡ℎ 𝑑𝑖𝑓𝑓𝑒𝑟𝑒𝑛𝑐𝑒 = ∗ Δ𝑥.
𝜆 𝜆

2𝜋
When Δ𝑥 = 4𝑐𝑚 , Δ𝜙 = ∗ 0.80 ∗ 4 = 3.2𝑟𝑎𝑑 .
2𝜋

2𝜋
WhenΔ𝑥 = 0.5𝑚, Δ𝜙 = 2𝜋 ∗ 0.80 ∗ 0.5 = 0.40𝑟𝑎𝑑.

𝜆 2𝜋 𝜆
When Δ𝑥 = 2 , Δ𝜙 = ∗ 2 = 𝜋𝑟𝑎𝑑.
𝜆

3𝜆 2𝜋 3𝜆 3𝜋
WhenΔ𝑥 = , Δ𝜙 = ∗ = 𝑟𝑎𝑑.
4 𝜆 4 2

13. QUESTION: What are beats? Prove that the number of beats produced per second by the two
sound sources is equal to the difference between their frequencies?

ANSWER: Beats;- the phenomena of regular rise & fall in the intensity of sound ,when two
waves of nearly equal frequencies travelling along the same line in the same direction super
impose each other is called beats

Mathematical treatment;- as we know that general equation of any wave i.e. displacement
relation along y- axis is given as

Y= a sinωt-------(1).
Let us consider two wave trains of equal amplitude & slightly different frequencies v1 & v 2
,then for two wave trains, above eq. becomes

Y1= a sin 2Π v1 t -------(1).

Y2= a sin 2Π v2t ------- (2).

As the two waves are in the same direction, then according to the principle of superposition,

The resultant displacement is

Y = Y1 + Y2

Y= a sin 2Π v1 t + a sin 2Π v2t = a [ sin 2Π v2t + sin 2Π v2t ]

Then eq. becomes

Y=2asinΠ (v1 + v 2 ) cosΠ (v1 - v 2 )t

Or Y= Z sin Π (v1 + v 2 )t -------(3)

Where Z= 2acosΠ (v1 - v 2 )t represents the amplitude of the resultant wave gives by
equation(3).

The amplitude ‘Z’ will be maximum ,when

cosΠ (v1 - v 2 )t= max= +/- 1 = cosnπ

Π (v1 - v 2 )t= nπ

Or t= -n/ (v1 - v 2 ) -----(4) where n=0,1,2…

Hence the resultant amplitude has maximum values at time =0,

T= 0 , t= 1/ (v1 - v 2 ) (for n=1), t= 2/ (v1 - v 2 (for n=2),……..&so on.

The time interval between two successive maximum of sound is

T= t1 – t (or t2 – t or t3 – t -….&soon.)

= 1/ (v1 - v 2 ) –0 = 1/ (v1 - v 2 ) =T= 1/ (v1 - v 2 ) & with that, the frequency of


maxima(N=1/T)is N= (v1 - v 2 ) ………… 5)

Similarly, the amplitude ‘Z’ will be minimum, when

cosΠ (v1 - v 2 )t=min=0=cos(2n+1) Π/2

Π (v1 - v 2 ) t =(2n+1) Π/2 where n=0,1,2,3,

Ort= 2n +1 / 2 (v1 - v 2 ) ----------(6)

Hence the resultant amplitude has minimum values at time

T = 1/ 2 (v1 - v 2 ) (for n=0), t1 = 3/ 2 (v1 - v 2 ) (for n=1),…..& so on The time interval between
It shows that the intensity of sound will be (v1 - v 2 ) times maximum & (v1 - v 2 ) times
minimum in one second i.e. (v1 - v 2 ) , beats will be heard in one second . also from eq.(5 & 7)

We say that, “no. of beats/sec=diff. in frequency of two source of sound

a)in determination of unknown frequencies

b)to detect the presence if dangerous gases in mines

c) in designing low frequency oscillator.

14. QUESTION: (I) Show that in a closed end organ pipe the first three harmonics are in the ratio
1: 3 : 5 ?

(II) When two tuning forks were sounded together, 20 beats were produced in 10 seconds.
On loading one of the forks with wax, the number of beats. If the frequency of unloaded fork
is 512 Hz. Calculate the frequency of other?

ANSWER: (i) First mode of vibrations. In this simplest mode of vibrations, there is only one
node at the closed end and one antinode at the open end. If L is the length of the organ pipe,
then,

L = ƛ1 / 4 or ƛ1 = 4L

Frequency ,

V1 = ѵ/ ƛ1 = ¼ L 𝑽 = √Ὺ𝑷⁄𝝆 = ѵ (say)

This frequency is called first harmonic or fundamental frequency.

(i) Second mode of vibrations. In this simplest mode of vibrations, there is only one node at
the closed end and one antinode at the open end. If L is the length of the organ pipe, then,

L = 3ƛ2 / 4 or ƛ2 = 4L/3

Frequency ,

V2 = ѵ/ ƛ2 = 3/4 L 𝑽 = √Ὺ𝑷⁄𝝆 = 3ѵ (say)

This frequency is called first overtone or third harmonic.

(i) Third mode of vibrations. In this simplest mode of vibrations, there is only one node at the
closed end and one antinode at the open end. If L is the length of the organ pipe, then,

L = 5ƛ3 / 4 or ƛ3 = 4L/3
Frequency ,

V3 = ѵ/ ƛ3 = 5/4 L 𝑽 = √Ὺ𝑷⁄𝝆 = 5ѵ (say)

Hence different frequencies produced in a closed organ pipe are in the ration 1 :3:5:7 ……. i.e.
only odd harmonics are present in a closed organ pipe.

(II) Beat frequency = 20/10 = 2/s

Possible unknown frequencies = 512 +/- 2 = 514 or 510 Hz

As loading decrease the frequency , also the beat frequency increases , so unknown
frequency = 510 Hz ( lower one)

ADDITIONAL QUESTIONS (ONE MARK QUESTIONS):

1. QUESTION: A harmonic wave in a medium has a period T and wave length ƛ. How are ƛ and T
related?

ANSWER: v = ƛ/T and ƛ=vT

2. QUESTION: Which type of waves do not require a material medium for their propagation?

ANSWER: Electromagnetic waves

3. QUESTION: What is the phase difference between two nearest crests (or troughs) ?

ANSWER: The phase difference is 2Π.

4. QUESTION: What is the wavelength range of audible sound and visible light ?

ANSWER: (i) wavelength range of audible sound in air is from 16.6 X 10 -3 to 16.6 m.

(ii) wavelength range of visible light in air is from 4 X 10 -7 to 8 X 10 -7 m.

5. QUESTION: What is the audible rage of frequency?

ANSWER: The audible range of frequency is 20 Hz to 20 KHz

6. QUESTION: What is the effect of pressure on the speed of sound in air?

ANSWER: The increase of pressure has no effect on the speed of sound in the air.

7. QUESTION: How far the consecutive nodes are separated from each other?

ANSWER: The separation between two consecutive node is ƛ/2


8. QUESTION: What is the distance between a node and the nearest antinode?

ANSWER: The distance between a node and the nearest antinodes is ƛ/4.

9. QUESTION: What is the phase difference between particles being on either side of a node ?

ANSWER: Π rad

10. QUESTION: What is the minimum frequency with which a string of length L stretched under
tension T can vibrate?

ANSWER: Minimum frequency

V= 1/ 2L √(T/m) , where m is the mass per unit length.

11. QUESTION: What is the essential condition for the formation of beats?

ANSWER: The difference in frequency of the two sound waves should not exceed 10 Hz.

12. QUESTION: If tension of a wire is increased to four times how is the wave speed changed?

ANSWER: As v = √T , therefore wave speed becomes twice.

13. QUESTION: An open organ pipe is closed at one end. How will the frequency of fundamental
note of the pipe change?

ANSWER: The fundamental frequency shall reduce to half.

14. QUESTION: Name two important properties of a material medium responsible for the
propagation of waves through it?

ANSWER: Properties of elasticity and inertia.

15. QUESTION: Two astronauts on the surface of moon talk to each other why?

ANSWER: Sound wave require medium for propagation as there is no atmosphere on the
moon hence the sound wave cannot propagate.
SAMPLE PAPER
TERM-2
CLASS XI
PHYSICS (042)
2021-22
SAMPLE PAPER-1
TERM -2 EXAMINATION, 2021-22
CLASS – XI
SUBJECT: PHYSICS
M. M. 35 TIME : 2 HOURS

GENERAL INSTRUCTIONS
(i) There are 12 questions in all. All questions are compulsory.
(ii) This question paper has three sections: Section A, Section B and
Section C.
(iii) Section A contains three questions of two marks each, Section B
contains eight questions of three marks each, Section C contains one
case study-based question of five marks.
(iv) There is no overall choice. However, an internal choice has been
provided in one question of two marks and two questions of three
marks. You have to attempt only one of the choices in such questions.
(v) You may use log tables if necessary but use of calculator is not allowed.

SECTION A
1 Why does the velocity increases when water flowing in a broader pipe enters a 2
narrow pipe?
OR
A barometer accelerating downwards reads 76 cm of Hg. What will be the possible
air pressure inside the jar?
2 What happens to the change in internal energy of a gas during (a) isothermal 2
expansion and (b) adiabatic expansion?
3 A simple harmonic motion of acceleration a and displacement x is represented by 2
a+4π²x=0. What is the time period of SHM?
4 What are parallel axis theorem and perpendicular axis theorem in respect of 3
moment of inertia of a lamina, Define the same and give mathematical expressions.
Draw diagrams to support your answer.
5 Find the density of the metal under a pressure of 20000 N/cm². Given that the 3
density of metal 11 g/ cm³ , bulk modulus of metal 8x10⁹N/m²
OR
A solid cube is subjected to a pressure of 5×10⁵ N/m². Each side of the cube is
shortened by 1% . Find volumetric strain and bulk modulus of elasticity of the
cube.
6 Explain the following. 3
a. Stainless steel cooking pans are preferred with extra copper bottom, why?
b. Why does aquatic animal survive under frozen lake?
c. Why does boiling take longer time at hills?
OR
What is Absolute zero, explain. At what temperature, do the readings of Celcius
and Fahrenheit scales coincide?

7 Derive an expression for the work done during the adiabatic expansion of an ideal 3
gas.
8 State the law of equipartition of energy. 3
Calculate the molecular kinetic energy of 1 gram of helium (molecular weight 4) at
127⁰C.
OR
Find the degrees of freedom of monatomic diatomic and triatomic gas molecules
9 A body of mass 0.1 kg is executing SHM according to the equation 3
Y = 0.5cos(100t+3π/4) m. Find
(i) the frequency of oscillation
(ii) initial phase
(iii) maximum velocity
(iv) maximum acceleration and
(v) total energy.
10 State the laws of vibrations of a stretched string. 3
11 What are beats? explain. 3
A tuning fork when vibrating along with a sonometer produces 6 beats per second
and the length of the wire is either 20 cm or 21cm. Find the frequency of the tuning
fork.
12 Read the passage given below and answer the following questions from 1 to 5. 5

It states that for the streamline flow of an ideal liquid through a tube, the total
energy (the sum of pressure energy, the potential energy and kinetic energy) per
unit volume remains constant at every cross-section throughout the tube.

If the liquid is flowing through a horizontal tube, then h is constant, then according
to Bernoulli’s theorem,

Bernoulli’s theorem is based on law of conservation of energy.


(i). Bernoulli’s equation for steady, non-viscous, incompressible flow expresses the
(a) conservation of linear momentum
(b) conservation of angular momentum
(c) conservation of energy
(d) conservation of mass
(ii). Applications of Bernoulli’s theorem can be seen in
(a) dynamic lift of aeroplane
(b) hydraulic press
(c) helicopter
(d) none of these
(iii). A tank filled with fresh water has a hole in its bottom and water is flowing out of
it. If the size of the hole is increased, then
(a) the volume of water flowing out per second will decrease
(b) the velocity of outflow of water remains unchanged
(c) the volume of water flowing out per second remains zero
(d) Both (b) and (c)
(iv). At what speed will the velocity head of stream of water be 40 cm?
(a) 28cm/s
(b) 280m/s
(c) 280 cm/s
(d) 0.28 cm/s

(v). What is the value of pressure head at the surface of liquid in a vessel.
(a) one atm
(b) one Pa
(c) one torr
(d) Zero Pa
MARKING SCHEME (Sample Paper-1)
Q.1 In a stream-line flow of a liquid, according to equation of continuity, av = a 2
constant, where 'a' is the area of cross-section and v is the velocity of the liquid
flow. When water flowing in broader pipe enters a narrow pipe, the area of
cross-section of the water decreases, therefore, the velocity of water increases.

OR
The net acceleration of the elevator accelerating downwards
=g−a 1 mark
Pressure inside the elevator
=hρ(g−a)=76×13.6×(g−a)13.6×g cm of Hg 1 mark
Clearly, this pressure will be less than 76 cm of Hg.
Q2 (i) In isothermal expansion ,temperature remains constant. Therefore 2
internal energy which is a function of temperature will remain
constant.
(ii) for adiabatic change dQ = 0 and hence first law of thermodynamics
becomes 0 = dU + dW
dW = - dU During expansion, work is done by the gas i.e. dW is
positive. Hence , dU must be negative. Thus ,in an adiabatic
expansion , the internal energy of the system will decrease.
Q3 2

expression 1 mark, correct calculation and answer 1 mark


Q4 Correct explanation 1 mark 3
Correct expression 1 mark
Correct diagram 1 mark
Q5 3

OR
Q6 For each Correct explanation 1 mark (1+1+1=3)
OR

Correct explanation 1 mark

Q7 1 mark for correct concept , expression and calculation total 3 marks 3


Q8 1 mark for correct definition 3

OR
For correct calculation of degree of freedom of each mono, di and tri atomic gas
1 mark (3 marks)
Q9

Q10 Correct explanation of all the six point. 3


(1/2 marks*6=3 marks)
Q11 Correct definition 1 mark 3
1
ʋ+6= 2 × 20 √(𝑇/𝑚) 1/2 mark
1
ʋ-6= 2 × 2𝑙 √(𝑇/𝑚) 1/2 mark

ʋ+6 21
=
ʋ−6 20
or 1/2 mark
(ʋ+6)x(ʋ-6)= (ʋ-6)21
On solving ʋ= 246Hz. 1/2 mark
Q12 Case Study 5
(i) (c) conservation of energy 1
(ii) (a) dynamic lift of aeroplane 1

(iii) (b) the velocity of outflow of water remains unchanged 1

(iv) (c) 280 cm/s 1

(v) (a) one atm 1


SAMPLE PAPER-2
TERM -2 EXAMINATION : 2021-22
CLASS : XI
SUBJECT : PHYSICS (042)
TIME : 2 HOURS M.M. 35

General Instructions:

(i) There are 12 questions in all. All questions are compulsory.


(ii) This question paper has three sections: Section A, Section B and Section C.
(iii) Section A contains three questions of two marks each, Section B contains eight questions of three marks each, Section C
contains one case study-based question of five marks.
(iv) There is no overall choice. However, an internal choice has been provided in one question of two marks and two
questions of three marks. You have to attempt only one of the choices in such questions.
(v) You may use log tables if necessary but use of calculator is not allowed.

SECTION A
1 Why the passengers are advised to remove the ink from their pens while going up in an aeroplane?
2 What is an isothermal process? What are the essential conditions for an isothermal process to take place?
OR
What is an adiabatic process? What are the essential conditions for an adiabatic process to take place ?
3 The amplitude of a simple harmonic oscillation is doubled. How does it affect

Page 1 of 4
(i) periodic time (ii) Maximum velocity (iii) maximum acceleration and (iv) maximum energy?
SECTION B
4 (i) It is difficult to open the door by pushing it or pulling it at the hinge. Why?
(ii) A ballet dancer stretches her hand out for slowing down. Name the conservation obeyed.
(iii) Why is moment of inertia also called rotational inertia?
5 A wire stretches by a certain amount under a load. If the load and radius of wire are both increased to four times, find the
elongation in the wire.
6 Define the coefficient of thermal expansion and derive the relation between coefficient of volumetric and linear thermal
expansion.
7 Can we increase the temperature of a gas without supplying heat to it? A gas is suddenly compressed to 1/4th of its
original volume. Calculate the rise in temperature when original temperature is 27 0 C. γ = 1.5.
8 A particle is moving in SHM in a straight line. When the distance of the particle from equilibrium position has values x 1
1⁄
𝑥̈ 22 −𝑥12 2
and x2, the corresponding values of velocities are u1 and u2. Show that the time period of vibration is T = (𝑢2 −𝑢2 )
1 2

OR
When the displacement is one half of the amplitude, what fraction of the total energy is kinetic energy and what fraction is
potential energy in SHM? At what displacement is the energy half kinetic and half potential?
9 State law of equipartition of energy. Using this law calculate the ratio of specific heat of diatomic gas.
10 Write Newton’s formula for the speed of sound in a gas. Why and what correction was applied by Laplace in this formula?
11 Show that in case of stretched string the first four harmonics are in the ratio of 1:2:3:4 .
OR
Show that in case of closed organ pipe the first four harmonics are in the ratio of 1:3:5:7.

Page 2 of 4
SECTION C
12 CASE STUDY: A spray gun is shown in the figure where a piston pushes air out of a nozzle. A thin tube of uniform cross
section is connected to the nozzle. The other end of the tube is in a small liquid container. As the piston pushes air through
the nozzle, the liquid from the container rises into the nozzle and is sprayed out. For the spray gun shown, the radii of the
piston and the nozzle are 20 mm and 1 mm respectively. The upper end of the container is open to the atmosphere.

(i) If the piston is pushed at a speed of 5 ms-1 , the air comes out of the nozzle with a speed of
(a) 0.1 ms-1
(b) 1 ms-1
(c) 2 ms-1
(d) 8 ms-1

(ii) If the density of air is ρa and that of the liquid ρl than for a given piston speed the rate (volume per unit time) at which
the liquid is sprayed will be proportional to

Page 3 of 4
(iii) While studying about fluid mechanics, the equations and postulates stand for the fluids which are
(a) non-viscous only
(b) incompressible only
(c) non-viscous and incompressible
(d) liquids only

(iv) Bernoulli’s equation is an example of conservation of


(a) energy
(b) momentum
(c) angular momentum
(d) mass

(v) The rate of flow of liquid through an orifice of a tank does not depend upon
(a) the size of orifice
(b) density of liquid
(c) the height of liquid column
(d) acceleration due to gravity

Page 4 of 4
MARKING SCHEME(SAMPLE PAPER-2)
CLASS : XI
SUBJECT : PHYSICS (042)
TIME : 2 HOURS M.M. 35

Q.1 As atmospheric pressure decreases with height. Since ink inside the pen is 2
filled at the atmospheric pressure existing on the surface of earth, it tends to
come out to equalise the pressure. This can spoil the clothes of the
passengers, so they are advised to remove the ink from the pen.
Q.2 A process in which temperature remains constant is called isothermal ½+ ½ +1
process.
The essential conditions for an isothermal process to take place are : (i)
The walls of the container must be perfectly conducting to allow free
exchange of heat between the gas and the surroundings. (ii) The process of
compression or expansion should be slow, so as to provide sufficient time
for the exchange of heat.
OR
A process in which temperature remains constant is called adiabatic
process.
The essential conditions for an adiabatic process to take place are : (i) The
walls of the container must be perfectly insulating. (ii) The process of
compression or expansion should be so fast.
Q.3 𝑙 ½+ ½+½+
(i) As T= 2𝜋√𝑔 , does not change
½
(ii) V = ωA so velocity becomes double
(iii) As amax = ω2A acceleration becomes double
1
(iv) Total energy E = 2 𝑚(2𝜋𝜈)2 𝐴2 = 2𝜋 2 𝑚𝜈 2 𝐴2 so energy becomes four

times.
Q.4 (i) at the hinge, torque is min. as 𝜏⃗ = 𝑟⃗ × 𝑃⃗⃗ and r is min. 1+1+1
(ii)Conservation of angular momentum
(iii) It gives the measure of inertia in rotational motion so it is rotational
inertia.
Q.5 𝐹𝑙 𝐹𝑙 ½+½
Young’s modulus, 𝑌 = ⇒ 𝛥𝑙 ⋅= -------(i)
𝜋𝑟 2 𝛥𝑙 𝜋𝑟 2 𝑌

When load F and radius r are increased to four times,


4𝐹𝑙 4𝐹𝑙
𝑌= ⇒ 𝛥𝑙′ = --------(ii)
𝜋(4𝑟)2 𝛥𝑙′ 16 𝜋𝑟 2 𝑌 ½+½
Using eq. (i) and (ii),
we get ∆l’ = ∆ l /4 Clearly, if the load and radius are increased to four times,
the final elongation will be 1/4 of the initial elongation. 1
Q.6 Coefficient of thermal expansion:- Change in dimension per unit original 1+2
dimension per degree Celsius.
To prove γ = 3 α
Q.7 Yes, the temperature of a gas can be increased by compressing the gas ½+½
under adiabatic conditions.
Given V2 = V1/4 T1 = 27+273 = 300 K γ = 1.5 1½
γ-1 γ-1 1.5-1
T2V2 = T1V1 So T2 = 300(4) = 600 K
½
So T2 – T1 = 600-300 = 300 K = 3000C
Q.8 As V2 = ω2(A2 – y2) ½+½
U12 = ω2(A2 – X12) ------(i) , U22 = ω2(A2 – X22) ------(ii)
Subtracting (i) and (ii) U12 - U22 = ω2(X22 – X12) and ω = 2π/ T
1⁄
𝑥̈ 22 −𝑥12 2
So T = ( 2 2) 2
𝑢1 −𝑢2

OR
Fraction of total energy which is KE = KE/TE
1 𝑎2
𝑚𝜔2 (𝑎 2 −𝑦 2 ) 𝑎 2 −𝑦 2 𝑎2 − 3 1
2 4
= 1 = 𝑎2
= 𝑎2
=4
𝑚𝜔2 𝑎 2
2

Fraction of total energy which is potential = P.E./T.E.


1 𝑎2
𝑚𝑤 2 𝑦 2 𝑦2 1
2 4
1 = 𝑎2
= 𝑎2
=4 1
𝑚𝜔𝑧 𝑎 2
2

K.E. = P.E.
1 1
2
𝑚𝜔2 (𝑎2 − 𝑦2) = 2
𝑚𝜔2 𝑦 2 on solving y = a / √2 1
Q.9 Law of equipartition:- At thermal equilibrium, at temperature T each mode of 1
1
energy contributes an average energy equal to 2 𝑘𝑇.

for diatomic gas γ = Cp/ Cv = 7/5


2
Q.10 According to Newton’s sound waves travel through gas under isothermal 1 +1+ 1
𝑝
conditions So, V = √𝜌

Laplace pointed out that sound travels through adiabatic condition not under
isothermal because (i) temp. rises in the region of compression and falls in
regions of rarefactions. (ii) gas is a poor conductor of heat. (iii) compression
and rarefactions are formed so rapidly.
𝛾𝑃
Hence V = √ 𝜌
Q.11 Correct diagram of four modes ¼+¼+¼ +¼
Correct calculation ½+½+½+½
Q.12 (i) C 1+1+1+1+1
(ii) A
(iii) C
(iv) A
(v) D

You might also like